ORTHOPEDIC MCQS ONLINE OB 20 2B RECONSTRUCTION

ORTHOPEDIC MCQS ONLINE OB 20 2B RECONSTRUCTION 

156) A 66-year-old male undergoes the procedure shown in figures A and B. After 4 years, he develops progressive pain and limitations in his daily function that is refractory to conservative measures. He is indicated for conversion to a total knee replacement with almost complete relief of his symptoms postoperatively. What preoperative factor likely led to the subsequent failure?

 

  1. ACL reconstruction 20 years ago

  2. Spontaneous osteonecrosis of the knee involving the medial compartment

  3. A history of inflammatory arthritis

  4. Body weight of 80kg

  5. 5-degree varus deformity Corrent answer: 3

Inflammatory arthritides are considered contraindications for a partial knee replacement due to the likely progression of arthritis in the native compartments.

 

Unicompartmental arthroplasty (UKA) is generally considered an excellent procedure provided that appropriate surgical indications are used for patient selection. Classic indications for UKA include unicompartmental disease in patients older than 60 with relatively low activity demands. Additionally, the patients should weigh less than 82 kg, have minimal pain at rest, have motion

>90 degrees, varus deformity <10 degrees, and flexion contractures <10 degrees. While most of these classic indications are still used today, recent studies are showing excellent results in younger, heavier, and more active patients. Inflammatory arthropathy is a contra-indication to unicompartmental knee replacement due to the high risk of continued arthritis progression in the other compartments. These patients should undergo total knee replacement instead.

 

Heyse et al. retrospectively reviewed the results of unicompartmental arthroplasty for spontaneous osteonecrosis of the knee (SONK). 75.7% of patients were "very satisfied" with the procedure and 21.6% were "satisfied." At 15 years, 90.6% of unicompartmental arthroplasties had not been revised. Overall, the authors feel it is an excellent procedure for SONK.

 

Price et al. reviewed 682 medial Oxford UKA knees for 20-year survival. They found the all-cause revision survival rate was 91%. 29 revision procedures occurred at a mean time of 3.3 years postoperatively. The most common cause of revision was lateral arthrosis (34%), followed closely by component loosening (31%). They concluded that Oxford medial UKA has good survival even into the second decade postoperatively.

 

Figure A and B show AP and lateral images of a left knee status post medial unicompartmental knee arthroplasty. Specifically, this is the Oxford UKA system. Illustration A is a T2-weighted MRI demonstrating SONK.

 

Incorrect Answers:

Answer 1: A prior ACL reconstruction is not a contraindication to performing a

unicompartmental arthroplasty. The absence of an ACL is a contraindication for mobile-bearing UKA.

Answer 2: Unicompartmental arthroplasties have shown excellent results for patients with SONK.

Answer 4: Historically a preoperative requirement was body weight less than 82kg, however more current research suggests that heavier patients still have good outcomes with UKA.

Answer 5: 5-degree varus deformity preoperative is within the limits of what can be accepted.

 

 

 

 

 

 

OrthoCash 2020

 

  1. Which of the following is the most common cause of early revision surgery (<20 weeks) following a hip resurfacing arthroplasty?

    1. Periprosthetic fracture

    2. Rupture of abductors

    3. Dislocation

    4. Heterotopic ossification

    5. Post-operative stiffness

     

    Corrent answer: 1

    Periprosthetic fracture, specifically femoral neck fracture, is the most common cause of early revision less than 20 weeks following surgery.

     

    The rate of femoral neck fractures following hip resurfacing varies, but most literature reports a rate of 1%. A majority of these fractures happen in the early post-operative period and are the most frequent cause of revision surgery within several months following surgery. The cause is usually multifactorial, but placing the femoral implant in varus, osteonecrosis, and notching have been proven risk factors for fracture.

     

    Little el al. report on 377 patients undergoing hip resurfacing. 13 required revision including 8 for fracture of the femoral neck and 3 for loosening of a component. Evidence of osteonecrosis was seen in two of these cases, leading the authors to believe it may contributed to fracture.

     

    Illustration A shows a comparison of a typical total hip replacement and a hip resurfacing arthroplasty. Illustration B shows notching of the femoral neck, a known cause of femoral neck fracture following hip resurfacing. Illustration C shows a femoral neck fracture in a patient with a hip resurfacing.

     

     

     

     

     

     

     

     

     

     

     

    OrthoCash 2020

  2. A 55-year-old male undergoes a revision total knee arthroplasty of an implant that is only 3 years old. At the time of surgery, the tibial polyethylene liner shows catastrophic delamination and cracking. What is the most likely cause of this extensive, accelerated wear of the polyethylene liner?

    1. Sterilization in ethylene oxide

    2. Gamma irradiation of the polyethylene liner in the presence of air

    3. Gamma irradiation of the polyethylene liner with vacuum packaging

    4. Gamma irradiation of the polyethylene liner in nitrogen

    5. Gamma irradiation of the polyethylene liner in argon Corrent answer: 2

    Irradiation of polyethylene in air (i.e. oxygen present) has been shown to be a risk factor for catastrophic failure after total knee replacement.

     

    Free radicals are generated when polyethylene is irradiated in the presence of air. Initially, these free radicals result in cross-linking. However, if the polyethylene is exposed to these free radicals for an extended period of time, delamination, cracking, and catastrophic failure may ensue. The industry has completely abandoned this method of sterilization as a result. Currently, the standard of care is irradiation of polyethylene in an inert gas (e.g. argon, nitrogen or vacuum packaging). The amount of oxidative products when polyethylene is sterilized in the absence of oxygen is much less and does not lead to catastrophic failure.

     

    Sterilization without irradiation is another option (ethylene oxide). When this occurs, there is no cross-linking and thus the increased wear properties are lost. However, since there is no oxidization, you do not have the risk of catastrophic failure as seen in those liners irradiated in the presence of oxygen.

     

    The cited reference by McNulty et al. from Orthopedics discusses the influence of sterilization methods on wear performance. They found that gamma irradiation and storage of the polyethylene components in an essentially oxygen-free environment imparted by gamma irradiation in a vacuum foil pouch (GVF) protects the components from oxidization.

     

    Illustration A shows a polyethylene liner that has undergone catastrophic wear as a result of irradiation in the presence of oxygen.

     

    Incorrect Answers:

    Answer 1: Sterilization in ethylelne oxide has not been shown to lead to

    catastrophic wear, although wear properties are less than gamma irradiation in the absence of air.

    Answer 3: Vacuum packaging minimizes oxidation while still allowing some cross-linking.

    Answer 4: Unlike air, nitrogen is an inert element that minimizes formation of free radicals.

    Answer 5: Unlike air, argon is an inert element that minimizes formation of free radicals.

     

     

     

     

     

     

    OrthoCash 2020

     

  3. Which of the following interventions reduces osteolysis around distal portion of the femoral stem when performing a total hip arthroplasty?

    1. Use of an extended offset femoral neck component

    2. Use of a proximal circumferentially coated ingrowth stem

    3. Use of a collared stem

    4. Use of a long femoral stem

    5. Ensuring that the stem fills the diaphysis of the femur Corrent answer: 2

    Osteolysis of the femur is caused by activation of macrophages by microscopic polyethylene particles within the "effective joint space", defined as any area where joint fluid can come into contact with bone. This can occur above the acetabular cup, through screw holes, and down the femoral shaft around the prosthetic stem. Ideally, with a cementless stem, both the proximal and metaphyseal femur are well filled by the prosthesis. Collared stems are used to augment poor calcar bone quality or bone loss.

    Sinha et al showed in a retrospective review of 101 hips with cementless circumferentially coated femoral stems no distal femur osteolysis occurred, but 82% showed “evidence of proximal femur stress shielding”, though only 38% showed proximal femoral osteolysis.

     

     

     

    OrthoCash 2020

     

  4. During a minimally invasive approach to total hip arthroplasty a femoral periprosthetic fracture occurs. Which of the following steps is crucial to properly treat this complication?

    1. Transitioning to an extensile approach to adequately visualize and reduce the fracture

    2. Limiting post-operative weight bearing

    3. Switching to a cemented femoral stem to avoid the stresses created during press-fit fixation

    4. Delaying the arthroplasty until the fracture has healed

    5. Supplementing the fracture with autograft Corrent answer: 1

    Proper treatment of an intraoperative femoral fracture during total hip arthroplasty involves adequate exposure, anatomic reduction, and bypassing the fracture site by 2 cortical diameters of the femur with a long stem. This may involve repositioning the patient on the table if the arthroplasty is performed in the supine position. Minimally invasive surgical techniques have been developed to insert the components through smaller exposures and less soft tissue dissection. The purported advantages include faster rehabilitation, less blood loss, shorter hospital stays, and better cosmesis. However, complications an arise if the surgeon sacrifices surgical exposure and visualization.

     

    Fehring et al review 3 cases of total hip arthroplasty performed through minimally invasive techniques with catastrophic outcomes. Intra-operative fracture, chronic instability, and death were all identified.

     

     

     

    OrthoCash 2020

     

  5. A 70-year-old man underwent total hip arthroplasty 4 months ago and has experienced 3 dislocations. Radiographs reveal no failure of the hardware and an acetabular component that has an abduction

    angle of 40 degrees and a version of 10 degrees retroverted. What is the most appropriate treatment for the recurrent dislocations?

    1. hip abduction brace

    2. revision of the acetabular liner to a constrained type

    3. revision of the entire acetabular component

    4. revision of the femoral head to a larger size

    5. revision to an extended offset prosthesis Corrent answer: 3

    Per Dorr et al: post-operative hip instability can be caused by several factors: soft tissue imbalance, component malposition, or position. Component malposition, as in this case, should be treated with revision of the offending component. In this case the acetabulum was placed in retroversion when it should have been 15-20 degrees anteverted. None of the other options addresses the cause of the instability. According to Morrey, the most signficant risk factors to instability are prior hip surgery, trochanteric nonunion, and posterior surgical approach. He wrote that the most reliable way to correct instability is to reorient a retroverted acetabular cup.

     

     

     

    OrthoCash 2020

     

  6. In patients with sickle cell disease and asymptomatic osteonecrosis of the femoral head identified with magnetic resonance imaging, what percentage will eventually go on to femoral head collapse?

    1. < 10%

    1. 11% to 25%

    2. 26 to 50%

    3. 51 to 75%

    4. > 75%

     

    Corrent answer: 5

     

    Untreated asymptomatic osteonecrosis of the femoral head in patients with sickle cell disease has a > 75% likelihood of progression to pain and collapse.

     

    In contrast to asymptomatic osteonecrosis in patients without sickle cell disease, patients with sickle cell disease have a high incidence of progression to collapse and functional impairment secondary to pain (33% compared to 75%, respectively). In patients with sickle cell disease who present with a symptomatic hip, the contralateral (presumably asymptomatic) hip should be screened carefully and periodically for signs of disease. Given the high rate of

    progression, some argue for operative treatment of the asymptomatic hip at the same time the symptomatic hip is being treated.

     

    Hernigou et al. studied patients with sickle cell disease who had symptomatic avascular necrosis (AVN) in one hip and asymptomatic AVN in the contralateral hip. Their goal was to understand the natural history of the asymptomatic side. Ninety-one percent eventually developed pain and 77% developed collapse. Collapse was preceded by symptoms of pain in all patients.

     

    Aguilar et al. report bone and joint disorders are the most common cause of chronic pain in patients who have sickle cell disease, and that the femoral head is the most common area of bone destruction in sickle cell patients.

     

    Incorrect Answers:

    Answers 1-4: Patients with sickle cell disease and osteonecrosis of the femoral head have a high rate of progression to painful collapse.

     

     

     

    OrthoCash 2020

     

  7. All of the following are contraindications to medial unicondylar knee arthroplasty EXCEPT:

    1. Flexion contracture greater than 10 degrees

    2. Varus deformity greater than 10 degrees not correctable with stress testing

    3. Lateral knee joint line pain

    4. Rheumatoid arthritis

    5. Osteonecrosis of the medial femoral condyle Corrent answer: 5

      Contraindications to performing a unicompartmental/unicondylar knee arthroplasty include: inflammatory arthritis, fixed varus/valgus deformity more than 10 degrees, flexion contracture more than 10 degrees, less than 90 degrees of flexion pre-operatively, ACL insufficiency, significant arthritis in the other compartments, and joint subluxation of more than 5mm. Marmor et al reviewed 34 cases of UKA performed for osteonecrosis of the knee. At 5.5 year follow up they reported 90% good or excellent results. They did notice a trend to worse clinical results in cases of lateral compartment osteonecrosis. Parrate et al reviewed 31 cases of UKA's completed for osteonecrosis of the knee. At 12 year follow-up, they reported 96% good clinical outcomes and a 96% survivorship as well.

       

       

      OrthoCash 2020

       

  8. A 65-year-old male presents with a painful right total knee arthroplasty, which was performed ten years ago. CRP is 15 mg/L. Knee aspiration reveals a purulent fluid with 3,100 WBC's with 83% PMN's. Culture results are pending. Which of the following is the best management option?

    1. Physical therapy, ice, and follow-up evaluation in 2 weeks

    2. Repeat aspiration if cultures are positive

    3. Oral antibiotics

    4. Intravenous antibiotics

    5. Surgical explant of components Corrent answer: 5

    This patient is infected and requires explantation to cure the infection.

     

    Ghanem et al reviewed 161 infected TKA's vs 268 aseptic failures and discovered synovial fluid aspiration with WBC of >1100 cells/mm3 and PMN > 64% are suggestive of infection. When both tests yielded results below their cutoff values, the negative predictive value was 98.2% (95% confidence interval, 95.5% to 99.5%), whereas when both tests yielded results greater, infection was confirmed in 98.6% (95% confidence interval, 94.9% to 99.8%) of the cases. Thus, according to the most recent literature, WBC >1100 and PMN > 64% should be considered suggestive of infection in a TKA.

     

    In an earlier, smaller study, Mason et al reported on 31 septic TKA failures and concluded that aspirates with a WBC count greater than 2,500 and 60% PMNs are highly suggestive of infection.

     

    In another small study focused on THA, Schinsky et al reviewed 55 infected THA's and found that when combined with an elevated erythrocyte sedimentation rate and C-reactive protein level, the optimal cut-point for the synovial fluid cell count was >3000 white blood cells/mL, which yielded the highest combined sensitivity, specificity, positive predictive value, negative predictive value, and accuracy of the tests studied.

     

     

     

    OrthoCash 2020

     

  9. After insertion of the trial components in a total knee replacement, the surgeon finds that he is unable to fully extend the knee and that the tibial tray lifts-off when the knee is flexed past 90

    degrees. What intervention should be taken to achieve a knee that is balanced in flexion and extension?

    1. Augment the distal femur

    2. Resect more distal femur

    3. Resect more proximal tibia

    4. Downsize the femoral component

    5. Increase polyethylene liner thickness Corrent answer: 3

    In this situation, the knee is both tight in flexion (tibial tray lift-off) and extension (unable to fully extend). Of the options listed, only resection of the proximal tibia will provide more room in both flexion and extension.

     

    Proper sagittal plane balancing in total knee arthroplasty can help prevent post-operative stiffness and/or instability through the flexion arc. In a

    cruciate-retaining knee arthroplasty, balancing is potentially more complicated given the presence of the posterior cruciate ligament. If the knee is too tight in flexion, the trial tibial tray can be seen "lifting-off." If the knee is too loose in flexion, the tibial tray can be easily extracted from beneath the femur when the knee is flexed to 90 degrees. If the knee is both tight in flexion and extension, which is the case here, the correct treatment should be to resect more proximal or downsize the polyethylene.

     

    Illustration A shows normal tibial tray seating at the time of total knee arthroplasty.

     

    Illustration B shows tibial tray lift off with flexion. This is due to the femoral component impinging on the tibial tray posteriorly and causing lift off of the tray anteriorly.

     

    Incorrect Answers:

    Answer 1: Augmenting the distal femur will increase the amount of tightness in extension.

    Answer 2: Resecting more distal femur will decrease tightness in extension, but will not change the tightness in flexion.

    Answer 4: Downsizing the femoral component will only affect the flexion space, not the extension space.

    Answer 5: Increasing the polyethylene thickness will increase tightness in both flexion and extension. In this case, we want to do the opposite.

     

     

     

     

     

    OrthoCash 2020

     

  10. A 61-year-old male had a minimally-invasive unicompartmental knee replacement 8 months ago. He did well until recently when he developed persistent right knee pain that is worse with weight bearing. He denies any fevers or recent trauma. He does report that he had been exercising more over the past few months in an attempt to lose weight. WBC, ESR and C-reactive protein levels are normal. An AP radiograph and bone scan are shown in Figure A and B. What is the most likely cause of his symptoms?

     

     

     

     

     

     

    1. Component failure/ polyethylene failure

    2. Infection

    3. Pes anserine bursitis

    4. Stress fracture

    5. Complex regional pain syndrome Corrent answer: 4

    The patient has knee pain following a unicompartmental knee replacement with a normal radiograph and uptake under the prosthesis on bone scan. This is consistent with a stress fracture.

     

    Brumby et al. reported on tibial plateau stress fractures after UKA due to the guide pin holes placed in the proximal tibia. All of their cases required a revision to a TKA. They recommend that if 3 or more pin holes are created that the surgeon be aware of this potential complication. Pandit et al. report similarly on tibial plateau fractures after medial Oxford UKA.

     

    Hamilton et al. retrospectively reviewed minimally invasive versus open unicompartmental arthroplasty. They found the MIS group to have a higher incidence of aseptic loosening and re-operations when compared to the open group.

     

    Incorrect Answers: The patient's presentation is not consistent with infection, pes bursitis, or complex regional pain syndrome. Radiographs do not show component or polyethylene failure.

     

     

     

    OrthoCash 2020

     

  11. An 80-year-old man has a Charnley all-polyethylene acetabular shell shown in Figure A. When defining failure as revision for any reason, what is the failure rate at 35 years for this type of prosthesis?

     

     

     

    1. Less than 10%

    2. 20% to 30%

    3. 40% to 50%

    4. 50% to 75%

    5. Greater than 75%

     

    Corrent answer: 2

     

    The estimated failure rate at 35 years for all polyethylene acetabular shells is 20-30%.

     

    Della Valle, et. al, reported a 77.3% survivorship for the component at a mean of 21 years, with revision or definite loosening as an endpoint.

     

    Kavanaugh et. al, reported that the probability of surviving 20 years without revision of the components was 84%. They did note that there is a significantly increased probability of revision based on the age of the patient at the time of the index THA(27% for <59 y.o., 13% for 59-65 y.o., 7.5% for 65-

    70 y.o., and 12% for >70 y.o.)

     

    Callaghan, et. al. has demonstrated the remarkable durability of cemented Charnley total hip replacements over a span of three decades, with 88% of the

    original prostheses intact at 20 years, and 78% at 35 years. In conclusion, the rate of revision definitely varies, but should be somewhere between 20-30%.

     

     

     

    OrthoCash 2020

     

  12. Following a left total knee arthroplasty, all of the following can cause the condition seen in Figure A EXCEPT:

     

     

     

     

    1. Internal rotation of the femoral prosthesis

    2. Internal rotation of the tibial prosthesis

    3. Lateralization of the femoral prosthesis

    4. Medialization of the tibial prosthesis

    5. Lateralization of the patellar prosthesis Corrent answer: 3

    Lateralization of the femoral prosthesis would not lead to lateral patellar subluxation. However, lateralization of the patella or medialization of the tibial tray could lead to lateral patellar subluxation by increasing the Q angle.

     

    The Malo article reviewed the common issues regarding the patellofemoral compartment after TKA. Patellar instability & maltracking can result from malpositioning of the component, prosthetic design, improper patellar preparation, or soft-tissue imbalance. Patellar instability & maltracking most often occurs due to internal malrotation of the femoral or tibial components, lateralization of the patellar component, or medialization of the femoral or tibial components. Medialization, not lateralization of the femoral prosthesis will increase the Q angle and lead to lateral subluxation of the patella.

     

     

    OrthoCash 2020

     

  13. Polyethylene gamma irradiation in nitrogen gas results in which of the following?

    1. Annealing

    2. Gas plasma

    3. Uncross-linking

    4. Free radicals

    5. Remelting

     

    Corrent answer: 4

     

    As described in the review by Gordan et al. sterilization of polyethylene with irradiation causes free radical production which leads to cross-linking and increased wear resistance. Irradiation in inert gas reduces excessive oxidation. Remelting and annealing are heat treatments that reduce the excess free radicals formed by irradiation.

     

    McKellop et al. showed sterilizing an ultra-high molecular weight polyethylene acetabular cup without radiation (ethylene oxide or gas plasma) avoids immediate and long-term oxidative degradation of the implant but does not improve the inherent wear resistance of the polyethylene.

     

    Incorrect Answers:

    1. Annealing is the process of heating the polyethylene close to its melting point to remove free radicals.

    2. Gas plasma does not result from gamma irradiation.

    3. Gamma irradiation does not result in reversal of cross-linking.

    5) Remelting involves going past the melting point to remove free radicals. These sterilization processing techniques significantly affect polyethylene performance and wear characteristics.

     

     

     

    OrthoCash 2020

     

  14. A 60-year-old male had a total hip replacement 8 years ago. There is evidence of eccentric polyethylene wear and some retroacetbular osteolysis. You discuss treatment options of acetabular revision if the component is found to be loose intra-operatively versus isolated polyethylene exchange if the acetabular component is stable intra-operatively with the patient. What is the most common complication of isolated polyethylene exchange with bone grafting that should be disclosed?

    1. Sciatic nerve injury

    2. Intraoperative acetabular fracture

    3. Postoperative hip instability

    4. Infection

    5. Catastrophic implant failure Corrent answer: 3

    The most common complication of isolated polyethylene exchange with bone grafting that should be disclosed is postoperative hip instability.

     

    Boucher et al looked at 24 patients who had an isolated polyethylene liner exchange for wear or osteolysis and found six patients (25%) dislocated and another 4 patients complained of instability.

     

    Restrepo et al compared the results of isolated polyethylene exchange versus acetabular revision and found a 10% failure rate when retaining the acetabular component. Their cohort had no dislocations, which they attribute to their direct lateral approach.

     

    Therefore, although both implant failure and instability are known complications of isolated polyethylene exchange, it is currently believed that hip instability and dislocation are the most common.

     

     

     

    OrthoCash 2020

     

  15. Which of the following benefits can be expected from unicompartmental knee arthroplasty compared to total knee arthroplasty for medial compartment knee arthritis?

    1. Better clinical outcomes at one year follow-up.

    2. Greater survivorship rate at 10 year follow-up

    3. Faster postoperative rehabilitation

    4. Better postoperative knee alignment

    5. Reduced risk of secondary surgery within the first year Corrent answer: 3

    Unicompartmental knee arthroplasty has been shown to result in faster rehabilitation compared to total knee arthroplasty. Theoretically, the knee kinematics are closer to anatomic as the ACL is preserved. This has not resulted in increased longevity, less pain, or better clinical outcomes on a consistent basis. Contraindications to unicompartmental knee arthroplasty include ACL deficiency, flexion contracture, coronal malalignment, and

    inflammatory arthritis.

     

    Heck et al performed a multi-center review of unicompartmental knee arthroplasties. Survivorship was 91% at 10 years. Obesity was associated with a higher rate of revision.

     

    Fisher et al reviewed 91 patients who underwent either TKA or UKA. Patients who underwent a UKA experienced a faster return to function and quicker discontinuation of postoperative narcotic medications. Blood loss and length of hospital stay were greater in the TKA group. Rate of complications was similar in both groups.

     

     

     

    OrthoCash 2020

     

  16. An active 40-year-old male undergoes hip surgery for arthritis. A post-operative radiograph is provided in Figure A. Each of the following are complications associated with this procedure EXCEPT:

     

     

     

     

     

    1. Deep vein thrombosis

    2. Femoral neck fracture

    3. Aseptic loosening

    4. Polyethylene debris

    5. Dislocation

    Corrent answer: 4

     

    Modern hip resurfacing implants as shown in Figure A utilize metal-on-metal bearing components, thus polyethylene debris (Answer 4) and its potential subsequent osteolysis is not an option. Hip resurfacing has the potential advantages of preserving proximal femoral bone and providing physiologic stress transfer. Potential disadvantages include the inability to adjust offset and limb-length as readily compared to a conventional total hip arthroplasty. Aseptic loosening of the components can occur and would be an indication for conversion to total hip arthroplasty. Fracture of the femoral neck is the most common complication (0-4%) of hip resurfacing. Resurfacing has a lower incidence of dislocation compared to conventional hip arthroplasty but can still occur. The article by Anglin et al states that implanting the femoral component in 10 degrees of relative valgus and high bone mineral density was correlated with increased load to fracture. Notching of the superior femoral neck is associated with increased fracture risk.

     

     

     

    OrthoCash 2020

     

  17. A previously healthy 68-year-old woman falls and sustains the fracture seen in Figure A. Her index procedure was approximately 10 years ago. The patient is taken to surgery, and the femoral stem is found to be loose. The acetabular component is found to be well fixed in good position. In addition to using a new poly liner, what other procedure(s) is now indicated in this patient?

     

     

     

     

     

    1. Cemented femoral revision

    2. Retention of current hardware and fixation using cerclage wires

    3. Open reduction and internal fixation with a locking plate

    4. Both uncemented femoral revision and revision of the acetabular shell

    5. Uncemented femoral revision bypassing the distal deficiency by two cortices.

    Corrent answer: 5

     

    This is a Vancouver B2 periprosthetic femur fracture as the fracture involves the tip of the stem and the stem is loose. In fracture patterns stabilization with an uncemented extensively porous coated femoral stem is recommended.

    However, in order to increase the femoral strength similar to the contralateral side, the surgeon must bypass the most distal cortical deficiency by two cortices.

     

    O'Shea et al retrospectively found satisfactory results in patients with Vancouver B2 and B3 periprosthetic femur fractures treated with uncemented extensively porous coated femoral stems with good distal fixation.

     

    Sledge et al described their technique in order to reconstruct the proximal femur as well as use an uncemented femoral stem to achieve good results.

     

    Larson et al found an increase in femoral strength up to 84% of the contralateral side was achieved by bypassing the most distal deficiency by two cortical diameters.

     

    Illustration A summarizes the Vancouver classification of periprosthetic femur fractures.

     

     

     

     

     

     

    OrthoCash 2020

  18. While performing a revision total knee arthroplasty, the surgeon decides to upsize the femoral component with use of posterior femoral augments. Which of the following intraoperative exam findings would have led to this decision?

    1. A knee that is balanced in extension and tight in flexion.

    2. A knee that is balanced in extension and loose in flexion.

    3. A knee that is tight in extension and tight in flexion.

    4. A knee that is loose in extension and loose in flexion.

    5. A knee that is loose in extension and balanced in flexion.

     

    Corrent answer: 2

     

    Increasing the size of the femoral component posteriorly will balance a flexion-extension mismatch where the knee is loose in flexion and stable/balanced in extension. The referenced article by Peters discusses multiple different soft tissue balancing techniques used for trialing scenarios as well as for varus or valgus instability, and knees with flexion contractures. A knee tight or loose on both flexion and extension can be corrected by decreasing or increasing the size of the polyethylene, respectively. A knee that is tight in flexion/balanced in extension can be corrected by decreasing the AP diameter of the femoral component.

     

     

     

    OrthoCash 2020

     

  19. A 67-year-old woman undergoes a total hip arthroplasty. Postoperatively, she develops a complete peroneal palsy that does not improve with flexion of the knee and removal of compressive dressings. All of the following are associated with a post-operative nerve palsy EXCEPT:

    1. Developmental dysplasia of the hip

    2. Lengthening of the extremity

    3. Surgeon self-rating the procedure as being a difficult intervention

    4. Avascular necrosis of the femoral head

    5. Posttraumatic arthritis

       

      Corrent answer: 4

       

      This patient has a motor and sensory (complete) peroneal nerve palsy. The incidence of postoperative nerve injury ranges from 0% to 3% and most often affects the peroneal branch of the sciatic nerve. Eggli et al reviewed 370 patients following total hip arthroplasty and found the only factor that correlated to nerve palsy was the surgeon self-rating the procedure as difficult potentially leading to direct or indirect mechanical trauma.

      Farrell et al reviewed 27,004 total hip arthroplasties and found that developmental dysplasia, extremity lengthening, posterior approach, posttraumatic arthritis, and uncemented femoral component increased the odds ratio of a nerve palsy. They found less than a 50% rate of nerve recovery.

       

       

       

      OrthoCash 2020

       

  20. A 65-year-old female undergoes total hip arthroplasty. Instability of the prosthesis with straight hip flexion is identified during intraoperative range of motion testing. A fluoroscopic image is shown in figure A. Impingement is caused by the structure labeled with the asterisk. Which muscle originates upon this structure?

     

     

     

     

     

    1. Sartorius

    2. Direct head of the rectus femoris

    3. Reflected head of the rectus femoris

    4. Obturator externus

    5. Obturator internus

     

    Corrent answer: 2

     

    The patient presents with hip instability due to anterior inferior iliac spine (AIIS) impingement which resolves after AIIS resection. The direct head of the rectus femoris originates upon the AIIS.

     

    The rectus femoris, according to Henry Gray (Gray's Anatomy), "is situated in the middle of the anterior region of the thigh; it is fusiform in shape, and its superficial fibres are arranged in a bipenniform manner, the deep fibres running straight down to the deep aponeurosis. It arises by two tendons: one,

    the anterior or straight (direct), from the anterior inferior spinous process of the ilium; the other, the posterior or reflected tendon, from a groove above the brim of the acetabulum."

     

    Pan et. al. report a case of femoroacetabular impingement caused by abutment of the femoral head-neck junction against a hypertrophic AIIS. Resection of the AIIS restored full painless hip range of motion.

     

    Davidovitch et. al. reported two cases of intra-operative hip instability caused by prosthetic or trochanteric impingement against a hypertrophic AIIS. Exposure and resection of the AIIS restored hip stability in both cases.

     

    Figure A is an intra-operative false profile fluoroscopy image which demonstrates impingement of the greater trochanter against the AIIS. The AIIS is labeled with a white asterisk.

     

    Incorrect Answers:

    1. The origin and insertion of the sartorius are the anterior superior iliac spine and the anteromedial tibia, respectively.

    1. The reflected head of the rectus femoris, as stated above, originates from a groove above the brim of the acetabulum.

    2. The origin and insertion of the obturator externus are the bony obturator foramen and trochanteric fossa, respectively.

    3. The origin and insertion of the obturator internus are the ischiopubic ramus/obturator membrane and the greater trochanter, respectively.

     

     

     

    OrthoCash 2020

     

  21. Figure A represents a free body diagram of the hip of a patient standing on the right leg. The forces and distances are labeled on the diagram and the resulting hip joint force (J) = 1800N. What is the resultant value for J when the acetabular component is medialized given the new distances shown in Figure B?

     

     

     

     

     

     

    1. 1000N

    2. 1200N

    3. 1800N

    4. 2200N

    5. 3600N

     

    Corrent answer: 2

     

    The new joint force (J) calculated is 1200N as shown in Illustration A. The joint force decreases as the acetabular component is moved medial. This is a result of decreasing abductor tension as the acetabular component is medialized (Distance B is 50mm instead of the initial 100mm). Mechanical equilibrium is when the sums of all forces and moments equal zero [(AT x A) - (5/6BW)x B))

    = 0]. Free body diagrams show the locations and directions of all forces and moments acting on a body. The body and the left leg weigh 5/6 x total body weight (BW) in a right-sided single leg stance as given in this example.

    The article by Johnston et al created a mathematical hip model that found that the loads on the hip were lowered by placing the center of the acetabulum as far medially, inferiorly, and anteriorly as possible.

     

     

     

     

     

     

     

    OrthoCash 2020

     

  22. Osteoprotegrin (OPG) binds to what structure to inhibit particle-induced osteolysis?

    1. RANK

    2. RANK-ligand

    3. OPG-receptor

    4. CD7

    5. CD10

     

    Corrent answer: 2

     

    Osteoprotegrin (OPG) binds to RANK ligand (RANKL) to inhibit it from binding to RANK which are present on osteoclast-precursor cells. Normally, RANKL interacts with RANK to stimulate activation of osteoclasts.

     

    Jacobs et al present a nice review of the cellular and chemical mediators of osteolysis.

     

    Illustration A demonstrates the interaction between OPG, RANK, and RANKL during osteoclast differentiation.

     

     

     

     

     

    OrthoCash 2020

     

  23. According to the latest recommendations made by the AAOS in their clinical guidelines for the treatment of osteoarthritis (OA) of the knee, which of the following nonoperative treatment modalities has the weakest supporting evidence for the treatment for knee osteoarthritis?

    1. Weight loss

    2. Activity modifications

    3. Quadriceps strengthening

    4. Intra-articular hyaluronic acid injections

    5. Intra-articular corticosteroid injections Corrent answer: 4

    Intra-articular hyaluronic acid is no longer recommended as an effective method of treatment for patients with symptomatic knee arthritis based on the revised AAOS clinical guidelines from 2013. The previous review from

    2009 guidelines was reported as inconclusive.

     

    Nonoperative treatment modalities whose use is supported by the literature include: activity modifications, weight loss, quadriceps strengthening, patellar taping, NSAIDs, tylenol (Now 3000mg/24hr is recommended from 4000mg/24hr), and intra-articular steroids. Treatment options that are NOT supported by the literature (or are considered inconclusive) include: intra-articular hyaluronic acid injections, lateral heel wedges for medial knee OA, glucosamine and chondroitin, needle lavage, and arthroscopy in patients with primary OA.

     

    Illustration A shows a chart showing the rates of accuracy of intra-articular knee injections.

    Incorrect Answers:

    Answer 1: Weight loss is considered a moderate recommendation according to the AAOS clinical guidelines.

    Answer 2: Activity modification is strongly recommended according to the AAOS clinical guidelines.

    Answer 3: Quadriceps strengthening is strongly recommended according to the AAOS clinical guidelines.

    Answer 5: The use of intra-articular corticosteroid injections cannot be recommended for or against according to the latest AAOS clinical guidelines.

     

     

     

     

     

     

     

    OrthoCash 2020

     

  24. All of the following are intraoperative techniques to treat a flexion contracture in total knee arthroplasty EXCEPT:

    1. Resect osteophytes

    2. Release posterior capsule

    3. Resect more distal femur

    4. Downsize the femoral component

    5. Tenotomize the hamstrings Corrent answer: 4

      Of the options listed, downsizing the femoral component would NOT be an intraoperative technique to treat a flexion contracture in total knee arthroplasty.

       

      Flexion contractures (an extension gap issue) can be addressed by resecting osteophytes, resecting the posterior capsule/gastrocnemius, resecting additional distal femur, and tenotomizing the hamstrings. Downsizing the femoral component would only change the flexion gap and have no impact on a flexion contracture (an extension gap issue.) Downsizing the femoral component means decreasing the size of the implant in the anterior-posterior dimension only. There is no change in the proximal-distal dimension, and thus changing the size of the femoral implant only affects the flexion gap, and not the extension gap or a flexion contracture.

       

      Bellemans et al. propose an algorithm to treat preoperative flexion contractures greater than 5 degrees. The steps include:

      Step 1: Mediolateral ligament balancing with meticulous osteophyte removal and over resection of distal femur by 2 mm.

      Step 2: Add posterior capsular release.

      Step 3: Add distal femoral resection up to 4 mm. Step 4: Add hamstring tenotomy.

      For flexion contractures less than 15 degrees they report steps 1 and 2 were sufficient for 100% of the cases in their study. Steps 3 and 4 were only necessary for some contractures greater than 35 degrees.

       

      Illustration A shows how changing the femoral component size only affects the flexion gap.

       

      Incorrect Answers

      Answers 1,2, 3, and 5 are all methods to treat a flexion contractures in total knee arthroplasty.

       

       

       

       

       

       

      OrthoCash 2020

       

  25. A 64-year-old healthy female patient underwent right total hip replacement (THR) through a posterior approach 6 months ago. She has now dislocated posteriorly 3 times, each followed by closed reduction under anesthesia in the operating room. A radiograph is provided in Figure A. Treatment should include:

     

     

     

    1. Hip spica casting

    2. Revision of the femoral component to a modular stem with retention of the acetabular component

    3. Revision of the acetabular component

    4. Hip abduction bracing

    5. Revision to a constrained liner with retention of the acetabular and femoral prostheses

    Corrent answer: 3

     

    The optimal acetabular component alignment is 30-50 degrees of abduction and 15-20 degrees of anteversion. This patient has a vertical acetabular component that requires revision.

     

    Reasons for recurrent THA instability include infection, non-compliance with precautions, component malposition, impingement and inadequate soft tissue tension. Strategies that do not address the underlying cause of recurrent dislocation will not definitively treat the instability. Non-operative measures cannot account for drastic component malposition.

     

    Figure A shows an AP pelvis after right total hip arthroplasty. The acetabular component is vertically positioned. The femoral component is in slight varus but not enough to warrant revision on its own. The version of each component cannot be definitively determined on this single film. The acetabular component has a significant amount of version that could be ante- or retroversion.

     

    Incorrect Answers:

    1. Hip spica casting will not correct the position of the components and will not be a viable long term solution for the patient. It may have a role in a setting of

      non-compliance with hip precautions with components well placed.

    2. Although the femoral component anteversion cannot be determined based on this view, there is no clear indication for revision to a modular stem in this case.

    1. Abduction bracing similarly will not correct component malposition.

    2. Simply changing the poly liner for a constrained liner does not fix the malalignment. It may provide more stability, but at the expense of much greater stress at the component-bone interface.

     

     

     

    OrthoCash 2020

     

  26. Which of the following situations is appropriate for revision of a total hip arthroplasty to a constrained acetabular liner?

    1. Periprosthetic acetabular fracture with resulting pelvic discontinuity

    2. Chronically infected total hip arthroplasty

    3. Recurrent dislocations in a patient whose femoral component is positioned in 15° retroversion

    4. Recurrent dislocations in a patient whose cup is positioned in 10° retroversion and 60° abduction

    5. Recurrent dislocations in a patient whose cup is positioned in 20° anteversion and 40° abduction

    Corrent answer: 5

     

    Use of a constrained acetabular liner is indicated in the setting of recurrent instability with well positioned components.

     

    Constrained acetabular liners provide excellent stability but may limit motion and fail prematurely if placed in the wrong patient. They could be considered in the following situations: 1) cases with no identifiable cause for instability, 2) abductor deficiency, and 3) patients with neuromuscular disorders. While multiple studies have shown good short-term and long-term results, they should be considered an option of last resort due to the risks of fatigue failure and component dissociation.

     

    Soong et al state that a constrained acetabular liner should be considered in patients who have recurrent instability without obvious cause. The components should be appropriately positioned (choice 5), as this may be the cause of instability (choices 3 and 4).

     

    Callaghan et al showed good results of contrained liners (<10% failure) in patients with recurrent instability. Use of a constrained liner was not associated with an increased osteolysis or aseptic loosening rate.

    Illustration A shows an example of a contrained acetabular liner.

     

    Incorrect Answers:

    Answer 1: A periprosthetic acetabular fracture with a pelvic discontinuity should be treated with revision of the acetabular component with or without internal fixation of the discontinuity.

    Answer 2: A chronically infected total hip arthroplasty is best treated with a two-stage exchange.

    Answer 3: In this situation, the femoral component is malpositioned and should be revised.

    Answer 4: In this situation, the acetabular component is malpositioned and should be revised.

     

     

     

     

     

     

    OrthoCash 2020

     

  27. A 66-year-old female presents with knee instability going down stairs 18 months after a posterior cruciate retaining total knee arthroplasty. She reports having recurrent effusions. Radiographs are shown in Figure A. What is the most likely cause for her instability?

     

     

     

    1. Intraoperative rupture of the patellar tendon

    2. Alteration of the joint line

    3. Posterior cruciate insuffiency

    4. Anterior cruciate insufficiency

    5. Catastrophic component loosening Corrent answer: 3

    Patients with posterior instability following TKA typically present with a sense of instability, recurrent knee effusions, anterior knee pain, and posterior instability (with a positive posterior drawer or a posterior sag sign). As evident on the radiograph, the tibia is subluxated posteriorly relative to femur. The other choices would present differently on radiographs.

     

    Pagnano et al report on 25 painful primary posterior cruciate ligament retaining total knee arthroplasties that were revised for flexion instability. They suggest that flexion instability can be a cause of persistent pain and functional impairment after posterior cruciate ligament retaining total knee arthroplasty. In the setting of flexion instability, revision to a posterior stabilized knee design along with meticulous ligament balancing seems to give reliable results.

     

    Waslewski et al performed a Level 4 study of 13 patients with posterior instability following PCL retaining TKA. They concluded that revision to a

    posterior stabilized component leads to improved outcomes in patients with flexion instability.

     

    Incorrect Answers:

    1: Patellar tendon rupture would result in loss of active knee extension

    2: Elevation of the joint line occurs with excessive distal femoral resection and consequent upsizing of the polyethylene insert

    4: The anterior cruciate ligament is sacrificed in almost all of the total knee arthroplasty systems today. More so, the lateral knee radiograph shows posterior subluxation of the tibia on the femur. This is more likely to be caused by PCL insufficiency.

    5: There is no evidence of component loosening or fracture

     

     

     

    OrthoCash 2020

     

  28. During a minimally invasive total knee arthroplasty with a quadriceps-sparing approach, the exposure is found to be limited and causing difficulties with jig alignment. What is the optimal next step?

    1. Conversion of the exposure to a subvastus approach

    2. Tibial tubercle osteotomy

    3. Conversion of the exposure to a two-incision approach

    4. Conversion of the exposure to a standard parapatellar arthrotomy

    5. Ligament release to improve exposure Corrent answer: 4

    As with difficulties in any minimally invasive exposure, conversion to a larger or more traditional exposure is indicated when the exposure is causing difficulties or potential complications. The referenced study by Tria mentions this, but the paper focuses on the importance of having appropriate instrumentation to be able to successfully perform minimally invasive procedures.

     

     

     

    OrthoCash 2020

     

  29. A 66-year-old woman with genu valgum osteoarthritis undergoes total knee replacement. What technical error could lead to postoperative lateral patellar instability?

    1. External rotation of the tibial component

    2. External rotation of the femoral component

    3. Internal rotation of the femoral component

    4. Lateralization of the femoral component

    5. Medialization of the patellar component Corrent answer: 3

    Internal rotation of the femoral component can lead to post-operative lateral patellar instability.

     

    The bony deficiency of a valgus arthritic knee is at the lateral femoral condyle. A total knee system that employs posterior condylar referencing can cause internal rotation of the femoral component if the lateral femoral condyle is deficient.

     

    Briard and Hungerford state that malalignment of any component can lead to patellar instability. Internal rotation of the femoral component increases the Q-angle, which increases the lateral pull of the knee extensors. External rotation of the femoral and tibial components both decrease the Q-angle. Lateralization of the femoral component and medialization of the patellar component also help stabilize the patella by decreasing the Q-angle.

     

    Lee et al showed good patellofemoral stability in a series of patients where attention was paid to avoidance of femoral and tibial component malrotation and prevention of overstuffing of the patellofemoral joint.

     

     

     

    OrthoCash 2020

     

  30. All of the following are acceptable indications for use of a constrained acetabular component EXCEPT:

    1. Recurrent dislocations due to abductor insufficiency

    2. Recurrent dislocations due to unsalvageable capsular attenuation

    3. Recurrent dislocations due to severe polyethylene wear

    4. Recurrent late dislocations without component loosening or malposition

    5. Recurrent dislocations due to cognitive or neuromuscular disease Corrent answer: 3

      Instability due to polyethylene wear alone is not a reason to use a constrained component, though revision of the acetabular component may be necessary.

       

      Indications for the use of a constrained acetabular component include: 1) recurrent dislocations due to unrepairable soft-tissue insufficiency from lack of abductor function or capsular attenuation; 2) severe cognitive disorders; 3) late dislocations with well positioned components.

      Shrader et al reviewed the Mayo registry and found that with the use of a constrained acetabular component instability was eliminated or prevented in 107 of 109 patients.

       

       

       

      OrthoCash 2020

       

  31. A 72-year-old female returns to clinic for 15 year follow up of left total hip arthroplasty. She ambulates without any assistive devices, has no pain, and denies any recent fevers or systemic illness. A radiograph is provided in figure A. Which of the following is the best treatment option?

     

     

     

     

     

    1. Follow up radiographs in 3 years

    2. Follow up radiographs in 5 years

    3. Revision surgery with femoral head and polyethylene exchange and retroacetabular bone grafting

    4. Revision of acetabular component with jumbo cup and femoral head exchange

    5. Revision of acetabular component with jumbo cup and femoral stem revision

    Corrent answer: 3

    The radiograph demonstrates osteolytic lesions behind the acetabular cup and eccentric wear of the polyethylene with superior migration of the femoral head within the cup. Options 1 and 2 are poor choices because the osteolysis must be addressed to prevent further bone loss and eventual loosening of the implant. The acetabular cup remains in acceptable position and does not need to be revised unless found to be grossly loose intraoperatively. The osteolytic lesions need to be addressed with bone grafting to prevent cavitary bone loss. The femoral stem shows no indication of loosening or osteolysis around the stem and can be retained.

     

    Maloney et al reported a case series of 35 patients who underwent femoral head and polyethylene exchange with retroacetabular bone grafting of osteolytic lesions. All 35 cups remained stable and had not been revised at 2 years.

     

    Maloney et al discusses the treatment of osteolytic defects following hip and knee arthroplasty at the 2007 Implant Wear Symposium.

     

     

     

    OrthoCash 2020

     

  32. All of the following are independent risk factors for dislocation after total hip arthroplasty EXCEPT?

    1. Female gender

    2. Osteonecrosis

    3. Inflammatory arthritis

    4. Post traumatic osteoarthritis

    5. Age >70

     

    Corrent answer: 4

     

    Dislocation continues to be a problem for patients undergoing total hip arthroplasty. The incidence of dislocation is highest in the first year after arthroplasty and then continues at a relatively constant rate for the life of the arthroplasty.

     

    Berry et al in a Level III retrospective study showed that patients at highest risk are female patients (relative risk 2.1), those with a diagnosis of osteonecrosis of the femoral head (relative risk 1.9), an acute fracture or nonunion proximal femur treated with THA (relative risk 1.8), history of inflammatory arthritis (relative risk 1.5), and age greater than 70 (relative risk 1.3). The relative risk of dislocation for patients with posttraumatic arthritis of the hip was not significantly different from that for patients with osteoarthritis (relative risk, 1.3; 95% confidence interval, 0.6 to 2.8) (p = 0.59).

     

     

    OrthoCash 2020

     

  33. In his first day home after undergoing a total hip arthroplasty a 65-year-old male falls down the stairs and sustains the fracture seen in Figure A and B. Intra-operative examination reveals the stem to be loose. What is the preferred treatment for this injury?

     

     

     

     

     

     

     

     

    1. Open reduction internal fixation with cables and proximal femoral locking plate

    2. Open reduction internal fixation with allograft strut and multiple cables

    3. Revision femoral component with proximal femoral replacement

    4. Revision femoral component with long stem diaphyseal press-fit stem

    5. Revision femoral component with cemented stem Corrent answer: 4

    Figure A and B show a Vancouver B2 peri-prosthetic fracture with loose femoral stem. In Vancouver B2 periprosthetic femur fractures it is recommended that the fracture be stabilized with the use of an uncemented extensively porous coated femoral component. However, in order to increase the femoral strength similar to the contralateral side, the surgeon must bypass the most distal cortical deficiency by two cortices.

     

    Greidanus et al is an Instructional Course Lecture highlighting the various methods of treatment for peri-prosthetic fractures depending on bone stock, fracture classification, and available implants.

     

    Sledge et al described their technique in order to reconstruct the proximal femur as well as use a uncemented femoral stem to achieve good results.

     

     

     

    OrthoCash 2020

     

  34. A 45-year-old male, with a history of avascular necrosis, undergoes a total hip arthroplasty using a metal-on-metal bearing. Which of the following statements regarding the use of metal-on-metal bearings is true?

    1. Produce lower serum metal ion concentrations than metal-on-cross linked polyethylene bearings

    2. Have a decreased volumetric wear rate compared to metal-on-ceramic bearings

    3. Have smaller wear particles compared to metal-on-cross linked polyethylene bearings

    4. Have a higher incidence of early soft-tissue sarcoma compared to all other bearing surfaces.

    5. The wear rate significantly increases after the first year of use.

     

    Corrent answer: 3

     

    Metal-on-metal bearings used in total hip arthroplasty form smaller wear particles compared to metal-on-cross linked polyethylene bearings. Metal-on-metal bearings produce higher serum metal ion concentrations, have lower volumetric wear rates, and achieve a "steady state" of wear after the first year of use whereby the wear rate then decreases. The volumetric wear rate of metal-on-metal articulations is greater than metal-on-ceramic as reported by

    in vitro studies. Also, there is not evidence to suggest an increased risk of soft tissue sarcoma with use of the metal-on-metal bearings.

     

    Heisel et al, in an instructional lecture series, systematically discusses all types of bearing options with their advantages and disadvantages.

     

    Amstutz et al attempts to answer questions and concerns over the use of metal-on-metal bearings for total hip arthroplasty.

     

    Haddad et al review current literature on soft-tissue reactions resulting in implant failure and associated complications in the use of metal on metal hips. Implant position and immunologic reactions are thought to be key players in failure.

     

     

     

    OrthoCash 2020

     

  35. A 64-year-old female underwent a total knee arthroplasty 4 years ago and has increasing pain for the past 6 months. Knee aspiration reveals 4,000 leukocytes with 80% polymorphonucleocytes and a 2-stage revision arthroplasty is planned. When comparing articulating cement spacers to static spacers following resection, all of the following are potential advantages of articulating spacers EXCEPT

    1. Decreased quadriceps shortening

    2. Decreased rate of infection recurrence

    3. Increased knee range of motion for duration of cement spacer implantation

    4. Better maintenance of joint space

    5. Decreased exposure time during reimplantation Corrent answer: 2

    Articulating cement spacers and static spacers have been found to have equivalent reinfection rates as well as equivalent functional outcomes (including ultimate knee range of motion) after reimplantation. The Level 4 study by Fehring et al states that static spacers prompt quadriceps shortening and can increase the difficulty and exposure time during reimplantation compared to articulating spacers. The review article by Jacobs et al also cite better maintenance of joint space and increased knee range of motion for the duration of the cement spacer placement with articulating spacers.

     

    Illustration A shows an example of a static spacer with a cement block and a rod temporarily locking the knee in extension. Illustration B shows an example of an articulating spacer.

     

     

     

     

     

     

     

     

    OrthoCash 2020

     

  36. A 65-year-old male undergoes a primary total knee arthroplasty. His preoperative radiographs are seen in figures A and B. Postoperative examination reveals an inability to dorsiflex his ankle or extend his toes. Sensation is decreased along the dorsum of his foot as well as between the 1st and 2nd toes. All of the following are risk factors for this complication following total knee arthroplasty EXCEPT?

     

     

     

    1. Aberrant retractor placement

    2. Postoperative epidural analgesia

    3. Correction of a 20 degree preoperative valgus deformity

    4. Excessive medial release

    5. Preoperative diagnosis of neuropathy Corrent answer: 4

    Peroneal nerve palsy following a total knee arthroplasty has been shown to be associated with postoperative epidural analgesia, correction of large valgus deformities, and preoperative diagnosis of neuropathy in the involved extremity (either centrally or peripherally). One may also sustain peroneal nerve palsies following aberrant retractor placement intraoperatively.

     

    Idusuyi et al performed a retrospective review and found a significant increase in the relative risk of developing a peroneal nerve palsy following total knee arthroplasty in patients who received postoperative epidural analgesia, had prior lumbar laminectomies, and had preoperative valgus deformities of greater than 12 degrees.

     

    The review by Nercessian et al discusses the above risk factors, treatments and prognosis of a peroneal nerve palsy following total knee arthroplasty. Most studies in this review demonstrated approximately 50% complete recovery with a higher likelihood of complete recovery associated with a less severe

    initial injury. The time to recovery ranged from 18 months to 5 years, and while nerve exploration was seen to help in a small group of patients, this treatment modality remains controversial.

     

     

     

    OrthoCash 2020

     

  37. A 67-year-old female underwent a total hip arthroplasty 6 months ago and has had recurrent prosthetic dislocations. Figure A is a representative drawing of the mechanism of her dislocation. During the time of surgery what is the most likely factor leading to the bone-on-bone impingement?

     

     

     

     

     

    1. Lateralizing the acetabular cup

    2. Decreased femoral offset

    3. Increased femoral offset

    4. Increased acetabular inclination (>55 degrees)

    5. Small head-neck ratio (<2)

     

    Corrent answer: 2

     

    Figure A displays bone-on-bone impingement where the trochanter strikes the pelvis. The article by Malik et al states that decreased hip offset places the hip at risk for the femoral bone impinging against the pelvis at the extremes of motion.

     

    There are several ways the femoral offset can be decreased leading to bone-bone impingment. One such way is medializing and raising the center of rotation, which is often done to prevent metal neck-on-cup impingement (Illustration A).

    Methods to correct a decreased offset includes a higher osseous femoral neck cut, a longer modular head, a high-offset stem, or a combination of these.

     

    The Level 4 study by Kristiansen reported a 4.9% total hip dislocation rate and emphasized the importance of maintaining the effective femoral neck length to increase prosthesis stability.

     

     

     

     

     

     

    OrthoCash 2020

     

  38. A 68-year-old male 2 weeks status post left total hip arthroplasty experiences a painful clunk getting out of bed in the morning. He is unable to bear any weight on the left leg. A radiograph is provided in figure A. Following closed reduction under sedation, the hip continues to dislocate with flexion up to 90 degrees. Each of the following operative interventions will increase the stability of the hip EXCEPT:

     

     

     

    1. Revising the acetabular component to a more medialized position

    2. Advancing the trochanter distal on the femur

    3. Converting to a femoral component with extended offset

    4. Replacing the acetabular polyethylene with a constrained liner

    5. Replacing the femoral head with a larger size Corrent answer: 1

    The radiograph demonstrates a dislocated total hip arthroplasty that remains unstable following reduction. Medializing the acetabulum decreases the lever arm of the abductors resulting in reduced soft tissue tensioning, greater laxity, and thus decreased stability.

     

    Conversely, stability can be increased in the following ways: Revising to an extended offset femoral component and advancing the trochanter increases the lever arm of the abductors increasing soft tissue tension. Increasing the head to neck ratio (choice 5) increases the range of motion to impingement. Constrained liners increase the functional depth of the cup. This increases the jumping distance from impingement to excursion at the cost of greater contact stresses at the acetabular bone, cup, and liner interfaces.

     

    Soong et al discusses the evaluation and management of dislocation following total hip arthroplasty in this review article. The basis of treatment is multifactorial including chronicity since implantation, mechanism of dislocation, position of the implants, and the age, health, and expectations of the patient. Interventions include bracing, soft tissue management, and component revision depending upon the etiology of the instability.

     

    Parvizi et al reviews surgical techniques of revision hip arthroplasty in this instructional course lecture.

    Illustration A provides a visual example of the importance the head to neck ratio plays in stability.

     

     

     

     

     

     

    OrthoCash 2020

     

  39. A 67-year-old man 6 years status post right total hip arthroplasty falls while walking his dog. He complains of pain and is unable to bear weight through the right leg. He denies any hip or thigh pain prior to this fall. A radiograph is provided in figure A. Which of the following is the most appropriate management?

     

     

     

    1. Traction for 3 weeks followed by 2 months of non-weight bearing mobilization

    2. Open reduction and plate fixation with cable augmentation proximally

    3. Revision arthroplasty with a cementless long stem bypassing the fracture site by two cortical diameters

    4. Revision arthroplasty with cemented femoral stem bypassing the fracture site by two cortical diameters

    5. Revision arthroplasty with cementless long stem bypassing the fracture site by two cortical diameters and allograft strut augmentation

    Corrent answer: 2

     

    The radiograph demonstrates an oblique fracture of the femur at the tip of the stem. The stem is stable within the proximal segment, a Vancouver B1 fracture. Illustration A depicts the Vancouver Classification of femoral periprosthetic hip fractures. B1 fractures are most appropriately treated with plate fixation. Choices 3 and 5 are appropriate treatments for B2 and B3 fractures, respectively.

     

    Ricci et al reviewed 50 Vancouver B1 patients treated with indirect reduction and fixation utilizing a lateral plate without allograft strut augmentation. All patients went on to union. One was complicated by infection.

    In an instructional course lecture, Ricci reviews the evaluation, classification, and management of femoral periprosthetic hip fractures.

     

     

     

     

     

     

    OrthoCash 2020

     

  40. Which of the following statements regarding the use of continuous passive motion (CPM) devices following total knee arthroplasty is true?

    1. The use of CPM decreases the incidence of knee flexion contracture at 6 months following surgery.

    2. The use of CPM has been associated with a decreased incidence of secondary surgery for knee manipulation.

    3. The use of CPM has not demonstrated any difference in clinical outcomes at one year following surgery.

    4. The use of CPM has been associated with increasing analgesic pain requirements in the first 3 days following surgery.

    5. The use of CPM decreases knee flexion at one year following surgery.

     

    Corrent answer: 3

     

    Continuous passive motion (CPM) devices have not demonstrated superior clinical outcomes in multiple level 1 studies.

     

    MacDonald et al conducted a level 1 prospective study of 120 TKA's randomly allocating the patients to no CPM, controlled CPM, and CPM as tolerated. He found no difference in knee ROM, Knee Society score, or length of hospital stay amongst patients who used a continuous passive motion device for 24 hours following TKA.

    Likewise, Leach et al randomized 85 TKA's to CPM or no CPM following TKA in a prospective level 1 study. He found no differences in knee ROM, pain scores, or analgesic use in patients who used CPM following TKA.

     

    Bourne conducted a meta-analysis and did find some evidence suggesting CPM can shorten the length of hospital stay and improve knee flexion at early time points, but did not find any evidence of improved clinical outcome scores.

     

     

     

    OrthoCash 2020

     

  41. Which of the following manufacturing techniques of UHMWPE results in the lowest susceptibility to fatigue crack formation and propagation in joint arthroplasty bearings?

    1. Ram bar extrusion with secondary machining into the desired product

    2. Hot isostatic pressing into bars with secondary machining

    3. Irradiation with 10 Mrad of radiation achieiving a polyethylene crystallinity of >99%

    4. Direct compression molding from PE powder to the desired product

    5. Addition of calcium stearate to the polyethylene resin followed by compression molding into bars with secondary machining into the desired product

    Corrent answer: 4

     

    Comprehensive Orthopaedic Review states that compression molding and ram extrusion of polyethylene (PE) exhibit equivalent wear rates, but compression molding has a lower susceptibility to fatigue crack formation and propagation. (Of note, Miller's Review states direct compression molding consistently produces better wear rates).

     

    With direct compression the resin is directly molded into the finished part. During ram bar extrusion polyethylene resin is extruded through a die (along with heat and pressure) to create a cylindrical bar that is subsequently machined into the final product. Calcium stearate was once used to prevent corrosion on PE machine parts but resulted in unfused PE particles (fusion defects) which subsequently lowered the mechanical properties of the PE. High dose irradiation (5-15 Mrad) prompts the amorphous regions of the PE to cross-link. The ideal level of crystallinity is around 50%, but a crystallinity over 70% is associated with increased polyethylene failure rates.

     

     

     

    OrthoCash 2020

  42. A new biomaterial has been designed for hard on soft bearings in total hip arthroplasty. The implant company claims this new material has a lower wear rate and greater toughness than highly cross linked polyethylene. Which of the following studies would most accurately evaluate wear between the two materials?

    1. Retrospective review of each material with immediate post-operative radiographs compared to annual follow-up radiographs

    2. Prospective trial comparing each material with radiostereometric analysis

    3. Prospective trial comparing each material with annual CT scans

    4. Prospective trial comparing each material with computer-assisted edge-detection radiographs

    5. Retrospective trial comparing each material with computer-assisted edge-detection radiographs

    Corrent answer: 2

     

    Radiostereometric analysis is the most effective method to evaluate polyethylene wear in arthroplasty. Radiopaque tantalum beads are inserted into the bone in strategic positions surrounding the implants. An immediate post-operative film records the position of the beads. The construct can then be followed with repeated radiographs over time by evaluating the position of the components relative to the beads.

     

    Bragdon et al used radiostereometric analysis to compare wear rates of highly cross linked polyethylene with 28 mm and 36 mm femoral heads. No differences were discovered in femoral head penetration between the two groups.

     

    McCalden et al summarizes the different methods available to evaluate polyethylene wear following total hip arthroplasty. Notably, radiostereometric analysis is the most accurate and precise, but is also expensive, requires technical expertise, and can only be used in a prospective manner.

     

     

     

    OrthoCash 2020

     

  43. A 65-year-old woman complains of worsening left knee pain 7 months following total knee arthroplasty. She reports good pain relief for the initial 5 months following surgery. Physical exam is notable for a stable knee with range of motion from 0-115 degrees. Radiographs are provided in Figures A and B. Which of the following is the most appropriate next step in management?

     

     

     

    1. Nuclear bone scan

    2. One stage revision total knee arthroplasty

    3. Knee MRI

    4. CRP, ESR, WBC

    5. Physical therapy with focus on range of motion and quadriceps strengthening

    Corrent answer: 4

     

    The radiographs demonstrate a well-positioned, cemented total knee arthroplasty with no evidence of loosening. The source of pain cannot be explained by the radiographs or physical exam. Continued pain 6 months following TKA needs to be further evaluated. Laboratory evaluation of inflammatory markers is useful to evaluate for an indolent infection.

     

    Gonzalez et al reviews the evaluation and management of patients who experience pain following total knee arthroplasty. Etiologies of pain include infection, malalignment, instability, stiffness, component loosening, and patellar dysfunction. Appropriate evaluation includes a thorough history and physical exam, radiographic imaging, laboratory evaluation for infection, and possibly advanced imaging studies.

     

    Incorrect Answers:

    1. Nuclear bone scan will not be helpful at 7 months out from surgery because

      there will still be increased uptake from the original surgery.

    2. Without a diagnosis, there is no indication for early revision surgery.

    3. MRI may be useful to evaluate the status of the stabilizing ligaments and extensor mechanism, but would not be helpful in this scenario.

    5. Physical therapy would be indicated if the patient presented with stiffness or mechanical gait difficulties.

     

     

     

    OrthoCash 2020

     

  44. All of the following intra-operative techniques may result in lengthening of the operative leg during total hip arthroplasty EXCEPT:

    1. Increasing the femoral neck offset

    2. Increasing the femoral neck length

    3. Incomplete insertion of a cemented femoral stem

    4. Making a high femoral neck cut just below the femoral head

    5. Bone grafting superior to the acetabulum to seat the cup more inferior Corrent answer: 1

      Limb length discrepancy, especially operative limb lengthening, is a common complication following THA that can lead to patient dissatisfaction. Extending the femoral neck offset does not lengthen the limb. Rather it increases the horizontal distance of the stem away from the femoral head. This increases abductor tension improving stability. It should be noted that weakness of the abductors can result in patient sensation of leg lengthening in the absence of any true LLD.

       

      Bhave et al reviewed 78 patients who complained of poor function 3 months following THA. The patients were evaluated by physical exam, 3-D gait analysis, and isokinetic strength testing. Muscle weakness, muscle tightness, limb-length discrepancies, and limb malalignment were the most common findings.

       

      Maloney et al reviewed the etiologies, complications, and proper management of limb length discrepancy following THA. Careful pre-operative physical exam and pre-operative templating can be useful in mitigating post-operative leg length discrepancy.

       

      Illustration A demonstrates an example of extended femoral neck offset.

       

       

       

       

       

      OrthoCash 2020

       

  45. A 55-year-old patient is scheduled for total knee arthroplasty. A radiograph is provided in Figure A. Each of the following are risk factors for heterotopic ossification EXCEPT?

     

     

     

     

    1. Incorrect sizing of femoral and/or tibial components

    2. Male gender

    3. Obesity

    4. History of trauma

    5. Presence of preoperative osteophytes (hypertrophic arthrosis) Corrent answer: 1

    Heterotopic bone ossification (HO) following TKA has not been associated with valgus knee deformity. HO formation can be problematic both after a THA and TKA, but unlike the hip, it rarely becomes a clinical problem. The overall incidence of HO after TKA varies greatly among published studies, ranging from 15-50%. In the study by Dalury et al, they noted a 15% HO incidence.

    Among those with HO, 95% had osteoarthritis and 5% had a history of inflammatory arthritis. In their study, obese and male patients also had a higher incidence of developing post-operative HO. Toyoda et al in their study showed a significantly higher rate of HO in patients with osteoarthritis and preoperative osteophyte formation (hypertrophic arthrosis). Overall, the signficant risk factors include hypertrophic arthrosis (often seen with trauma), male gender, and obesity.

     

     

     

    OrthoCash 2020

     

  46. Internal rotation of the femoral component in primary total knee arthroplasty may lead to which of the following?

    1. A net lateral patellar tilt and increased lateral subluxation

    2. A decreased Q angle

    3. Patella baja

    4. A loose medial compartment and tight lateral compartment

    5. Balanced medial and lateral flexion gaps Corrent answer: 1

    Internal rotation of the femoral component results in lateral patellar tracking/tilt, and can increase lateral patellar subluxation due to an increase in the Q angle.

     

    Olcott et al performed 100 consecutive CR-TKA's to determine the most appropriate method for femoral alignment necessary to create a rectangular flexion gap. The transepicondylar axis was most accurate in creating a balanced flexion space, however taking 3 degrees external rotation off the posterior condyles was least consistent especially in knees in valgus.

    Heesterbeek et al performed the balanced gap technique on 83 TKA's and found no difference in femoral component rotation of knees with or without ligament releases in extension. They also concluded that preoperative alignment had no influence on femoral component rotation.

     

    Incorrect answers:

    2: Internal rotation increases the Q angle.

    3: Patella baja occurs when the joint line is raised, and causes a loss of knee flexion.

    4: Internal rotation of the femoral component can lead to a tight medial compartment, and loose lateral compartment with the knee in flexion. (Illustration B)

    5: Because the tibia is cut perpendicular to its mechanical axis, the femoral component should be externally rotated to create a rectangular flexion gap. (Illustration A)

     

     

     

     

     

     

     

     

    OrthoCash 2020

     

  47. The preoperative pelvic radiograph of a 63-year-old female with osteoarthritis is shown in Figure A. She undergoes an uncomplicated total hip replacement. Six weeks post-operatively she complains that her right leg is longer than her left, and an AP pelvic radiograph is obtained which is shown in Figure B. Physical exam shows normal post-operative range of motion and strength in both hips. What is the most likely etiology for this patients gait impairment?

     

     

     

     

     

     

     

    1. Hip flexion contracture

    2. Excessive medialization of the acetabular component

    3. Patient's perceived leg length discrepancy

    4. Hip adduction contracture

    5. Malpositioning of the femoral component Corrent answer: 3

    Unilateral osteoarthritis normally causes a leg length discrepancy because of cartilage loss, bone deformity, and soft tissue contractures. Post-operatively, some patients take months to adjust to this apparent lengthening, even though they have been made equal to the non-operative side.

     

    Konyves et al evaluated leg length discrepancy (LLD) and hip function in 90 patients undergoing primary total hip arthroplasty before surgery and at three and 12 months post-operatively. Of the limbs that were over-lengthened (mean 9mm), LLD was perceived in only 43% of the patients after three months and by 33% after 12 months. Lengthening occurred in the femoral component in 98% of the patients with LLD. There was a strong correlation with perceived LLD and poorer function in the Oxford Hip Score.

     

    Figure B shows leg lengths to be equal based on the presence of symmetric teardrops, femoral head heights, and lesser trochanter levels.

     

    Incorrect Answers:

    1, 4: Hip flexion or adduction contractures would functionally shorten the limb so the longer leg would be perceived on the opposite side.

    2: Medialization of the acetabular component would not affect limb length. 5: Figure B does not represent malpositioning of the femoral component.

     

     

     

    OrthoCash 2020

     

  48. Which of the following is true regarding the conversion of hip arthrodesis to total hip arthroplasty?

    1. Implant survivorship is greater than 95% at 20 years following conversion to arthroplasty

    2. Conversion to arthroplasty should not be performed if arthrodesis is more than 15 years old

    3. Function of gluteus medius is predictive of ambulatory status

    4. Rate of complication is equivalent to primary total hip arthroplasty

    5. Incidence of nerve palsy is comparable to primary total hip arthroplasty Corrent answer: 3

    Hip arthrodesis is a viable solution for the very young laborer with debilitating hip disease. Conversion to total hip arthroplasty once the patient has become less active has been clinically successful in relieving adjacent knee and low back pain although complications are more frequent than primary hip arthroplasty.

     

    Incompetence of the gluteal muscle groups was predictive of poor ambulatory outcome. Preoperative EMG studies can be useful to evaluate the competence of the gluteal musculature prior to surgery.

     

    Joshi et al reviewed 208 hip fusions converted to total hip arthroplasties. 79% reported minimal pain and good to excellent range of motion. Arthroplasty survival was 96% at 10 years, 93% at 15 years, and 73% at twenty years.

    Complications included nerve palsy in 7% of hips and heterotopic ossification in 14% of hips.

     

    Hamadouche et al reports on 45 hip fusions converted to arthroplasty. Satisfactory outcome was reported in 91%.

     

    A radiographic example is provided in illustration A (Morsi. Journal of Arthroplasty Sept. 2007).

     

    Incorrect Answers:

    1. Implant survival was 97% at 8 years. Other studies have demonstrated much poorer clinical outcomes including survivorships of less than 70% at 10 years.

    2. There is no time limit for conversion as successful revision arthroplasty has been documented 30 years following arthrodesis.

    1. Complication rates are increased with conversions.

    2. Nerve palsy risk is also increased during conversion.

     

     

     

     

     

    OrthoCash 2020

     

  49. A 65-year-old male complains of continued groin pain 18 months following total hip arthroplasty. The pain is worse with activity, specifically with hip extension during gait. Hip radiographs show no fracture or loosening of the components. Lab values including ESR and CRP are within normal limits, and a hip aspiration yields a nucleated cell count of 500 and no growth on culture. Which of the following is most likely to determine the nature of the continued pain?

    1. Greater trochanteric bursa injection

    2. Repeat aspiration of the hip joint

    3. Local anesthetic injection of the iliopsoas tendon sheath

    4. Ober test on physical exam

    5. Radiographs of the knee Corrent answer: 3

    The clinical presentation is consistent with anterior iliopsoas tendon impingement following total hip arthroplasty. Underlying infection has been ruled out by the appropriate labs and hip aspiration. Groin pain, pain with passive hip extension, and snapping pain in the groin raise the suspicion of iliopsoas tendinopathy. Cross-table lateral imaging and CT scan can be used to evaluate for protrusion of the anterior rim of the acetabular cup causing impingement with the tendon. The diagnosis is confirmed by relief of pain with anesthetic injection of the tendon sheath. Lachiewicz et al reviews the evaluation, diagnosis, and management of iliopsoas impingement and tendinopathy following total hip arthroplasty. Cross-table lateral imaging of the acetabular components position in relation to the anterior rim of the native acetabulum and iliopsoas tendon injections are critical portions of the diagnostic process.

     

    Duffy et al review the evaluation process of patients with continued pain following total hip arthroplasty in an instructional course lecture. A thorough history and physical exam are crucial to successfully determining the origin of pain.

     

    Illustration A is a cross-table lateral radiograph from this article demonstrating prominence of the anterior acetabular rim causing iliopsoas tendinopathy.

     

     

     

     

     

    OrthoCash 2020

     

  50. A posterior cruciate retaining total knee arthroplasty is contraindicated in all of the following patients EXCEPT?

    1. 52-year-old female with severe rheumatoid arthritis of the knee

    2. 73-year-old male with post-traumatic arthritis of the knee and prior patellectomy

    3. 67-year-old male with degenerative arthritis and 10 degree valgus deformity of the knee

    4. 55-year-old male with post-traumatic arthritis of the knee 20 years after bicruciate ligament ruptures

    5. 63-year-old female with a chronic history of steroid treatment of systemic lupus erythematosus and an arthritic knee

    Corrent answer: 3

     

    Posterior cruciate retaining knees require a functional PCL to produce femoral rollback during knee flexion. Posterior-stabilized knees rely on the component to produce femoral rollback. Contraindications to using a posterior cruciate retaining knee include any condition that may render the PCL incompetent at the time of surgery or in the future including prior PCL rupture, inflammatory arthritis, patellectomy, and over-release of the PCL during surgery. Valgus deformity is not a contraindication.

     

    Gonzalez et al reviews the appropriate evaluation and management of the patient with unsatisfactory results following TKA. The most common etiologies include pain, stiffness, and instability.

    Illustrations A and B demonstrate the differences between posterior cruciate retaining knees and posterior-stabilized knees in regards to rollback.

     

     

     

     

     

     

     

     

     

    OrthoCash 2020

  51. Performing an isolated release of the popliteus tendon during a total knee arthroplasty is most appropriate in which of the following scenarios?

    1. Valgus deformity that is tight in extension

    2. Varus deformity that is tight in extension

    3. Valgus deformity that is tight in flexion

    4. Valgus deformity that is tight in both flexion and extension

    5. Varus deformity that is tight in flexion Corrent answer: 3

    An isolated release of the popliteus tendon during TKA is most appropriate in the setting of a valgus knee deformity that is tight in flexion.

     

    Lateral collateral ligament release is most appropriate for a valgus deformity that is tight in both flexion and extension. Iliotibial band release is indicated if the knee is tight in extension. For varus knee deformities, femoral and tibial osteophyte removal, release of the deep MCL, release of the posteromedial corner, release of the attachment of the semimembranosus, and partial superficial MCL release may be sequentially indicated to achieve coronal balancing.

     

    The review article by Favorito et al discusses the controversy over the correct order and sequence for anatomic release of the lateral structures with some authors preferring not to release the LCL as an initial step.

     

    The case-control study by Krackow et al found good success with a lateral release in valgus deformity that had competent medial structures. In cases with incompetent medial structures they reported good success with reconstruction and imbrication of the medial structures.

     

     

     

    OrthoCash 2020

     

  52. The use of vancomycin impregnated bone cement during total knee arthroplasty is most strongly recommended for which of the following patients?

    1. 68-year-old male undergoing second stage of revision arthroplasty for deep infection

    2. Primary TKA in a 55-year-old female with BMI of 40

    3. Primary TKA in a diabetic 70-year-old male

    4. 67-year-old male with posttraumatic arthritis and retained hardware

      undergoing primary TKA

    5. 53-year-old female with rheumatoid arthritis undergoing primary TKA Corrent answer: 1

    Antibiotic impregnated bone cement is used with the goal of reducing deep infection following arthroplasty. The benefits must be weighed against the risks including altering the mechanical properties of the cement, risk of systemic absorption of antibiotics, increased risk of aseptic loosening, and financial cost. The evidence to date most strongly supports the use of antibiotic cement in patients undergoing revision arthroplasty.

     

    Chiu et al conducted a prospective randomized study of 183 revision knee arthroplasties in which one group of patients' components were secured with vancomycin impregnated cement and the other group received cement without antibiotics. Significantly less deep infections occurred in the group with vancomycin impregnated cement (6 infections vs. 0).

     

    Namba et al performed an observational study of 22,889 primary TKA's from a large HMO. Nine percent received antibiotic loaded bone cement. The rate of infection was higher in patients who received antibiotic loaded bone cement.

    Surprisingly, in analyzing the results in diabetic patients only, there was no difference in the rate of post-operative deep infection. There are significant limitations to this study as patients with a higher risk for infection were much more likely to receive antibiotic loaded bone cement.

     

     

     

    OrthoCash 2020

     

  53. A design surgeon has created a new polyethlyene acetabular liner for total hip arthroplasty. Early clinical outcomes are comparable to conventional total hip arthroplasty. At 4 year follow-up, radioisometric analysis reveals 0.05 mm of annual wear in the surgeon's trial patients. Which of the following statements can be made from this data?

    1. Osteolysis and subsequent component loosening will be minimal if the wear rate continues

    2. Osteolysis and subsequent component loosening will be greater than conventional hip arthroplasty if the wear rate continues

    3. Component positioning should have no effect on wear rates of these bearings

    4. All total hip arthroplasties should use this new polyethylene

    5. Patient satisfaction will be greater at 10 year follow up

    Corrent answer: 1

     

    Polyethylene wear rates have shown association with osteolysis and subsequent component loosening. Wear rates above 0.1 mm per year are at significant risk of osteolysis.

     

    Dumbleton et al performed a meta-analysis of the literature concerning wear rates and osteolysis. They concluded that osteolysis was rarely observed when the wear rate was less than 0.1 mm per year.

     

    Sakalkale et al reported on 10 year follow up of 60 total hips performed with a cementless, tapered, circumferentially coated femoral stem. They reported no osteolysis of the femoral component even though there was a high incidence of acetabular osteolysis.

     

    Sochart et al reviewed 235 total hip arthroplasties performed by Charnley with 20 year follow-up. Osteolysis and revision was significantly associated with polyethylene wear rates. Average wear rates for hips that did not go on to revision was 0.09 mm per year compared to 0.19 mm per year for hips that were subsequently revised for loosening.

     

     

     

    OrthoCash 2020

     

  54. The polyethylene cup shown in Figure A has been sterilized in packaging that contains oxygen. Which of the following statements is true about the process of polyethylene sterilization?

     

     

     

     

     

    1. Irradiation of polyethylene in argon quenches free radicals

    2. Polyethylene sterilization with ethylene oxide causes cross-linking of the polymer and minimizes oxidation following implantation

    3. Oxidation occurs following polyethylene implantation regardless of sterilization process

    4. Irradiation of polyethylene in nitrogen quenches free radicals

    5. Polyethylene sterilization with gas plasma causes cross-linking of the polymer and minimizes oxidation following implantation

    Corrent answer: 3

     

    The critical point in this question is that polyethylene oxidation occurs in vivo regardless of sterilization technique (although some methods of sterilization accelerate the oxidation process). Figure A represents a polymer that has been irradiated in oxygen. This method of sterilization has unacceptably high rates of polymer chain degradation and oxidation. Irradiation of polyethylene sterilizes the polyethylene. When the irradiation occurs in the presence of argon or nitrogen, free radicals are created leading to cross-linking of the polymer, and a more abrasion resistant product. However, irradiation in the presence of oxygen leads to excessive free radical production and the polyethylene becomes more brittle with higher rates of early failure.

    Sterilization techniques including ethylene oxide and gas plasma do not create free radicals and there is no effect on polymer cross-linking.

     

    The articles by Medel and Currier are both retrieval analyses of polyethylene implants. These studies concluded that there are lower oxidation rates and oxidation potential in components irradiated in inert gas compared to oxygen. However, it is important to note that oxidation occurs in vivo regardless of sterilization technique, and oxidation may contribute to fatigue failure of the implanted polyethylene components. To avoid long-term oxidation, highly cross-linked UHMWPEs can undergo postirradiation melting or annealing (process termed quenching) to eliminate or reduce residual free radicals.

    Annealing below the melt temperature can have less effect on the cross-linked polyethylene mechanical properties.

     

    Currier et al concludes that the pathway for polymer oxidation that occurs in vivo is similar to the pathway witnessed during irradiation in oxygen.

     

     

     

    OrthoCash 2020

     

  55. A 65-year-old male with chronic right hip pain undergoes the procedure seen in Figure A utilizing a posterior approach. Which of the following hip positions would put the patient at the greatest risk for dislocation?

     

     

     

    1. Abduction and external rotation

    2. Flexion and external rotation

    3. Flexion and internal rotation

    4. Extension and internal rotation

    5. Extension and external rotation Corrent answer: 3

    The incidence of dislocation is highest in the first year after arthroplasty and then continues at a relatively constant rate for the life of the arthroplasty. At risk positions are based on component positioning and surgical approach. Postoperative hip instability can be caused by several factors including soft tissue imbalance, component malposition, and limb position.

     

    Hip extension and external rotation put the hip at risk for anterior dislocation following an anterior approach. Hip flexion and internal rotation put the hip at risk for posterior dislocation following a posterior approach.

     

    Soong et al review the common etiologies for hip dislocations following primary THA which can be divided into both patient and surgical factors. Patient factors include: neuromuscular and cognitive disorders, patient noncompliance, and previous hip surgery. Surgical factors include: soft-tissue tension, component positioning, impingement, head size, acetabular liner profile, and surgeon experience.

    Mahoney et al also divides dislocation risks into patient and surgical factors. Critical factors under surgeon control include component orientation and restoration of soft-tissue tension. Prosthetic factors lowering the risk of dislocation include increasing femoral head size, maximizing the head neck ratio, and optimizing the placement of the acetabular component.

     

     

     

    OrthoCash 2020

     

  56. Isolated transection of the posterolateral(PL) bundle of the ACL has what effect on anterior tibial translation and rotatory stability of the knee?

    1. Increased tibial translation and rotation at 30 degrees of flexion

    2. Increased tibial translation and rotation at 90 degrees of flexion

    3. Increased tibial translation at 30 degrees of flexion and increased rotation at 90 degrees of flexion

    4. Increased tibial translation at 90 degrees and negligible effect on rotatory stability

    5. Increased tibial translation at 30 degrees and negligible effect on rotatory stability

    Corrent answer: 1

     

    Isolated transection of the posterolateral(PL) bundle of the ACL leads to Increased tibial translation and rotation at 30 degrees of flexion.

     

    Zantop and Herbort et al. determined the influence of isolated deficiency of the AM or PL bundle of the ACL on the resulting knee kinematics. They found that transection of the anteromedial bundle leads to increased anterior tibial translation at 90 degrees of knee flexion, whereas transection of the posterolateral bundle shows an increased anterior tibial translation as well as a combined rotatory instability at 30 degrees. This rotatory stability provided by the PL bundle prevents the pivot shift phenomenon found in ACL deficient knees.

     

    Zantop and Wellman et al. performed a cadaveric study to determine the distances from the tibial and femoral center of the native AM and PL bundle to the articular cartilage and meniscus. They concluded that the center of the femoral PL bundle is shallow and inferior to the AM bundle, and on the tibia the AM bundle lies anterior when compared with the typical single-bundle ACL tunnel.

     

    Illustrations A and B highlight the ACL bundle anatomy.

     

     

     

     

     

     

     

     

    OrthoCash 2020

     

  57. When performing a total knee arthroplasty using intramedullary referencing, the knee is stable at full extension, but it will not flex past 90 degrees. Which of the following adjustments can achieve satisfactory range of motion and stability in flexion and extension?

    1. Downsizing the tibial insert

    2. Placing posterior femoral augments

    3. Resecting more distal femur

    4. Downsizing the femoral component

    5. Performing a medial tibial reduction osteotomy Corrent answer: 4

    Of the options listed, downsizing the femoral component is the only modification that will loosen the flexion gap.

     

    Balancing flexion and extension gaps in knee arthroplasty can be a challenge, especially in a revision setting. For a TKA to achieve greater flexion, options include downsizing the femoral component, recessing the PCL, and increasing the posterior tibial slope. More flexion can also be achieved by resecting more

    tibia or decreasing the height of the tibial insert, however, changes to the tibial side affect both the flexion and extension gaps.

     

    Dennis et al. review revision knee arthroplasty, and in particular, provide a complete discussion of gap balancing techniques. They note that appropriate balance is key to obtaining excellent outcomes, and that augmentation or increased constraint is often needed in revision cases.

     

    Ries et al. present a technique paper looking at balancing only in revision total knee arthroplasty. They make note of the need of full exposure to properly evaluate balancing and soft tissue restrictions to proper balance.

     

    Incorrect Answers:

    Answer 1: Downsizing the tibial insert would increase both the flexion and extension gaps; the knee would be unstable in extension.

    Answer 2: Placing augments on the posterior femur would make the flexion gap even tighter.

    Answer 3: Resection of more distal femur would loosen the extension gap without changing flexion.

    Answer 5: Tibial reduction osteotomy on the medial side would result in more laxity to valgus stress, but it would not symmetrically increase the flexion gap.

     

     

     

    OrthoCash 2020

     

  58. In order to determine the boundaries of the posterior-superior safe zone for acetabular screw placement during THA, a line is initially drawn through which of the following two anatomic landmarks, represented by dots on the illustration?

     

     

     

     

    1. A and C

    2. B and C

    3. D and C

    4. C and E

    5. A and E Corrent answer: 1

    Acetabular quadrants are formed from a line extending from the ASIS (Marker A) through the center of the acetabulum (Marker C) to the posterior fovea, forming acetabular halves. The second line is drawn perpendicular to the first at the center of acetabulum, forming four quadrants (Illustration A).

     

    In their initial cadaver study in 1990, Wasielewski et al studied the relationship of the at risk structures during transacetabular screw placement. The quadrants and structures most at risk are found in Illustration B.

     

    In a later study by the same group in 2005, they investigated the intrapelvic structures at risk in a “high hip” center which was reamed superiorly to a distance ½ of the native acetabular diameter. They found that in the new “high hip” the anatomic anterosuperior and anteroinferior quadrants were positioned in the posterosuperior quadrant. Therefore the safest place for screw placement was the peripheral 1/2 of the posterosuperior quadrant.

     

     

     

     

     

     

     

     

    OrthoCash 2020

     

  59. A 68-year-old right handed male golfer presents with significant left knee pain which has not been amenable to conservative management. A radiograph is shown in Figure A. He is interested in pursuing total knee arthroplasty (TKA). What can this patient expect with regards to his golf game after undergoing this procedure?

     

     

     

    1. A significant rise in his handicap

    2. No change in his drive distance

    3. Decreased pain compared to undergoing a right TKA

    4. A significant chance of having severe pain during play

    5. Patients are required to use a cart while golfing Corrent answer: 1

    Active golfers who undergo total knee arthroplasty (TKA) typically have a significant increase in their handicap when they return to the game.

     

    Mallon et al studied 83 (80 of which were right handed) active golfers who underwent TKA and found that they invariably experienced a significant rise in their handicap (mean +4.6 strokes) and also a decrease in the length of their drives. Approximately 15% of the cohort experienced a mild ache while playing, and golfers with left TKA's had more difficulty with pain during and after play than did golfers with right TKA's. It also should be stated that statistically significant increased pain ratings occur in golfers with a TKA on the target-side knee. Finally, almost 90% of the patients in this study utilized a cart while playing post-operatively.

     

    Mallon et al also evaluated the effect of total hip arthroplasty (THA) on the game of avid golfers. They found that hybrid and uncemented primary THA's had lower rates of radiographic loosening in active golfers when compared to

    cemented THA's. However, symptoms of pain while playing or after playing did not differ among these groups.

     

    Arbuthnot et al sent golfing habit questionnaires to 750 consecutive avid golfers who had undergone total hip arthroplasty. They found no significant change from their predisease state to their 1-year postoperative golf performance and level of participation.

     

     

     

    OrthoCash 2020

     

  60. A 73-year-old female undergoes a total hip arthroplasty (THA) using a cemented stem design shown in Figure A. She returns to clinic 3 years post-operatively with signifcant thigh pain. Current radiographs, shown in Figure B, demonstrate femoral subsidence. What affect does this have on the biomechanics of her THA?

     

     

     

     

     

     

    1. Excursion distance is decreased

    2. Primary arc range is increased

    3. Abductor complex tension is decreased

    4. Joint reactive forces are decreased

    5. Femoral offset is increased Corrent answer: 3

    Femoral stem subsidence effectively decreases the neck length of the prosthesis resulting in a lax abductor complex which causes an increase in the joint reactive force. This decrease in leg length can also lead to increased hip instability.

     

    Kim et al performed clinical, radiographic, and computed tomography

    examinations on 1268 patients to determine the prevalence of and factors contributing to dislocation after using a primary cementless total hip arthroplasty system. The significant risk factors for dislocation (3.6% rate overall) were female sex, advanced age, high ASA score, fracture of the femoral neck, nonrepair of the posterior soft-tissue sleeve, low or high cup anteversion, low or high stem anteversion, and low height of hip rotation center.

     

    Nishii et al evaluated component positioning in a series of THA patients who underwent the same surgical procedure to determine if there was a correlation with the occurrence of postoperative dislocation. They found that cup anteversion is one of the important factors for risk of dislocation, and that intentionally placing the cup at low anteversion to compensate for high femoral neck anteversion may predispose the hip to postoperative dislocation.

     

    Illustration A shows a free body diagram of the hip joint. The magnitude of the joint reaction force depends critically on the ratio of (d1:d2). As d2 decreases due to less offset, such as in this question, and body weight remains the same, the joint reaction forces increase.

     

    Incorrect Answers:

    Answer 1: The excursion distance (the distance the femoral head must travel to dislocate) is unchanged.

    Answer 2: The primary arc range (arc range before impingement) may be decreased due to early trochanteric impingement, but is not increased.

    Answer 4: A more lax abductor complex results in an increased joint reactive force.

    Answer 5: Femoral offset (distance from center of femoral head to long axis of femur) is unchanged as the radiographs show subsidence primarily in a caudal direction.

     

     

     

     

     

    OrthoCash 2020

     

  61. A 68-year-old woman underwent a right total knee arthroplasty 5 years ago and has increasing right knee pain over the past 2 months. Radiographs are seen in Figures A and B, respectively. Laboratory studies demonstrate a C-reactive protein of 10 mg/dL (normal < 2.0 mg/dL) and an erythrocyte sedimentation rate of 50 mm/h (normal < 20 mm/h). Knee aspiration shows white blood cell count of 3,400/mm3 with 90% polynuclear cells. The patient's gram stain and cultures are negative. What is the most appropriate next step in management?

     

     

     

     

     

     

     

     

    1. Irrigation and debridement with polyethylene spacer exchange

    2. One-stage revision

    3. Two-stage revision

    4. One-stage revision with antibiotic impregnated cement

    5. One-stage revision with direct antibiotic infusion into knee joint via hickman catheter

    Corrent answer: 3

     

    The patients history, labs, and imaging are consistent with an infected total joint prosthesis. Two-stage resection and replacement arthroplasty for hip and knee arthroplasty is the gold standard for treatment of infection beyond 4 weeks. Reimplantation within 2 weeks has a 35% success rate compared to success rates of 80% with delayed reimplantation (>6 weeks) and more extensive antimicrobial therapy.

     

    Spangehl et al conducted a Level 2 study of patients being diagnosed with prosthetic hip infection. They found that combination of a normal erythrocyte sedimentation rate and C-reactive protein level has the highest negative predictive value for infection. They found the gram stain to be unreliable and intraoperative frozen sections useful only in equivocal cases.

     

    Schinsky et al evaluated 55 total hip infections and 146 non-infected total hip patients to evaluate which markers are most reliable for diagnosis of prosthetic infection. They found that a synovial fluid cell count of >3000 white blood cells/mL was the most predictive perioperative testing modality when

    combined with an elevated preoperative erythrocyte sedimentation rate and C-reactive protein level.

     

    It should be noted that similar OITE questions in years past have cited a synovial fluid aspiration with WBC of >1100 cells/mm3 and PMN > 64% as suggestive of infection based on the article by Ghanem et al.

     

     

     

    OrthoCash 2020

     

  62. A 72-year-old man reports persistent, progressively worsening pain in his hip after undergoing a total hip arthroplasty 15 months ago. A current AP hip radiograph is shown in Figure A. What is the next most appropriate step in the care of this patient?

     

     

     

     

     

    1. IV Antibiotics

    2. Obtain serum metal ion values

    3. Obtain ESR, CRP, and WBC

    4. Obtain CT and MRI of the hip

    5. Urgent debridement and component explantation Corrent answer: 3

    The key to this question is recognizing the radiographic findings of periprosthetic infection. Figure A shows new, lacey periosteal bone formation about the metadiaphyseal region of the femur with scalloping resorption. This is suggestive for a deep periprosthetic infection. Initial work-up starts with ESR, CRP and WBC. If these are elevated, joint aspiration to confirm periprosthetic infection is warranted.

    Fitzgerald in this review article discusses 3 types of periprosthetic infection. Stage I is an acute postoperative infection that is radiographically silent. Stage II infections occur 6-24 months after the primary procedure and represent indolent infections that manifest radiographically with new bone formation as described above. Stage III infections occur more than 2 years after the primary procedure and are the result of hematogenous seeding of the joint via recent dental or surgical procedure.

     

    Meehan et al in this review article discuss the use of Vancomycin for preoperative prophylaxis in total joint arthroplasty. At their institution, staph aureus and staph epidermidis were resistant to cefazolin in 50 and 70% of cases, respectively.

     

    Incorrect Answers:

    Answer 1: IV antibiotics should be withheld until a diagnosis is made, or a culture has been obtained.

    Answer 2: The utility of serum metal ion levels in patients undergoing metal-on-metal hip arthroplasty is still unclear.

    Answer 4: Advanced imaging may be helpful in diagnosing soft tissue reactions or subtle fractures, however, they are not the most appropriate next step in this situation.

    Answer 5: Urgent debridement should only be considered after joint aspiration is performed to confirm the diagnosis of infection.

     

     

     

    OrthoCash 2020

     

  63. Failure to identify a hypoplastic lateral condyle in a valgus knee will result in which of the following errors if a posterior condylar referencing guide is used for total knee arthroplasty?

    1. External rotation of the femoral component

    2. External rotation of the tibial component

    3. Internal rotation of the femoral component

    4. Internal rotation of the tibial component

    5. Internal rotation of the tibial and femoral components Corrent answer: 3

    Failure to identify a hypoplastic lateral condyle will lead to internal rotation of the femoral component if a posterior condylar referencing guide is used for total knee arthroplasty.

     

    The posterior condylar axis of an average knee rests in 3 degrees of internal

    rotation compared to the transepicondylar axis. Posterior referencing guides are set with 3 degrees of external rotation to compensate for this discrepancy. In the case of a hypoplastic lateral condyle, greater than 3 degrees of internal rotation will be present. If the surgeon does not identify this abnormality and uses a posterior referencing guide, then the cuts will be made with too much internal rotation.

     

    Laskin et al. review techniques of total knee arthroplasty. Pertinent to this question, the posterior condylar axis may not be a suitable landmark to guide the posterior cut in patients with deformity. A hypoplastic lateral condyle will create a cut that is internally rotated if only 3 degrees of external rotation is applied.

     

    Illustration A demonstrates that the line perpendicular to the AP axis (Whiteside's Line) is the neutral rotational axis (approximately equal to the transepicondylar axis). The femoral component should be placed in 3° of external rotation in relation to the posterior condylar axis to maintain symmetric flexion gap.

     

     

     

     

     

     

    OrthoCash 2020

     

  64. A 64-year-old male undergoes acetabular revision of his failed total hip arthroplasty using a large uncemented component. Postoperatively he is noted to have a foot drop and radicular pain in the operative extremity. A CT scan of the hip is obtained and reveals screw penetration into the sciatic notch. Where was this screw most likely inserted in the acetabulum?

    1. Anterior superior quadrant

    2. Through the medial wall

    3. Anterior inferior quadrant

    4. Posterior superior quadrant

    5. Through the femoral nerve Corrent answer: 4

    Long screws placed into the posterior superior or posterior inferior quadrant may pass into sciatic notch and endanger the sciatic nerve and superior gluteal vessels. This is particularly a risk in revision surgery when the acetabular component may be placed in a high hip center position, as the sciatic nerve is at increased risk when placing transacetabular screws posteriorly.

     

    Meldrum et al evaluated the quadrant system used to guide screw placement in primary cadaveric uncemented total hip surgery in the high hip center, jumbo component, and 3 designs of reinforcement rings. Of all the acetabular revision scenarios tested, the high hip center showed increase risk of neurovascular injury in the center and anterior portions of the posterior superior quadrant. All of the other implants met the standard, non-revision scenario quadrant recommendations.

     

    Wasielewski et al performed an anatomical and radiographic study to determine the safest zones in the acetabulum for the transacetabular placement of screws during uncemented acetabular arthroplasty. They found that the posterior superior and posterior inferior acetabular quadrants are relatively safe for the transacetabular placement of screws. They also determined that the anterior superior and anterior inferior quandrants should be avoided whenever possible, because screws placed improperly in these quadrants may endanger the external iliac artery and vein, as well as the obturator nerve, artery, and vein.

     

    Illustration A demonstrates the four quadrants of the acetabulum relevant to transacetabular screw placement.

     

     

     

     

     

    OrthoCash 2020

     

  65. During trialing for a cruciate-sacrificing total knee arthroplasty, the surgeon notes an imbalance between the flexion and extension gaps with significant flexion instability. The extension gap is well balanced. Which of the following options is the best intra-operative solution?

    1. Downsize the femoral component

    2. Downsize the tibial component

    3. Upsize the femoral component and add posterior augments

    4. Upsize the tibial component

    5. Move the femoral component more anteriorly Corrent answer: 3

    Understanding flexion/extension gaps in total knee arthroplasty is paramount to patient success. Treatment for flexion instability consists of either increasing the size of the femoral component, shifting the femoral component posteriorly, or increasing the size of the polyethylene and then dealing with the tight extension gap.

     

    As discussed by Ries et al, increasing the size of the femoral component will change the anterior/posterior size of the component without changing the

    proximal/distal size of the component, thus changing only the flexion gap. With upsizing the femoral component, you will likely have to add augments as the bone cut will not match a larger component. Moving the femoral component more posterior will accomplish the same goal of decreasing only the flexion gap. Increasing the poly thickness will change both the flexion and extension gaps, and in this patient, the surgeon would then have to address the tight extension gap by resecting more distal femur and/or releasing the posterior capsule.

     

     

     

    OrthoCash 2020

     

  66. Patella baja is most likely to occur after which of the following procedures?

    1. Arthroscopic ACL reconstruction with cadaver allograft

    2. PCL reconstruction using tibial inlay technique

    3. High tibial osteotomy

    4. MPFL reconstruction with semitendinosus autograft

    5. Total knee arthroplasty (TKA) Corrent answer: 3

    Patella baja is a well known complication of high tibial osteotomies, especially opening wedge osteotomies. This procedure raises the tibiofemoral joint line and can cause retropatellar scarring and tendon contracture, decreasing the distance of the patellar tendon from the inferior joint line.

     

    Wright et al found that the patellar height after opening wedge medial tibial osteotomies decreased the patellar height in 100% of their patients. They explain that the decrease in distance between the patella and the tibiofemoral joint line following medial opening wedge proximal tibial osteotomy is a function of joint line elevation. Their results are important when considering possible future TKA in these patients, as patella baja may have deleterious effects on patellofemoral biomechanics for future procedures.

     

    Kolb et al studied the short-term results of opening-wedge high tibial osteotomies with locked plate fixation for patients with medial compartment arthrosis. Their results suggestted that opening-wedge high tibial osteotomy for presurgical varus deformity allowed for good short-term results and correction of the deformity.

     

    Illustration A shows an example of this complication.

     

     

     

     

     

    OrthoCash 2020

     

  67. A patient undergoes the procedure depicted in Figures A and B with standard components (non-gender specific). Which of the following outcomes most appropriately describes the difference in females compared to males for this procedure?

     

     

     

     

     

     

    1. Greater implant survivorship

    2. Decreased WOMAC scores

    3. Increased rate of extensor mechanism rupture

    4. Increased postoperative pain

    5. Increased component osteoloysis Corrent answer: 1

    Females undergoing total knee arthroplasty with standard (non-gender specific) components show improved implant survivorship compared to males.

     

    MacDonald et al performed a Level 2 study of 3817 patients who underwent 5279 primary total knee replacements (3100 female, 2179 male) with a minimum of 2 years followup. They found that women demonstrated greater implant survivorship, greater improvement in WOMAC scores, equal improvements in SF-12 scores, and less improvement in only the Knee Society function and total scores.

    Greene discusses the role of gender-specific implant designs that are currently marketed and their benefit to patients. The article concludes that the amount of attention that implant manufacturers have focused on female specific components(e.g. narrower M/L dimensions, decreased thickness of the anterior flange, and increased trochlear groove angle) is of interest, considering that there is no evidence suggesting that females have inferior outcomes with standard components.

     

     

     

    OrthoCash 2020

     

  68. A 64-year-old female with rheumatoid arthritis is undergoing a left total knee arthroplasty. During the tibial cut, a ligament is transected by a reciprocating saw. The ligament is not able to be repaired. The surgeon is balancing the tibial and femoral cuts with sizing blocks and finds that the knee has valgus instability greater than 1cm in full extension. Which implant offers the most appropriate level of constraint while limiting the amount of implant-host interface stresses?

    1. Unlinked constrained (varus-valgus constrained)

    2. Fixed bearing PCL-substituting (posterior-stabilized)

    3. Mobile bearing PCL-substituting (posterior-stabilized)

    4. PCL-retaining (cruciate-retaining)

    5. Rotating-hinge constrained

     

    Corrent answer: 1

     

    The history and intraoperative examination are consistent with an iatrogenic MCL injury that is irreparable. An unlinked constrained (varus-valgus constrained) prosthesis has a tall tibial post and a deep femoral box, which provide more inherent coronal plane stability than do standard cruciate retaining or cruciate-substituting prostheses. Because there is no axle connecting the tibial and femoral components, these implants are sometimes referred to as unlinked constrained implants.

     

    Morgan et al discuss in their Level 5 review that the added degrees of implant stability confer disadvantages. As the amount of constraint increases, stress transmitted to the modular implant-host or prosthesis-host interface also increases. The heightened stress may result in increased backside polyethylene wear in modular tibial components or in early implant loosening, and ultimately to failure. Therefore, a rotating-hinge constrained knee would offer sufficient stability for a MCL deficiency but offers more constraint than is necessary and appropriate.

    Gonzalez et al present a Level 5 reivew stating that the primary causes of failure of total knee arthroplasty include pain, postoperative stiffness, and instability. They state that medial-lateral instability can be a product of improper implant balancing or deficient medial or lateral collateral ligaments.

     

    Illustration A shows a varus-valgus unlinked constraint knee implant and Illustration B shows a rotating hinge constraint knee implant. Illustration C and D show a cruciate-retaining implant on the left and a cruciate-substituting implant with femoral box and tibial polyethylene post on the right.

    Illustration E depicts a cadaveric right knee with a MCL (sutured in picture) that has been transected during a tibial cut.

     

     

     

     

     

     

     

     

     

     

     

     

     

     

     

     

    OrthoCash 2020

  69. During trialing for a cruciate-retaining total knee arthroplasty, the surgeon is unable to fully extend the knee and is left with a 15 degree flexion contracture. The flexion gap is well balanced. Which of the following options will create a knee that is balanced in both flexion and extension?

    1. Recess the PCL

    2. Increase the tibial slope

    3. Decrease the size of the femoral component

    4. Resect more distal femur

    5. Resect more proximal tibia Corrent answer: 4

    Flexion/extension gap balancing is crucial to the success in total knee arthroplasty. The inability to achieve full extension suggests extension tightness. This can be improved by either resecting more distal femur or releasing the posterior capsule from the femoral insertion. While resecting more proximal tibia will improve the extension gap, it will loosen the flexion gap and require either upsizing of the femoral component with placement of posterior augments or translation of the femoral component posteriorly.

    Recessing the PCL and increasing the tibial slope would be appropriate for flexion not extension tightness.

     

    In their review, Ries et al discuss flexion/extension balancing, focusing on revision total knee arthroplasty.

     

     

     

    OrthoCash 2020

     

  70. A 62-year-old female has persistent activity related anterior groin pain 10 months after total hip arthroplasty (THA). Infection workup is negative. New radiographs are unchanged compared to the intial films provided in Figures A and B. Pain is temporarily relieved following an injection of lidocaine and cortisone into the iliopsoas tendon sheath. What is the next appropriate treatment option?

     

     

     

     

     

     

    1. Indefinite activity modification

    2. Iliopsoas tendon release

    3. Femoral component revision

    4. Acetabular component revision

    5. Femoral and acetabular component revision Corrent answer: 2

    After diagnosis of iliopsoas impingement, iliopsoas muscle tenotomy or resection is the treatment of choice if radiographs are within normal limits. In contrast, if imaging shows anterior acetabular overhang (as shown in Illustration A), then acetabular revision would be the next appropriate step in

    management.

     

    Lachiewicz et al provide a great review on iliopsoas impingement after THA. Anterior iliopsoas impingement can cause functional disability after total hip arthroplasty. The diagnosis may be confirmed by one or more imaging studies, including a cross-table lateral radiograph, computed tomography, magnetic resonance imaging, and ultrasonography, in combination with a confirmatory diagnostic injection into the iliopsoas sheath. Treatment, consisting of release or resection of the iliopsoas tendon, alone or in combination with acetabular revision for an anterior overhanging component, usually provides permanent pain relief.

     

    Trousdale et al also reviewed cases of iliopsoas impingment after THA. They studied two cases of iliopsoas tendinitis following THA due to a malpositioned, uncemented, metal-backed acetabular component. In cases of anterior acetabular overhang, acetabular revision to reduce anterior impingement is the appropriate management.

     

    Illustration A shows an example of anterior acetabular overhang which would require acetabular revision if symptomatic.

     

     

     

     

     

     

    OrthoCash 2020

     

  71. Figure A shows an AP hip radiograph of a 72-year-old woman who had had a right total hip arthroplasty fifteen years previously. CT imaging of the affected hip shows non-contained defects in both the anterior and posterior columns of the peri-acetabular region affecting greater than 50% of the weight bearing surface. Which of the

    following revision procedures would restore the most acetabular bone stock and be most appropriate for this patient?

     

     

     

     

    1. Morselized allograft and/or autograft bone, combined with a cemented acetabular component

    2. Acetabular revision with use of a bilobed cementless component and morselized allograft

    3. Morselized allograft and/or autograft bone, combined with a cementless acetabular component

    4. Revision using an ilioischial reconstruction ring acetabular component and structural corticocancellous graft

    5. Revision using a roof ring acetabular component and structural corticocancellous graft

    Corrent answer: 4

     

    In cases of minor, contained, acetabular defects, morcellized allograft and/or autograft bone, combined with a cemented or cementless acetabular component can lead to successful reconstruction. However, these constructs do not confer enough stability when the loss of bone stock is more extensive and encroaches on the acetabular columns, or compromises >50% of the weight-bearing surface. A bilobed implant is a viable option in these scenarios, however these components replace lost bone with artificial materials rather than restoring acetabular bone stock making revision very difficult. Roof ring

    acetabular components have been largely replaced by cementless cups fixed with multiple screws, and do not offer the same degree of fixation stability found with reconstruction rings for large bone defects.

     

    Goodman et al review the complications, management, and outcome of a consecutive series of 61 ilioischial reconstruction rings performed by 1 surgeon over a 15-year period. On the acetabular side, allograft failure was the most common complication.

     

    Illustration A shows an example of an ilioischial reconstruction ring, and Illustration B shows this reconstruction ring in situ.

     

     

     

     

     

     

     

     

    OrthoCash 2020

  72. A 56-year-old man reports progressively worsening left knee pain after undergoing total knee arthroplasty 6 years ago. He was initially very happy with his progress, but 18 months after surgery he began to have knee pain. Radiographs are shown in Figures A and B. Laboratory values reveal a C-reactive protein of 0.1 mg/dL (normal 0.0-0.6 mg/dL) and an erythrocyte sedimentation rate of 3 mm/h (normal 0-15 mm/h). An aspiration of the knee reveals 157 leukocytes/ml with 18% polymorphonucleocytes. What is the most appropriate next step in management?

     

     

     

     

     

    1. One-stage revision

    2. Irrigation and debridement with polyethylene spacer exchange

    3. Antibiotic impregnated cement spacer placement

    4. Two-stage revision

    5. Broad-spectrum, empiric oral antibiotics Corrent answer: 1

    The history, radiographs, and laboratory values are consistent with aseptic loosening. The lateral radiograph demonstrates a thin cement mantle that has separated from the prosthesis. The question stem details that infection is not likely given the normal serology and aspirate values. A one-stage revision of the arthroplasty components is the most appropriate next step in management among the options provided.

    Brown and Bartel present a Level 5 review of the intrinsic and extrinsic factors that can effect wear behavior in arthroplasty bearing surfaces. They state that increased sliding distance, third body wear, and impingement can be sources for accelerated wear rates of bearings.

     

    Gonzalex and Mekhail present a Level 5 review discussing the etiologies for a failed joint arthroplasty. Sources identified for continued pain were aseptic loosening, component failure, patellar dysfunction, infection, or complex regional pain syndrome.

     

     

     

    OrthoCash 2020

     

  73. A 50-year-old woman underwent cemented total knee arthroplasty 3 weeks ago. She reports that she has 1 week of drainage the size of a quarter on a gauze pad that she places over the incision three times daily. Her body mass index is 53 and her medical problems include hypertension and type 2 diabetes. Blood work shows a CRP of

    1.1mg/L (normal 1-3mg/L). Knee aspiration yields WBC of 673 cells/mm(3) with 30% polymorphonucleocytes, and a negative gram stain. There is no surrounding erythema but there is a 1cm area at the inferior aspect of the wound that has a large amount of serous drainage able to be expressed. She has a painless range of motion is 0° to 117°. What would be the next most appropriate step in management?

    1. Removal of all components with antibiotic spacer placement and staged revision

    2. One-stage irrigation and debridement with removal of components to a cementless prosthesis

    3. Empiric oral antibiotics for 4 weeks and steri-strips over the area of drainage

    4. Surgical exploration with debridement and possible polyethylene exchange

    5. Bone scan and repeat aspiration with empiric intravenous antibiotics for 4 weeks

    Corrent answer: 4

     

    Irrigation and débridement with possible polyethylene exchange is the most appropriate treatment for persistent drainage within a few weeks from total joint arthroplasty surgery.

     

    Malinzak et al performed a Level 4 review of 8494 patients undergoing a total knee arthroplasty. They found that patients with a body mass index greater

    than 50 had an increased odds ratio of infection of 21.3 (P < .0001). Diabetic patients were 3 times as likely to become infected compared to nondiabetic patients (P = .0027).

     

    Rasul et al performed a Level 4 review of 24 patients for a duration of 2 years with total knee arthroplasty infections. They found that patients with chronic (>1 month) deep infections were successfully treated 75% with debridement, intravenous antibiotics, tobramycin-impregnated polymethylmethacrylate beads, and delayed exchange arthroplasty with mean interval of staged reimplantation being 8 weeks.

     

     

     

    OrthoCash 2020

     

  74. A 47-year-old man presents with 1 week of left leg pain. 6 months prior he underwent a vascularized free-fibula bone graft from his left leg to his right hip for avascular necrosis. The pain is located at the level of his donor site and is worse with weight-bearing and relieved by rest. Physical exam shows focal tenderness over his tibia. A lateral radiograph from the day of presentation is shown in Figure A. WBC, ESR, and CRP are all within normal limits. What is the next best step in management to confirm the diagnosis?

     

     

     

     

    1. Compartment pressure measurements

    2. CT scan

    3. MRI scan

    4. Ultrasound to rule out deep abscess

    5. Bone biopsy

     

    Corrent answer: 3

     

    The clinical presentation is suspicious for a stress fracture of the tibia following free-fibula bone grafting. If plain radiographs are negative, more sensitive imaging such as a MRI or bone scan should be performed.

     

    Tibial stress fractures are a known complication following free-fibula bone grafting. Radiographs may be normal (as is the case in figure A), or might show the "dreaded black line" and/or new periosteal bone formation. If a stress fracture is confirmed with imaging, appropriate management would then consist of protective weight bearing until symptoms subside.

     

    Pacifico et al detail a case report of tibial stress fractures after vascularised free-fibula graft to the mandible. They report non-traumatic stress fracture to the tibia following a vascularised free-fibula graft is an uncommon but important complication.

     

    Ivey et al detail a case report of a tibial stress fracture after vascularised free-fibula graft for repair of non-union of the humerus.

     

    Emery et al report a case-series of 5 patients who sustained tibial stress fractures after a graft had been obtained from the ipsilateral fibula for use in anterior reconstruction of the spine. They theorize that the increased load the tibia bears as a result of the missing fibular graft may result in stress fractures.

     

    Illustration A shows new periosteal bone formation on the lateral cortex of the tibia consistent with a stress fracture.

     

    Incorrect Answer Choices:

    1: While compartment syndrome is on the differential diagnosis, his signs and symptoms are not most consistent with that diagnosis.

    2: While CT scan may show evidence of a stress fracture, MRI/bone scans have been shown to be superior methods for detection.

    4: As infectious laboratories are normal, an ultrasound to rule out a deep abscess would likely be negative.

    5: Bone biopsy is not appropriate without evidence of a lesion or concern for

    osteomyelitis.

     

     

     

     

     

     

    OrthoCash 2020

     

  75. A 65-year-old female with a history of developmental dysplasia of the hip (DDH) undergoes a total hip arthroplasty (THA) utlizing a posterior approach. Following THA, she notices an inability to dorsiflex the ankle of her operative extremity. Her pre-operative and postoperative radiographs are seen in figues A and B. Which of the following intra-operative techniques could have avoided this complication in this patient?

     

     

     

     

     

     

    1. Utilization of an anterior approach

    2. Modular components

    3. Use of a larger femoral head

    4. Femoral shortening osteotomy

    5. Acetabular osteotomy

     

    Corrent answer: 4

     

    Patients with DDH undergoing THA are at risk for post-operative sciatic nerve palsy due to intra-operative limb lengthening which increases tension on the sciatic nerve. Appropriate management after discovering a sciatic nerve palsy

    after surgery should include immediate knee flexion and hip extension to decrease tension on the sciatic nerve. Sciatic nerve palsy following THA most commonly only affects the common peroneal nerve branch, and spares the tibial nerve and can present as an inability to dorsiflex and evert the ankle.

     

    Farrell et al retrospectively looked at the risk factors for motor nerve palsy after THA. They found while motor nerve palsy is uncommon following primary THA, it can be a devastating complication. Some risk factors include: preoperative diagnosis of developmental dysplasia of the hip, posttraumatic arthritis, the use of a posterior approach, lengthening of the extremity, and use of an uncemented femoral implant. In their review, many of the motor nerve deficits did not fully resolve.

     

    Barrack et al reviewed neurovascular complications following THA. They stated that sciatic nerve injury is the most common nerve injury following THA utilizing a posterior approach. In comparison, femoral nerve injury is much less common and is usually from an anterior approach.

     

     

     

    OrthoCash 2020

     

  76. A cane held in the contralateral hand reduces joint reactive forces through the affected hip approximately 50% by which of the following mechanisms?

    1. Reducing hip abductor muscle pull

    2. Increasing hip flexor muscle pull

    3. Moving the center of rotation for the femoroacetabular joint

    4. Increasing joint congruence at the femoroacetabular joint

    5. Moving the center of gravity posterior to the second sacral vertebra Corrent answer: 1

    A cane held in the contralateral hand reduces joint reactive forces through the affected hip up to 50% by reducing abductor muscle pull.

     

    A cane create an additional force that keeps the pelvis level in the face of gravity's tendency to adduct the hip during unilateral stance. The cane's force must substitute for the hip abductors of the affected hip and creates a moment arm that is relatively long and originates on the side opposite the hip whose abductor muscles are weak. Additionally, the person needs adequate strength in the muscles of the wrist, elbow, shoulder girdle, and trunk.

     

    Brand and Crowninshield performed a 3-dimensional hip joint reactive force evaluation of 4 different groups of patients. The groups included normal

    subjects, preoperative THA subjects walking without a cane, preoperative THA subjects walking with a cane, and subjects following total hip reconstruction. Each of the 3 groups evaluated without the cane had statistically similar hip joint reactive forces. The preoperative THA subjects walking with a cane and significantly lower joint reactive forces (approximately 60%).

     

    The article by Blount was named by JBJS as a "Classics in JBJS" in 2003. It is a commentary encouraging the use of canes by describing how the biomechanics of the hip joint are altered while using a cane.

     

    Illustration A shows some of the mathematics behind cane use.

     

     

     

     

     

     

    OrthoCash 2020

     

  77. Which of the following is an example of an antalgic gait pattern not typically seen in clinical practice?

    1. Patient's knee is maintained in slight flexion throughout the stance period for ipsilateral knee arthritis

    2. Patient's contralateral step length is shortened with ipsilateral ankle arthritis

    3. Patient leans their trunk laterally over the painful leg during stance phase with ipsilateral hip arthritis

    4. Patient ambulates on their toes with an ipsilateral calcaneal stress fracture

    5. Patient ambulates predominately through the heel for ipsilateral knee arthritis

    Corrent answer: 5

     

    The term antalgic gait is non-specific and describes any gait abnormality resulting from pain. A patient with knee arthritis maintains slight flexion throughout the gait cycle. This compensatory knee flexion is exacerbated if the patient has a concomitant effusion in the knee as flexion reduces tension on

    the knee joint capsule. Gait compensation for knee arthritis also involves toe walking on the affected side, reducing the stride length, and reducing time of weight bearing on the painful leg.

     

    Gok et al performed a case-control gait analysis study of 13 patients with OA and 13 normal patients. They found that walking velocity, cadence and stride length were reduced in the OA group and that the overall stance phase was prolonged in the OA group. They concluded that computerized gait analysis can be used to reveal various mechanical abnormalities accompanying arthrosis of the knee joint at an early stage.

     

    Cole and Harner present Level 5 evidence about knee arthritis in the active patient. They stress that weightbearing radiographs are important in the diagnosis of arthritis. They also discuss the importance of looking for medial or lateral thrusts during gait and dynamic gait changes such as quadriceps avoidance or out-toeing.

     

    Incorrect Answers:

    Answer 1: Maintaining slight flexion is an example of quadriceps avoidance as keeping the knee flexed will decrease patellofemoral movement.

    Answer 2: Shortening the stride length allows less time on the painful extremity.

    Answer 3: Leaning laterally decreased the moment arm of body weight and reduces the joint reaction force on an arthritic hip.

    Answer 4: Toe walking is another example of both quad avoidance for knee arthritis or avoiding weight bearing through the ankle joint in ankle arthritis.

     

     

     

    OrthoCash 2020

     

  78. A 78-year-old male falls at home four months following a right total hip arthroplasty. Right leg deformity, pain, and inability to bear weight are present on physical exam. An injury radiograph is provided in Figure A, while radiographs taken immediately following the initial total hip arthroplasty are provided in Figures B and C. The patient denies any prodromal groin pain prior to his fall. Which of the following is the best treatment option?

     

     

     

     

     

     

     

     

    1. Traction for 6 weeks followed by slow return to weight bearing

    2. Open reduction and internal fixation

    3. Revision to a long, cementless femoral stem

    4. Revision to a long, cementless stem with strut allograft

    5. Revision to a long, cemented stem Corrent answer: 2

    The clinical presentation and radiograph are consistent with a Vancouver B1 periprosthetic femur fracture. The stem appears stable within the femur, and there is no evidence of subsidence with comparison to the initial post-THA radiographs. This fracture pattern is best treated with internal fixation.

    Illustrations A and B are radiographs of this patient following fixation. Illustrations C and D show bone healing at 2 years following the fracture.

     

    Duwelius et al report on 33 periprosthetic femur fractures. All fractures that demonstrated a stable stem at the time of surgery were treated with internal fixation, while those that were unstable were treated with a long, cementless revision femoral stem. At 2.5 years complications were minimal and the patients had regained their pre-fracture level of function.

     

    The review article by Kelley outlines the evaluation, classification, and treatment of periprosthetic femur fractures reinforcing the importance of stem stability within the femur. Periprosthetic fractures around a hemiarthroplasty should be treated with the same algorithm. However, if the patient had antecedent groin pain, then conversion to a total hip arthroplasty should be considered to prevent continued groin pain.

     

     

     

     

     

     

     

     

     

     

     

     

     

    OrthoCash 2020

     

  79. A 64-year-old male underwent the procedure shown in Figures A and B 7 weeks ago. He complains of difficulty with going down stairs. He reports no pain and denies constitutional symptoms. On examination the incision is well healed and no effusion is present. He is able to perform a straight leg raise with 5/5 strength. He lacks 2 degrees of terminal extension and has 80 degrees of active flexion. The knee is stable to varus and valgus stress testing at extension and mid flexion. His C-reactive protein and erythrocyte sedimentation rate are normal. What is the next most appropriate step in management?

     

     

     

     

     

     

    1. Manipulation under anesthesia

    2. Cortisone injection followed by physical therapy for quadriceps strengthening

    3. Aspiration to evaluate for septic arthritis

    4. Revise femoral component by downsizing A-P diameter

    5. Revise tibial component and add 5 degrees of posterior tibial slope Corrent answer: 1

    The history, physical examination, laboratory studies, and imaging are consistent with a total knee arthroplasty patient with arthrofibrosis. The next most appropriate option includes a manipulation under anesthesia to increase the patient's flexion.

     

    Maloney presents Level 4 evidence discussing TKA postoperative arthrofibrosis. They report that manipulation under anesthesia was successful in improving flexion from an average of 67 degrees premanipulation to 111 degrees

    postmanipulation.

     

    Keating et al report Level 4 evidence of 113 patients that underwent manipulation following TKA. They found that 90% of the patients achieved improvement of ultimate knee flexion following manipulation. The average improvement in flexion from the measurement made before manipulation to that recorded at the five-year follow-up was 35 degrees.

     

     

     

    OrthoCash 2020

     

  80. Which of the following total hip arthroplasty patients appropriately meets the criteria for a surgical debridement with isolated femoral head and polyethylene liner exchange?

    1. Prosthesis infection of 4 months duration

    2. Prosthesis infection 8 weeks following implantation

    3. Prosthesis infection 3 days following a systemic infection

    4. Acetabular component loosening due to osteolysis

    5. Vancouver Type A periprosthetic fracture.

     

    Corrent answer: 3

     

    Femoral head and polyethylene liner exchange is an appropriate treatment for the acutely infected arthroplasty. Acute infection has been defined as 3-6 weeks following surgery or following a systemic infection depending on the literature source. Subacute and chronic infections must be treated with a complete explant and exchange of all components. (One-stage or two-stage is controversial).

     

    Salvati et al review the management of total hip arthroplasty infection. Most importantly, the pathogen must be isolated to direct antibiotic treatment. The acuity of the infection must also be recognized to direct surgical management.

     

     

     

    OrthoCash 2020

     

  81. A 54-year-old woman is at physical therapy 3 months after a total knee arthroplasty when she feels a pop and develops increased pain in her knee. She continues therapy for another 3 months but reports weakness and frequent buckling. On exam, she has full passive extension but a 60 degree extensor lag. A lateral radiograph is shown in Figure A. What is the treatment of choice?

     

     

     

    1. Reconstruction with a bone-tendon allograft

    2. Repair augmented with hamstring autograft

    3. Continued therapy and strengthening

    4. Arthrodesis

    5. Treatment with orthotics for support Corrent answer: 1

    The patient has a chronic patellar tendon rupture following a TKA with marked extensor lag and patella alta on radiograph. A study by Barrack et al concluded that allograft reconstruction for the chronically-disrupted extensor mechanism after TKA could restore active extension and improve ambulatory function. In chronic cases, primary repair with or without local tissue augmentation have had disappointing results. Extensor mechanism injuries after TKA was reviewed by Parker et al. Patellar tendon ruptures are rare complications after TKA with an incidence reported <2.5%. Quadriceps tendon ruptures are even more rare with an incidence ~1%.

     

     

     

    OrthoCash 2020

     

  82. When compared to the standard medial parapatellar approach for revision total knee arthroplasties, the oblique rectus snip approach

    showed impairment in which of the following post-operative outcomes?

    1. range-of-motion

    2. patient satisfaction

    3. pain

    4. WOMAC function score

    5. no difference in outcomes Corrent answer: 5

    Meek et al compared the rectus snip to a standard medial parapatellar approach for revision total knee arthroplasty. The WOMAC function, pain, stiffness and satisfaction scores demonstrated no statistical difference. They concluded that use of a rectus snip as an extensile procedure had no adverse effect on outcome.

     

     

     

    OrthoCash 2020

     

  83. What is the range of pore size of cementless porous implants to allow for optimal bony ingrowth?

    1. Less than 1 micron

    2. 50 to 400 microns

    3. 1,000 to 5,000 microns

    4. 10,000 to 50,000 microns

    5. 100,000 to 500,000 microns

     

    Corrent answer: 2

     

    The range of 50 to 400 microns is the optimal pore size for cementless porous implants to allow for optimal bony ingrowth.

     

    Bobyn et al looked at the optimum pore size for fixation of porous surfaced metallic implants. Four different pore sizes were examined and placed in canine femurs for 4, 8, and 12 weeks and tested to measure the shear strength based on pore sizes. A pore size of 50 to 400 microns provided the maximum fixation strength in the shortest time period (8 weeks), implying maximal bony ingrowth.

     

    Pilliar et al discussed two independent canine studies which showed that initial implant movement relative to host bone can result in attachment by a nonmineralized fibrous connective tissue layer. They state that implant movement of greater than 150 microns leads to fibrous ingrowth.

    Jasty et al implanted porous-coated implants in the distal femoral metaphyses of twenty dogs and subjected them to zero, twenty, forty, or 150 micrometers of oscillatory motion. They found that that the implants that had been subjected to 150 micrometers of motion were surrounded by dense fibrous tissue.

     

     

     

    OrthoCash 2020

     

  84. An active 73-year-old male presents with progressive pain and instability 15 years after undergoing a left total knee arthroplasty. He denies any recent trauma. A comprehensive workup for infection is negative. What is the most appropriate management of this patient?

     

     

     

     

     

    1. Protected weight bearing for 6 weeks

    2. Revision total knee arthroplasty

    3. Bisphosphonate therapy

    4. Routine follow-up in 1 year

    5. Polyethylene liner exchange and bone grafting Corrent answer: 2

    This patient has evidence of periarticular osteolysis and component loosening around a previous total knee arthroplasty. He is symptomatic and would benefit from revision total knee arthroplasty (TKA).

     

    Osteolysis is one of the leading causes for late reoperation in patients who undergo TKA. Osteolysis occurs as the result of a foreign body response to particulate wear debris from the prosthetic joint. These particles consist of polyethylene, polymethylmethacrylate cement, and metal, all of which have been shown to elicit a distinct inflammatory response. Once the particles are generated from and around the implant, they become phagocytosed by macrophages and giant cells in the synovial or periprosthetic tissue. These cells, in turn, become activated and can directly or indirectly cause osteolysis. The femur is prone to osteolysis in the region of the femoral condyles and near the attachments of the collateral ligaments of the femur. Osteolysis around the tibia tends to occur along the periphery of the component or along the access channels to the cancellous bone.

     

    Maloney & Rosenberg reviewed the management and outcome of periprosthetic osteolysis around hip and knee implants. They recommended surgical intervention for periprosthetic osteolysis around a TKA with (1) first-time presentation of advanced osteolysis in the presence of an identifiable cause of wear particle production or in the presence of associated bone loss that places the structural integrity of the bone or fixation of the components at risk, (2) bearing surface wear in the presence of impending wear-through or related mechanical symptoms, (3) progressive osteolysis in an active individual, and (4) symptoms of wear debris-related synovitis that are refractory to conservative treatment.

     

    Griffin et al. evaluated the results of isolated polyethylene exchange for wear and/or osteolysis in 68 press-fit condylar TKAs from four centers. At a minimum of 24 months after polyethylene exchange surgery, there were 11 failures (16.2%).

     

    Gupta et al. discuss the etiology, diagnosis, contributing factors, and management of osteolysis as it relates to TKAs. They recommend that if the patient is asymptomatic with minimal osteolysis on plain radiographs, regular

    follow-up at 6 months to 1 year with medical management including calcium and bisphosphonates would be adequate. If the patient becomes symptomatic or the osteolysis is progressive, then early liner exchange with or without tibial baseplate exchange is considered.

     

    Figure A & B are AP and lateral radiographs of periarticular osteolysis and component loosening. Illustration A is an AP and lateral radiograph of the revision TKA.

     

    Incorrect Answers:

    Answer 1: Protecting this patient's weight bearing will not address the underlying cause of their pain, which is osteolysis and should be addressed with revision TKA.

    Answer 3: Bisphosphonate therapy would not be appropriate in this case due to the extensive osteolysis and component loosening present.

    Answer 4: Observation for 1 year is not advised because the amount of osteolysis is extensive.

    Answer 5: The patient is symptomatic (i.e., pain and instability) and has evidence of osteolysis and component loosening on x-ray. Liner exchange and bone grafting would not adequately address this degree of osteolysis as the components are loose and failure rate would be unacceptably high.

     

     

     

     

     

     

     

    OrthoCash 2020

     

  85. Which of the following statements is true regarding the thirty-year follow-up data obtained from the Charnley "low-friction" total hip arthroplasty?

    1. Acetabular component failure was the least common reason for revision surgery

    2. The number of revisions required for periprosthetic fractures was higher than that for deep infections

    3. Acetabular component failure was a more common reason for revision than deep infection

    4. Femoral component failure was a more common reason for revision than acetabular component failure

    5. Deep infection was the most common reason for revision Corrent answer: 3

    Failure of the acetabular component was the most common reason for revision at thirty-years for the Charnley "low-friction" total hip arthroplasty.

     

    The Charnley low-friction torque arthroplasty was introduced in 1962. It consisted of a 22mm diameter metal head, a cemented femoral component, and a cemented ultra-high-molecular-weight polyethylene acetabular component. Overall, the results were very good at thirty years with only 11.8% requiring revision.

     

    Charnley et al. in 1972 reported the 4-7 year results of 379 "low-friction" total hip arthroplasties. Overall, their short-term results were very good with only 2 loose acetabular components, 0 loose femoral components, and 1 late dislocation.

     

    Wroblewski et al. in 2009 reported the 30 year follow-up of 110 patients who underwent the "low-friction" total hip arthroplasty. 13 hips (11.8%) had to be revised. Of these, 5 were for problems with the acetabular component, 4 were for loosening of both components, 2 were for deep infection, 1 was from a loose femoral component, and 1 was from a fractured femoral component.

     

    Illustration A shows a radiograph after a Charnley low-friction total hip arthroplasty. Note the all poly-ethylene acetabular component. Illustration B shows the components used for the operation.

     

    Incorrect Answers:

    Answer 1: Acetabular component failure was the most common reason for revision.

    Answer 2: Revision for deep infection was more common than for fracture. Answer 4: Acetabular component failure was more common than femoral component failure.

    Answer 5: Acetabular component failure was the most common reason for revision.

     

     

     

     

     

    OrthoCash 2020

     

  86. A 71 year old gentleman underwent left total hip arthroplasty 10 years ago. Eighteen months ago he began having hip and thigh pain. Over the past 6 weeks, the pain has become excruciating and he has been unable to ambulate, even with the aid of a walker. He has mild pain with passive internal and external rotation of the hip. He is unable to ambulate in the office. Laboratory values are notable for a WBC of 10,300, CRP of 0.2, and ESR of 13. A radiograph is provided in figure A. Which of the following is the best treatment option?

     

     

     

    1. Radionuclide bone scan and MRI

    2. Open reduction internal fixation with a cable plate and allograft strut

    3. Revision arthroplasty with a fully coated cementless stem, cable wiring, and bone graft

    4. Revision arthroplasty with a modular, tapered stem and bone grafting of the diaphyseal fixation

    5. Revision arthroplasty with a total femur prosthesis Corrent answer: 3

    The radiograph is consistent with a periprosthetic femur fracture, with a loose femoral stem, and a Paprosky IIIA femoral defect. This is best treated with a fully-coated cementless stem with metaphyseal onlay allograft.

     

    Paprosky devised a classification for femoral bone loss following THA. The classification is as follows:

     

    Type I: minimal metaphyseal bone loss and intact diaphyseal fixation Type II: extensive metaphyseal bone loss with intact diaphyseal fixation

    Type IIIA: severe metaphyseal bone loss with greater than 4 cm of diaphyseal bone preservation for distal fixation.

    Type IIIB: severe metaphyseal bone loss and less than 4 cm of diaphyseal

    bone preservation for distal fixation

    Type IV: extensive metaphyseal and diaphyseal bone loss.

     

    Type IIIA may be treated with a fully coated stem. Type IIIB should consider a tapered, modular stem and/or bone grafting. Type IV likely needs a megaprosthesis. In this patient, given the preserved diaphyseal bone, revision arthroplasty with a fully coated femoral stem is the most appropriate treatment.

     

    The Sporer article reviews a case series of patients undergoing revision hip arthroplasty for femoral bone loss. Type IIIB defects with a femoral canal less than 19 mm may be treated with a fully porous-coated stem. However, patients with Type IIIB defect and a cavernous canal greater than 19 mm or a Type IV defect may need a modular tapered stem or a bone grafting procedure.

     

    The Paprosky article summarizes his classification of femoral bone loss in revision hip arthroplasty and provides an algorithm for treatment. Extensively porous-coated, diaphyseal filling femoral components showed excellent results in Paprosky IIIA defects.

     

    Radiograph A shows a total hip arthroplasty with severe metaphyseal bone loss and a supportive diaphysis.

     

    Incorrect Answers:

    Answer 1: No additional work-up is required prior to revision arthroplasty if laboratory results are negative for infection.

    Answer 2: Given the amount of bone loss and the loose femoral stem, fixation of the fracture/defect would not be advisable.

    Answers 4,5: These would be reasonable options if extensive bone loss was seen in the diaphysis.

     

     

     

    OrthoCash 2020

     

  87. A 74-year-old man presents with start-up thigh pain following a total hip replacement 10 years ago. Immediate post-operative radiograph is shown in Figure A. A current radiograph is shown in Figure B. Aspiration of the hip yields 1,005 white blood cells/ml. ESR is 12 (normal <40) and CRP is 0.4 (normal <1.2). Which of the following is the most appropriate management at this time?

     

     

     

     

     

    1. Revision of the femoral component to an uncemented, long, fully porous-coated stem

    2. Revision of the femoral component to a cemented stem

    3. Revision of the femoral component to an allograft prosthetic composite

    4. Revision of the femoral component to a proximal femoral replacement

    5. Removal of prosthesis with insertion of antibiotic spacer Corrent answer: 1

    The clinical presentation is consistent with symptomatic, aseptic femoral component loosening with no evidence of femoral bone defects. Appropriate management consists of revision of the femoral component to an uncemented, fully porous-coated stem.

     

    Aseptic loosening remains one of the most common indications for revision total hip arthroplasty. After infection has been ruled-out, management is determined by gauging the patients symptoms, the rate of progression of the subsidence, and the amount of femoral bone loss. Uncemented revision femoral components have shown superior results to cemented revision femoral components in the long-term. In the setting of Paprosky Type I, II, and IIIA defects of the femur, revision to an uncemented, fully porous-coated stem is advised.

     

    Moreland et al. review the results of 134 patients (137 hips) who underwent revision arthroplasty with an extensively porous-coated cobalt chrome femoral prosthesis. At a mean follow-up of 9.3 years, only 10 (7%) had to removed for any reason.

     

    Sporer et al. review the results of fully porous-coated stems, impaction bone grafting, and modular tapered stems for Paprosky III and IV femoral defects. They found a high rate of failure with fully porous-coated stems when used in patients with Type IIIB defects >19mm and Type IV defects. They attribute these failures to instability and the inability to eliminate micromotion.

     

    Figure A shows a cementless, metaphyseal engaging femoral component in good alignment. Figure B is a post-operative radiograph from 10 years later showing significant subsidence of the femoral component.

     

    Incorrect Answers:

    Answer 2: Uncemented femoral component revision stems have shown superior results to cemented femoral component revision stems.

    Answer 3-4: Both of these options would be reasonable if there were high-grade femoral bone loss (Paprosky IIIB, IV) in the setting of a loose stem. Answer 5: Aspiration and laboratory values are not consistent with infection.

     

     

    OrthoCash 2020

     

  88. A 72-year old female who underwent an uncemented right total hip arthroplasty 2 years ago complains of right hip pain after a fall. Figure A shows her current radiograph. Which acetabular bone defect classification and treatment option best describes this scenario?

     

     

     

     

     

    1. AAOS Type III - anti-protrusio cage with augmentation and a posterior column plate

    2. AAOS Type IV - anti-protrusio cage with screw fixation and a posterior column plate

    3. AAOS Type II - jumbo cup with augmentation and a posterior column plate

    4. AAOS Type I - total acetabular allograft with a cemented cup

    5. AAOS Type II - custom triflange acetabular component Corrent answer: 2

    Figure A shows pelvic discontinuity, which is consistent with a AAOS Type IV defect. Acetabular antiprotrusio cage with screw fixation and a posterior column plate is a reasonable treatment option for this condition.

     

    Acetabular bone loss following total hip arthroplasty is a challenging problem with a wide variety of treatment options available. The two most widely accepted classification systems are the AAOS and Paprosky classifications.

    AAOS type I defects are segmental, type II are cavitary, type III are combined cavitary and segmental, type IV is discontinuity, and type V is arthrodesis. All of the treatment options listed above are described for pelvic discontinuity,

    with none being described as superior.

     

    DeBoer et al. describe the results of 28 patients with pelvic discontinuity treated with a custom-made porous-coated triflange acetabular prosthesis. 20 of these patients were followed for 10 years. There were no re-operations, 5 hip dislocations, 1 sciatic nerve palsy, and an average improvement in the Harris hip score from 41 to 80.

     

    Paprosky et al. retrospectively reviewed patients who had an acetabular revision using a trabecular metal acetabular component for a pelvic discontinuity and compared these patients with a cohort of patients who had a previous reconstruction for a pelvic discontinuity using an acetabular cage.

    They found a decreased incidence of pain and need for walking aids in those patients who had revision with a trabecular metal acetabular component.

     

    Figure A shows pelvic discontinuity, likely acute given the lack of associated bony defects and recent fall. Illustration A details the AAOS hip acetabular defect classification and Illustration B is the often cited Paprosky classification.

     

    Incorrect Answers:

    Answer 1: Type III defects are combined cavitary and segmental. Answer 3: Type II defects are cavitary.

    Answer 4: Type I defects are segmental. Answer 5: Type II defects are cavitary.

     

     

     

     

     

     

     

     

    OrthoCash 2020

     

  89. Which of the following is indicative of type 1 collagen breakdown and can be utilized as a marker of bone turnover?

    1. Increased urinary N-telopeptide

    2. Increased urinary cAMP and phosphate

    3. Increased urinary phosphoethanolamine

    4. Increased urinary Bence Jones proteins

    5. Increased serum bone sialoprotein Corrent answer: 1

    Urinary N-telopeptide is a marker of increased bone turnover and is a breakdown product of Type 1 collagen.

     

    Increased serum alkaline phosphatase level and increased urinary markers of N-telopeptide, hydroxylproline, deoxypyridinoline indicate high bone turnover and can be seen in metabolic bone diseases such as Paget's disease.

     

    von Schewelov et al. reviewed 160 patients that underwent total hip replacements and examined their urine specimens to see if N-telopeptide levels correlated to periprosthetic osteolysis. They found that n-telopeptide levels were 1/3 higher in the patients that had evidence of osteolysis. N-

    telopeptide release and annual wear were both associated with increased prevalence of osteolysis in the study.

     

    Illustration A shows a radiograph of Pagets disease of the femur, an example of a condition where there is an increased level of N-telopeptide in the urine. Illustration B is a radiograph showing periprosthetic osteolysis, another condition where there is an increased level of N-telopeptide in the urine.

     

    Incorrect Answers:

    Answer 2: Increased urinary cAMP is found in Type 2 pseudohypoparathyroidism.

    Answer 3: Phosphoethanolamine is found in the urine in patients with hypophosphatasia.

    Answer 4: Bence Jones proteins are found in the urine of patients with multiple myeloma.

    Answer 5: Bone sialoprotein (BSP) is a component of mineralized tissues such as bone, dentin, cementum and calcified cartilage.

     

     

     

     

     

     

     

     

    OrthoCash 2020

     

  90. A 78-year-old female undergoes total hip arthroplasty through a minimally invasive surgical approach. During insertion of a metaphyseal fixation stem with a cementless press-fit technique, a crack in the calcar is identified. The stem is removed, two cable wires are passed around the calcar, and the same stem is reinserted. Which of the following statements is true?

    1. The patient should be advised she is at greater risk of stem subsidence and early revision

    2. Female sex is a risk factor for intraoperative calcar fracture

    3. A better outcome would be expected if a long-stem diaphyseal fixation stem had been inserted after recognition of the calcar fracture

    4. Cementless press-fit technique is not a risk factor for intraoperative fracture

    5. Minimally invasive surgical approach is not a risk factor for intraoperative fracture

    Corrent answer: 2

     

    Of the statements listed, the only true statement is that female gender is a risk factor for intraoperative calcar fracture.

     

    Calcar fractures are a documented complication of total hip arthroplasty. Studies have shown that successful outcomes can be achieved with stem removal, cable wiring of the calcar, and re-insertion of the primary stem.

     

    Berend et al. reviewed a series of 58 total hip arthroplasties who sustained an intraoperative calcar fracture. All were treated with cable wiring of the calcar and stem insertion. The authors report no femoral component subsidence or failure otherwise at 16 year follow-up.

     

    Graw et al. review a series of 46 revision THA's. Of the 46, fifteen underwent primary THA through a minimally invasive technique. The average length of time from primary THA to revision was 1.4 years for the minimally invasive group versus 14.7 years for the traditional exposure THA's. The authors conclude minimally invasive THA is a risk for early revision.

     

    Davidson et al. review intraoperative periprosthetic hip fractures. "Risk factors for intraoperative periprosthetic fractures include the use of minimally invasive techniques; the use of press-fit cementless stems; revision operations, especially when a long cementless stem is used or when a short stem with impaction allografting is used; female sex; metabolic bone disease; bone diseases leading to altered morphology such as Paget disease; and technical errors at the time of the operation." The authors summarize techniques for treatment and postulate that long term outcome is unaffected when the intraoperative fracture is identified and treated appropriately.

     

    Illustration A shows a nondisplaced calcar crack that was treated with a single Luque wire.

     

    Incorrect Answers:

    Answer 1. The patient is not at risk for further complications. See Berend reference.

    Answer 3. The patient is not at risk for further complications. See Berend reference.

    Answer 4. Intraoperative fractures occur more often with press-fit technique compared to cemented stems.

    Answer 5. Minimally invasive surgical approaches are a risk factor for intraoperative fracture during THA.

     

     

     

     

     

     

    OrthoCash 2020

     

  91. Which of the following types of prosthetic designs, seen in figures A-E, has been shown to have a high rate of loosening secondary to overconstraint?

     

     

     

     

     

     

     

     

     

     

     

     

     

    1. Figure A

    2. Figure B

    3. Figure C

    4. Figure D

    5. Figure E

     

    Corrent answer: 3

     

    Figure C shows an example of an Walldius hinge total knee prosthesis. This design had a higher rate of aseptic loosening (up to 20%) secondary to a high-degree of constraint.

     

    Constraint is defined as the effect of the elements of knee implant design that provides the stability needed to counteract forces about the knee after arthroplasty in the presence of a deficient soft-tissue envelope. While increasing component constraint increases the stability of the knee, it also transmits forces to the fixation and implant interfaces, which may lead to premature aseptic loosening. First-generation total knee hinged prostheses were highly constrained devices that only allowed a single axis of rotation.

     

    Lombardi et al. provide an Instructional Course Lecture on the different prosthetic designs in total knee arthroplasty. They argue that PCL sacrificing implants are more appropriate than cruciate-retaining implants in rheumatoid arthritis, previous patellectomy, previous high tibial osteotomy or distal femoral osteotomy, and in cases where the PCL is absent secondary to trauma.

    Morgan et al. discuss constraint in primary total knee arthroplasty. They argue that a hinge total knee arthroplasty should be reserved for severe instability, elderly patients with comminuted distal femur fractures, patients with

    extensor-mechanism disruption and unstable knees, and those with substantial bone loss not amenable to augmentation.

     

    Figure C shows an example of a Walldius hinged prosthesis.

     

    Illustration A shows an intra-operative example of a constrained-hinged knee prosthesis. Note the link between the tibial and femoral components, which differentiates it from a constrained, non-hinged prosthesis.

     

    Incorrect Answers: The following responses are incorrect as they all have lower rates of aseptic loosening than than varus/valgus constrained prostheses or hinged designs.

    Answer 1: Figure A shows a posterior-stabilized total knee arthroplasty. Answer 2: Figure B shows a patellofemoral arthroplasty.

    Answer 4: Figure D shows an uncemented total knee replacement. Answer 5: Figure E shows a uni-compartmental total knee replacement.

     

     

     

     

     

    OrthoCash 2020

     

  92. A 28-year-old football player sustains a contact knee injury while being tackled. On physical exam, he has a 1A Lachman, and a normal McMurray test. His posterior drawer, dial, and varus stress tests are normal. He has pain and 5mm opening on valgus stress at 30 degrees of flexion. Which statement is true regarding the injured structure?

    1. Resides between layers 1 and 2 on medial side of knee

    2. Inserts onto Gerdy's tubercle

    3. Originates slightly posterior and proximal to the medial epicondyle

    4. Courses intraarticularly thru hiatus of lateral meniscus

    5. Has an attachment between adductor tubercle and medial epicondyle at Schöttle's point

    Corrent answer: 3

     

    The clinical presentation is consistent with an injury to the superficial medial collateral ligament (MCL) of the knee, which originates slightly posterior and proximal to the medial epicondyle.

     

    The superficial portion of the MCL is the primary stabilizer to valgus stress at all angles, contributing 57% and 78% of medial stability at 5 degrees and 25 degrees of knee flexion, respectively. Anatomic studies have shown that the superficial MCL originates approximately 3.2 mm proximal and 4.8 mm posterior from the medial femoral epicondyle and inserts into the periosteum of the proximal tibia (deep to pes anserinus). The superficial MCL lies in layer 2, just deep to gracilis and semitendinosus tendons.

     

    Wijdicks et al. (2009) looked at radiographic identification of the primary medial knee structures including the superficial MCL. On the lateral radiograph, they found that the attachment of the superficial MCL was an average of 6.0 mm from the medial epicondyle.

     

    Wijdicks et al. (2010) reviewed injuries to the MCL and associated medial structures of the knee. They state that physical examination is the initial method of choice for the diagnosis of medial knee injuries through the application of a valgus load both at full knee extension and between 20 degrees and 30 degrees of knee flexion. Treatment of isolated grade-III injuries to the MCL, or such injuries combined with an anterior cruciate ligament tear, should start with nonoperative treatment of the MCL due to high rates of success with nonoperative treatment. If operative treatment is required, an anatomic repair or reconstruction is recommended.

    Illustration A shows the femoral and tibial attachments of the superficial MCL. Illustration B shows the osseous landmarks and attachments of medial knee structures (AT, adductor tubercle; GT, gastrocnemius tubercle; ME, medial epicondyle; AMT, adductor magnus tubercle; MGT, medial gastrocnemius tendon; sMCL, superficial MCL; MPFL, medial patellofemoral ligament; POL, posterior oblique ligament).

     

    Incorrect Answers:

    Answer 1: Superficial MCL resides in layer 2. Answer 2: Describes iliotibial band.

    Answer 4: Describes the popliteus.

    Answer 5: Describes the medial patellofemoral ligament.

     

     

     

     

     

     

     

    OrthoCash 2020

     

  93. Which of the following best describes normal tibio-femoral joint kinematics ?

    1. The femur undergoes internal rotation with knee flexion

    2. The lateral femoral condyle remains stationary on the lateral tibia plateau during knee flexion from 0 to 120 degrees

    3. The tibia undergoes internal rotation with knee flexion

    4. The medial femoral condyle moves posteriorly on the medial tibial plateau during knee flexion from 0 to 120 degrees

    5. Beyond 120 degrees of flexion only the lateral femoral condyle participates in femoral rollback

    Corrent answer: 3

     

    Tibia is subjected to internal rotation with knee flexion and the tibia EXternally rotates on femur as the knee EXtends.

     

    The axis of rotation shifts posterior on the lateral condyle with knee flexion. Flexion and extension at the knee occur about a constantly changing center of rotation (polycentric rotation).

     

    Freeman et al. conducted a biomechanical experiment and found that the medial femoral condyle does not move much from 0 to 120 degrees of flexion. They also found that the lateral femoral condyle and the contact area between that condyle and the tibia move posteriorly and tibial internal rotation occurs with knee flexion. They found that from 120 degrees to full flexion both condyles participate in "roll back".

     

    Illustration A shows why the screw-home mechanism occurs. The medial tibial plateau is longer than the lateral tibial plateau, leading to external rotation of the tibia during extension as the femoral condyle rotates about the tibia. Video V shows an example of external tibial rotation during extension.

     

    Incorrect Answers:

    Answer 1: Femur does not internally rotate with knee flexion.

    Answer 2: Laterally the femoral condyle and the contact area moves posterior on the tibia during knee flexion from 0 to 120 degrees.

    Answer 4: Medially the femoral condyle and the contact area remain relatively stationary during knee flexion from 0 to 120 degrees.

    Answer 5: Beyond 120 degrees of flexion both condyles participate in femoral rollback.

     

     

     

     

     

    OrthoCash 2020

     

  94. Which of the following molecules is associated with macrophage induced osteolysis surrounding orthopaedic implants?

    1. BMP-7

    2. IL-10

    3. SOX-9

    4. Osteoprotegrin

    5. IL-1

     

    Corrent answer: 5

     

    Of the options provided, IL-1 is most associated with macrophage induced osteolysis surrounding orthopaedic implants.

     

    Macrophages initiate the inflammatory cascade associated with aseptic loosening of orthopaedic implants by secreting platelet-derived growth factor (PDGF), prostaglandin E2 (PGE2), TNF-alpha, IL-1, and IL-6.

     

    Archibeck et al. state the primary cells involved in the process of periprosthetic loosening include the macrophage, osteoblast, fibroblast, and osteoclast. They report the chemical mediators that are responsible for the cellular interactions and effects on bone primarily include PGE2, TNF-alpha, IL-1, and IL-6.

     

    Drees et al. discuss the molecular pathway of aseptic loosening of orthopedic implants. They describe the following steps: 1) Wear debris particles released at the cement–bone interface attract macrophages, which, in turn, are stimulated to produce proinflammatory mediators and proteolytic enzymes; 2) RANKL, TNF-alpha, IL-1, IL-6, IL-17, and M-CSF mediate the differentiation of myeloid precursor cells into multinucleated osteoclasts, which release cathepsin K and acid and cause resorption lacunae; 3) Mesenchymal cells (prosthesis-loosening fibroblasts) present at the bone surface contribute actively to bone resorption.

    Illustration A shows the pathway described by Drees et al.

     

    Incorrect Answers

    Answer 1: BMP-2,4,6, and 7 all exhibit osteoinductive activity but BMP-3 does not.

    Answer 2: IL-10 inhibits osteoclast formation along with calcitonin

    Answer 3: SOX-9 is a key transcription factor involved in the differentiation of cells towards the cartilage lineage

    Answer 4: Osteoprotegrin binds to RANKL on the osteoblast, preventing RANK activation and inhibiting osteoclast activity.

     

     

     

     

     

     

    OrthoCash 2020

     

  95. Which of the following templates, seen in Figures A-E, will increase the offset while keeping the leg lengths the same?

     

     

     

     

     

     

     

     

     

     

     

     

     

    1. Figure A

    2. Figure B

    3. Figure C

    4. Figure D

    5. Figure E

     

    Corrent answer: 5

     

    If the total hip prosthesis is inserted according to the template in Figure E, the offset will be increased, while the leg lengths will remain unchanged.

     

    Restoration of limb length is essential following total hip arthroplasty. The amount of limb-length change will be the vertical distance between the center of rotation of the femoral component and the center of rotation of the acetabular component. Thus, when the femoral center of rotation on templating is inferior to that of the acetabular component, the limb will be shortened. Restoring femoral offset is also important. If the center of rotation of the prosthetic head lies lateral to that of the cup on templating, the reconstruction will produce decreased offset.

     

    Scheerlinck et al. present a stepwise approach to hip templating through four steps. Step 1 involves identifying landmarks, step 2 involves assessing the

    quality of the radiograph, step 3 identifies mechanical references, and step 4 optimizes implant positioning.

     

    Tripuraneni et al. discuss the common errors encountered when templating for total hip arthroplasty. They found the most common error was excessive limb lengthening and incomplete medialization of the acetabular component.

     

    Della Valle et al. review hip templating. They stress the importance of preoperative planning, but also discuss the importance of tactile feedback during the surgery in choosing the correct implants.

     

    Illustration A shows a table of how offset and leg lengths can be changed through positioning and selection of components.

     

    Incorrect Answers:

    Answer 1: Figure A will decrease leg length and decrease offset. Answer 2: Figure B will maintain offset but decrease leg length. Answer 3: Figure C will decrease offset and maintain leg lengths. Answer 4: Figure D will decrease leg length and increase offset.

     

     

     

     

     

     

     

    OrthoCash 2020

     

  96. A 91-year-old male with a history of chronic leukemia and dementia falls and sustains the hip fracture shown in Figure A. He undergoes a hemiarthroplasty through a posterior approach. A postoperative radiograph is shown in Figure B. Three weeks later he dislocates the hip arising from the toilet seat. A radiograph is shown in Figure C. The patient undergoes a closed reduction and is placed in a hip abduction brace. Post reduction radiograph is shown in Figure D. One month later he returns to clinic complaining of pain and inability to bear weight through the leg. A radiograph of the hip is included in Figure E. Which of the following factors has MOST likely contributed to the instability of the hip hemiarthroplasty?

     

     

     

     

     

     

     

     

     

     

     

     

     

    1. Femoral stem subsidence

    2. Increased offset

    3. Inadequate femoral stem neck length

    4. Patient's dementia status

    5. Patient's gender

     

    Corrent answer: 4

     

    The most likely contributing factor to the instability include the patient's dementia.

     

    Sultan et al use a basic science model to show liners with elevated rims placed in the posterior superior quadrant allow greater range of motion to dislocation than standard liners. They also show that 32 mm heads have greater range of motion to dislocation compared to 28 mm heads.

     

    Morrey et al reviewed a series of 19,680 primary THA's for late dislocation (first dislocation greater than 5 years after surgery). 165 hips (0.8%) had a late dislocation. Factors associated with late dislocation include implant malposition, neurologic decline, trauma, and polyethylene wear.

     

    Figure A shows a femoral neck fracture. Figures B and D show a hip hemiarthroplasty in appropriate position. Figure C and E show a dislocated hip hemiarthroplasty

     

    This patient's instability was managed by converting the hip hemiarthroplasty to a total hip arthroplasty with a constrained liner as shown in illustration A. No further instability episodes occurred following the revision.

     

    Incorrect Answers:

    Answer 1: There is no evidence of femoral neck subsidence on any of these radiographs.

    Answer 2: Increased offset would not lead to an increased risk of hip dislocation.

    Answer 3: Post-operative radiographs suggest that the native femoral neck length has been re-established adequately.

    Answer 5: Females have higher rates of dislocation than males.

     

     

     

     

     

     

    OrthoCash 2020

     

  97. What surgeon is credited for designing the prosthesis seen in Figure A?

     

     

     

    1. John Charnley

    2. San Baw

    3. Sir Harry Platt

    4. Austin T. Moore

    5. Charles Frederick Thackray Corrent answer: 4

    Figure A shows an example of an Austin-Moore hemiarthroplasty.

     

    Austin Moore developed the most popular long-stemmed prosthesis in the 1950s. The Austin-Moore prosthesis was a large, uncemented femoral stem that didn't use polyethylene. The Austin-Moore prosthesis had fenestrations for self-locking which later became the impetus for biological fixation. These implants were originally used to treat hip fractures and certain cases of degenerative arthritis. Later, in the 1960s, John Charnley introduced the idea of replacing the eroded acetabulum with a Teflon component.

     

    Moore et al. describe the first metallic hip replacement surgery in 1940. The patient had a proximal femoral resection for a giant cell tumor. The original prosthesis he designed was a proximal femoral replacement, with a large fixed head, made of the Cobalt-Chrome alloy Vitallium.

     

    Charnley et al. discuss the long-term results (up to 7 years) of the "low-friction" total hip arthroplasty. Infection rate was 3.8%, late mechanical failure was 1.3%, and most patients had excellent pain relief.

     

    Figure A shows a radiograph of an Austin-Moore hemiarthroplasty. Illustration A shows an Austin-Moore prosthesis. Illustration B shows an example of Charnley's "low-friction" total hip arthroplasty, with a stainless steel head and

    stem and a polyethylene acetabular component.

     

    Incorrect Answers:

    Answer 1: Charnley is credited with the invention of the "low-friction" total hip arthroplasty.

    Answer 2: Dr. San Baw pioneered the use of ivory hip prostheses to replace ununited fractures of the neck of femur.

    Answer 3: Sir Harry Platt was a mentor to John Charnley.

    Answer 5: Charles F. Thackray Limited (now a subsidiary of DePuy Orthopaedics) was instrumental in the growth of Dr. Charnley's implants.

     

     

     

     

     

     

    OrthoCash 2020

     

  98. A 45-year-old man has had the gait disturbance shown in Video A ever since a total hip replacement two years ago. Since then he has undergone physical therapy and nerve exploration without any clinical improvement. Extensive AFO bracing was attempted but was not tolerated by the patient. A recent ankle radiograph is shown in Figure

    A. The Silfverskiold test reveals dorsiflexion of 20 degrees with knee flexion, and 10 degrees with full knee extension. The results of muscle

    testing using a Cybex dynamometer are shown in Figure B. What is the most appropriate next step in in treatment.

     

     

     

     

     

     

     

    1. Ankle arthrodesis in 30 degrees of dorsiflexion

    2. Posterior tibial tendon transfer to the lateral cuneiform through the interosseous membrane

    3. Split anterior tibial tendon transfer to the cuboid

    4. Peroneus longus transfer to the navicular and gastrocnemius recession

    5. Flexor hallucis transfer to the navicular and tendo Achilles lengthening (TAL) Corrent answer: 2

    The clinical presentation is consistent with a sciatic neuropathy following THA in a patient that does not tolerate AFO bracing. Posterior tibialis tendon transfer is the next most appropriate step in treatment.

     

    Sciatic neuropathy, especially involving the common peroneal branch, is a known complication of total hip arthroplasty. Typically a patient is adequately treated with an AFO. In some clinical situations an AFO is not tolerated, and a tendon transfer is required. The posterior tibial tendon is the most commonly used donor muscle. A tendon transfer is feasible only if the tendon possesses at least 4/5 power. There is a loss of 1 MRC grade of strength following transfer.

    Rodriguez et al. retrospectively reviewed the results of the Bridle procedure 10 patients (11 feet) with a foot drop. The Bridle procedure consists of a posterior tibial tendon transfer through the interosseous membrane to the dorsum of the foot with a dual anastomosis to the tendon of the anterior tibial and a rerouted peroneus longus in front of the lateral malleolus. In their study all 11 feet were brace-free at final followup at 6.68 years.

     

    Yeap et al. retrospectively reviewed 12 patients who were treated with tibialis posterior tendon transfer for footdrop. They found good/excellent patient satisfaction in 10/12 patients. Additionally they found favorable variables for a good outcome include common peroneal nerve palsy over sciatic nerve palsy, male gender less than 30 years of age.

     

    Figure V is a Video that shows a right footdrop with high steppage gait. Figure A shows normal ankle radiographs. Figure B shows the results of dynamometer testing described above. Illustration A shows the Bridle procedure. The left panel shows how the tibialis posterior tendon (C) is tunneled through the interosseous membrane and through a slit in the tibialis anterior tendon (A) and inserted into the second cuneiform. The peroneus longus (B) is also transected and the distal stump is routed anterior the lateral malleolus and anastomosed to the tibialis anterior and tibialis posterior (at the slit where it passes through the tibialis anterior). The right panel shows retrieval of the tibialis posterior tendon above the ankle and passage through a window in the interosseous membrane.

     

    Incorrect Answers:

    Answer 1: There is no arthrosis of the ankle joint and several tendons possess sufficient strength to make a tendon transfer feasible. Tendon transfer should be attempted first.

    Answer 3: The anterior tibial tendon attaches to the plantar-medial aspect of the medial cuneiform and 1st metatarsal base. This muscle is weak (0/5 power) and transfer of its tendon muscle will not correct footdrop.

    Answer 4: The peroneus longus attaches to the medial cuneiform and 1st metatarsal (plantar-posterolateral aspect). This muscle is weak (2/5 power) and transfer of this tendon will not correct footdrop. Gastrocnemius recession will not increase the effectiveness of this transfer as there is no gastrocnemius contracture.

    Answer 5: The flexor hallucis longus is a secondary plantar flexor of the ankle. Its power is not mentioned in the question stem. But it is a less desirable tendon transfer compared with the posterior tibialis tendon. TAL will not increase its effectiveness. TAL is not necessary as there is dorsiflexion to 10degrees past neutral with the knee extended.

     

     

     

     

     

    OrthoCash 2020

     

  99. Which of the following variables is associated with elevated serum metal ion levels following metal-on-metal hip resurfacing arthroplasty?

    1. Smaller implant diameter

    2. Smaller acetabular cup abduction angle

    3. Higher postoperative functional scores

    4. Severe preoperative osteoarthritis

    5. Anteversion of acetabular cup between 10 and 20 degrees Corrent answer: 1

    Smaller femoral head diameter is associated with elevated serum metal ion levels with metal-on-metal hip resurfacing arthroplasty.

     

    Metal-on-metal (MOM) hip resurfacing arthroplasty has the advantage of better wear properties (lower linear wear rate and volume of particles) than metal on polyethylene. However, elevated serum metal ion levels is one of the negatives which has received much attention recently. Studies have found smaller implant diameter and acetabular cup abduction angle >55 degrees are associated with elevated serum metal ion levels. Cup abduction angles of greater than 55 degrees lead to a more vertical cup and edge loading.

    Desy et al. found that smaller implant diameter, larger cup inclination, and lower postoperative functional scores are associated with increased cobalt and chromium levels after metal-on-metal hip resurfacing. They found that severity of preoperative osteoarthritis, acetabular version, femoral stem-shaft and valgus angle, and anterior orientation of the femoral component had no effect on the circulating metal ion levels.

     

    DeHaan et al. obtained serum ion levels in 214 MOM resurfacing patients at least 1 year following surgery. They found that cup abduction angles greater than 55 degrees combined with smaller component sizes led to edge loading and elevated ion levels.

     

    Illustration A shows how a metal-on-metal prosthesis design allows you to have a larger femoral head as opposed to a metal on polyethylene design (example in THA). Illustrations B and C show a photo of a metal on metal resurfacing implant and radiographs of the implant.

     

    Incorrect Answers:

    2: Elevated cup abduction angle leads to elevated serum metal ions.

    3: Higher postoperative functional scores have not been shown to increase serum metal ions.

    4: Severity of pre-operative arthritis has not been associated with increased serum metal ions.

    5: Anteversion of the acetabular cup between 10 and 20 degrees has not been shown to increase serum metal ions.

     

     

     

     

     

     

     

     

     

     

    OrthoCash 2020

     

  100. Which of the following intra-operative steps would put a patient at risk for lateral patellar maltracking during total knee arthroplasty (TKA)?

    1. External rotation of the femoral component

    2. Medial placement of the patellar component

    3. Internal rotation of the tibial component

    4. Lateral translation of the femoral component

    5. Superior placement of the patellar component Corrent answer: 3

    Internal rotation of the tibial component increases the Q angle and causes an increased risk of lateral patellar maltracking.

     

    During TKA, useful techniques that help prevent patellar maltracking include: external rotation of the femoral and tibial components, lateral translation of the femoral component, and medial placement of the patellar component.

     

    In an instructional course lecture, McPherson looked at patellar tracking in primary TKA. He reviews the concept of patellofemoral maltracking, the importance of the Q angle, mechanical alignment, femoral component rotation, tibial component positioning, patellar component positioning, patellar height, and patellar resurfacing as factors related to patellofemoral tracking.

    Bengs et al. studied the effect of patellar thickness on intra-operative knee flexion and patellar tracking during PCL retaining TKAs. Using 2mm increments (2-8 mm), passive knee flexion was recorded and gross mechanics of patellofemoral tracking were assessed. On average, passive knee flexion decreased 3 degrees for every 2-mm increment of patellar thickness, there was no gross effect on patellar subluxation or tilt.

     

    Illustration A shows how internal rotation of the tibial component would increase the Q angle, and thus be more likely to have lateral patellar maltracking.

     

    Incorrect answers:

    Answer 1,2,4,5- Would all prevent lateral patellar maltracking.

     

     

     

     

     

     

    OrthoCash 2020

     

  101. A patient undergoes a primary total hip arthroplasty with a highly cross-linked ultra-high molecular weight (UHMW) polyethylene acetabular liner. In comparison to a 28mm femoral head, a 32mm femoral head will provide which of the following?

    1. Increased risk of dislocation

    2. Decreased range of motion

    3. Decreased risk of osteolysis

    4. Equivalent wear rate of the polyethylene acetabular liner

    5. Increased risk of periprosthetic fracture Corrent answer: 4

    Wear rates of highly cross-linked UHMW polyethylene liners are independent of femoral head size between 22 and 46 mm in diameter.

     

    While the wear rates of old polyethylene liners increased with increasing femoral head size, wear rates of the new highly cross-linked UHMW polyethylene liners have shown to be independent of head size. This is extremely advantageous, as increasing the femoral head size improves range of motion and increases jump distance, thereby decreasing dislocation rates.

     

    Geller et al. report a prospective series of 42 patients that had a total hip arthroplasty with a highly cross-linked UHMW polyethylene liner and a femoral head >32 mm in diameter. After three years, there were no cases of osteolysis or failure due to aseptic loosening.

     

    Muratoglu et al. studied the wear rates of several polyethylene liners with varying femoral head sizes. In the highly cross-linked UHMW polyethylene group, wear rates were independent of femoral head size.

     

    Illustration A shows how increasing femoral head size increases the jump distance required for dislocation.

     

    Incorrect Answers:

    Answer 1: Increasing femoral head size decreases the risk of dislocation. Answer 2: Increasing femoral head size increases range of motion.

    Answer 3: Increasing femoral head size has not been shown to affect rates of osteolysis.

    Answer 5: Increasing femoral head size has not been shown to increase the rate of periprosthetic fracture.

     

     

     

     

     

    OrthoCash 2020

     

  102. A 56-year-old male undergoes an uncomplicated revision total knee arthroplasty. Post-operatively, he is noted to have a foot drop that has persisted despite conservative management including bracing and physical therapy. At two months, the patient undergoes external neurolysis with no improvement in function. At 18 months follow-up, he demonstrates passive ankle dorsiflexion 10 degrees past neutral, complete absence of active dorsiflexion, and 5/5 inversion strength. Which of the following is the most appropriate treatment at this time?

    1. Continue Ankle-foot orthosis (AFO) and physical therapy

    2. Repeat neurolysis with possible nerve repair

    3. Peroneus tertius transfer

    4. Peroneus tertius transfer with achilles tendon lengthening

    5. Posterior tibial tendon transfer to dorsum of foot Corrent answer: 5

    A peroneal nerve palsy (with intact posterior tibial tendon strength) that has failed conservative management is best treated with a posterior tibial tendon transfer to the dorsum of the foot.

     

    Peroneal nerve palsy following total knee arthroplasty or knee dislocation is a potentially devastating complication that may lead to lack of active dorsiflexion and a compensatory steppage gait pattern. Initial management consists of an ankle-foot orthosis (AFO) and physical therapy to maintain passive ankle dorsiflexion. If nerve function fails to return during the course of conservative management and the patient demonstrates intact posterior tibialis muscle strength, posterior tibial tendon transfer to the dorsum of the foot has been shown to improve functional outcomes and eliminate the need for continued bracing. The most common procedure for posterior tibial tendon transfer involves transferring the tendon through the interosseous membrane and inserting the tendon onto the lateral cuneiform.

     

    Prahinski et al. review the results of 10 patients at 61 months' follow-up who underwent the Bridle transfer (posterior tibialis transfer through interosseous membrane and peroneus longus to front of lateral malleolus) for peroneal nerve palsies. They conclude the Bridle procedure is adequate for return to function in low-demand individuals, but may fail over time in those who return to vigorous physical activity.

     

    Rodriguez et al. review the results of 10 patients who underwent the Bridle procedure for peroneal nerve palsy in an attempt to balance their foot and

    provide dorsiflexion. All of their patients were brace free at an average followup of 6.8 years.

     

    Video V shows the clinical results 10 weeks after transfer of the tibialis posterior tendon for a drop foot.

     

    Incorrect Answers:

    Answer 1: An AFO and physical therapy is appropriate for initial management while awaiting potential nerve recovery.

    Answer 2: Repeat neurolysis is unlikely to achieve clinical improvement after initial failure.

    Answer 3: The peroneus tertius is also located within the anterior compartment and is likely to be involved in her nerve injury pattern. Answer 4: The peroneus tertius is also located within the anterior compartment and is likely to be involved in her nerve injury pattern.

     

     

     

    OrthoCash 2020

     

  103. An 82-year-old male sustains a ground level fall and sustains the injury shown in Figure A. Which of the following treatment methods is most appropriate for treating this injury?

     

     

     

    1. Closed reduction and functional bracing

    2. Open reduction and fixation with a plate with screws and cerclage cables

    3. Open reduction and fixation with a cortical allograft strut and cerclage cables

    4. Revision hip arthroplasty with bridging of the fracture with a plate with screws and cerclage cables

    5. Total femoral replacement Corrent answer: 2

    This fracture pattern is typically referred to as an interprosthetic fracture; this is increasing in incidence due to increasing numbers of patients with ipsilateral hip and knee arthroplasty.

     

    The first reference by Ricci et al reviewed 50 Vancouver B1 fractures treated with a lateral plate without allograft. They reported 100% union rate at a mean of 12 weeks and only one deep infection. Nearly 75% of patients were able to return to their baseline ambulatory status.

     

    The second reference by Ricci et al reviewed 59 patients with periprosthetic femur fractures (THA or TKA) treated with ORIF without bone grafting. They report 58/59 patients healed after the index procedure and 49/59 were able to

    return to their baseline functional level.

     

    The reference by Fulkerson et al reported on 24 patients who underwent LISS plate fixation of periprosthetic femur fractures around well-fixed THA or TKA. They reported union in 21/24 at a mean of 6.2 months, with only one failure of fixation. They note that percutaneous fixation is effective although technically demanding.

     

    Figure A shows an interprosthetic femur fracture between well-fixed hip and knee arthroplasties.

     

    Incorrect Answers:

    Answer 1: Nonoperative management is not indicated for this fracture pattern in this patient.

    Answer 3: Use of a cortical allograft strut without plate support is not indicated.

    Answer 4: Revision of the femoral stem is not indicated in this case because the stem appears completely stable on the provided radiograph.

    Answer 5: Total femoral replacement is not indicated as a primary procedure for this injury pattern.

     

     

     

    OrthoCash 2020

     

  104. A 62-year-old female undergoes an uncomplicated primary total knee replacement. Her knee range-of-motion pre-operatively was 0-135 degrees of flexion. Which of the following is true regarding the immediate post-operative use of a continuous passive motion machine in this patient?

    1. Reduced risk of venous thromboembolism

    2. No long-term difference in ROM compared to patients not using CPM

    3. Increased passive knee flexion at 6 months

    4. Increased length of hospitalization

    5. Decreased risk of surgical site infection Corrent answer: 2

    The use of a continuous passive motion (CPM) machine following primary total knee arthroplasty has not shown any long-term benefits with regards to

    range-of-motion.

     

    The concept of CPM was created by Dr. Robert Salter in 1970 and is currently being used in select patients following total knee replacement, ACL

    reconstruction, and a variety of other procedures about the knee. In theory, the CPM allows for movement of synovial fluid to allow for better diffusion of nutrients into damaged cartilage. Additionally, it has been thought to prevent fibrous scar tissue formation about the joint. While some studies have shown increased early active knee flexion at two weeks, these results were not significant at later follow-up. Controversy exists as to whether these small benefits offset the patient inconvenience and expense of the CPM.

     

    Lotke et al. expolre the effects of tourniquets and CPM machines in 121 patients undergoing total knee arthroplasty. They found that immediate CPM combined with intraoperative release of the tourniquet increased blood loss. The patients with the least amount of blood loss had the tourniquet released after a compressive dressing was applied and in whom CPM was delayed for a few days.

     

    Bourne et al. perform a meta-analysis on the effectiveness of CPM following total knee arthroplasty. They found the CPM plus physical therapy increased active knee flexion more than physical therapy alone 2 weeks after surgery with a decreased length of hospitalization. The benefits of increased active knee flexion were not maintained after 2 weeks.

    Illustration A shows an example of a CPM machine. Incorrect Answers:

    Answer 1: A current meta-analysis has shown there is not enough evidence

    from available RCTs to conclude that CPM reduces the risk of venous thromboembolism following total knee arthroplasty.

    Answer 3: There are no difference in passive range of motion at any time points following total knee arthroplasty.

    Answer 4: Some studies have shown decreased length of hospitalization in those patients that use a CPM.

    Answer 5: CPM has not shown to have any effect on rates of surgical site infections.

     

     

     

     

     

    OrthoCash 2020

     

  105. A 67-year-old female complains of anterior groin pain one year following a primary, uncemented total hip arthroplasty. The pain is exacerbated when she tries to climb stairs or get up from a seated position. She denies any recent fevers or chills. On physical exam, the pain is reproduced with resisted seated hip flexion. Laboratory analysis, including WBC, ESR, and CRP are within normal limits. Radiographs reveal that the components are appropriately positioned without evidence of loosening or fracture. Which of the following is the most appropriate at this time?

    1. Revision of the acetabular component

    2. Image-guided diagnostic injection of lidocaine into the iliopsoas tendon sheath

    3. Hip aspiration

    4. Bone scan

    5. Conservative management including activity modifications, NSAIDs, and physical therapy

    Corrent answer: 2

     

    The patients history and physical exam are most consistent with iliopsoas impingement. This diagnosis is most reliably confirmed with a diagnostic/therapeutic injection of steroid or lidocaine into the iliopsoas tendon sheath.

     

    Iliopsoas tendinitis following total hip arthroplasty is an uncommon but treatable cause of anterior groin pain following total hip arthroplasty. The true incidence is unknown, but some studies suggest it is the cause of a painful

    total hip arthroplasty in up to 4.3% of cases. Potential causes include a malpositioned acetabular component, excessively long screws, limb length discrepancy, or retained cement. Diagnosis is confirmed by injecting the iliopsoas tendon sheath. Most cases are refractory to conservative management and often require surgical intervention. In the case of a malpositioned acetabular component, revision to a more agreeable position is advisable. In the absence of a defined etiology, iliopsoas tendon release offers adequate pain relief and return to function in a majority of patients.

     

    Lachiewicz et al. review anterior iliopsoas impingement after total hip arthroplasty. They state that most patients with iliopsoas impingement often require surgical treatment, with options including iliopsoas tendon release or resection, removal of protruding cement or screws, and acetabular revision.

     

    O' Sullivan et al. review 16 cases of iliopsoas impingement following primary total hip arthroplasty. Only 1 of the cases was secondary to a malpositioned acetabular component, with the other 15 cases being attributed to altered anatomy of the iliopsoas tendon as a result of the surgery. These 15 patients underwent iliopsoas tendon release, and all had improvement in pain and function following surgery.

     

    Nunley et al. review 27 patients with a presumed diagnosis of iliopsoas impingement following total hip arthroplasty who were treated with fluoroscopically guided injections of the iliopsoas bursa. The average modified Harris hip score in the patients who underwent injection improved, however, 30% required an additional injection and 22% underwent surgical release for continued pain.

    Illustration A shows a flouroscopic injection into the iliopsoas tendon sheath. Incorrect Answers:

    Answer 1: Radiographs reveal well positioned components. In addition,

    revision of the acetabular component without a confirmed diagnosis is not advisable.

    Answer 3: Infectious laboratories are negative, and the patient denies constitutional symptoms.

    Answer 4: Bone scan is unlikely to provide any additional information as her presentation is more consistent with iliopsoas impingement rather than aseptic loosening.

    Answer 5: Conservative management could be entertained after confirming the diagnosis of iliopsoas impingement.

     

     

     

     

     

    OrthoCash 2020

     

  106. A 72-year-old female underwent an uncomplicated primary total hip replacement 18 years ago. Current radiographs reveal the abnormality shown in Figure A. Which of the following cell types (Figures B-F) is implicated in the process shown by the arrow?

     

     

     

     

     

     

     

     

     

     

     

     

     

    1. Figure B

    2. Figure C

    3. Figure D

    4. Figure E

    5. Figure F

     

    Corrent answer: 5

     

    Figure F shows an example of a macrophage, which is a key mediator in the osteolytic process shown in Figure A.

     

    Osteolysis is the end result of a biologic process that begins when the number of wear particles following a joint replacement overwhelms the body's capacity to clear them from circulation. The residual particles are phagocytosed by macrophages, which then release an array of cytokines and other inflammatory mediators that recruit osteoclasts to resorb bone.

     

    Gupta et al. review osteolysis following total knee arthroplasty, including etiology, diagnosis, and management. Amongst other things, they highlight the importance of design changes to minimize osteolysis including highly cross-linked polyethylene and alternative bearing materials.

     

    Ren et al. performed a study where they implanted a hollow titanium rod into the distal femur and pumped polyethylene particles into the femoral bone marrow cavity. They found that macrophage migration occurs at a systemic (rather than local) level, and that the recruitment of macrophages led to localized osteolysis.

     

    Holt et al. review the biology behind aseptic osteolysis. Specifically, they highlight the importance of the RANK-RANKL-OPG pathway as the final

    common pathway to osteoclastogenesis, and the possibility of eliminating osteolysis by blocking this pathway. AMG-162 is a human immunoglobulin monoclonal antibody with a high affinity for RANKL, and studies are currently being undertaken to determine its safety and efficacy.

     

    Figure F shows an example of a macrophage, which may be identified by its irregular shape and phagocytic inclusions. Illustration A shows the pathway by which marcrophages induce osteolysis following a joint replacement surgery.

     

    Incorrect Answers:

    Answer 1: Figure B shows is an eosinophil, which may be seen in Eosinophilic Granulomatosis. It has a bilobed nucleus and granules that stain pink when eosin is used in the staining process.

    Answer 2: Figure C shows a lymphocyte, with a round nucleus and a narrow rim of cytoplasm. Lymphocytes are seen in ALVAL, an adverse reaction to metal-on-metal hip prostheses.

    Answer 3: Figure D shows a basophil with a multi-lobed nucleus and blue-stained granules.

    Answer 4: Figure E shows the characteristic appearance of an erythrocyte.

     

     

     

     

     

     

    OrthoCash 2020

     

  107. A 45-year-old with a history of sickle cell anemia reports hip pain for the past 6 months. A radiograph of the affected hip is shown in Figure A. Which of the following interventions has been shown to have the best outcomes in this patient population?

     

     

     

    1. Observation

    2. Bisphosphonates

    3. Hemi-arthroplasty

    4. Uncemented metal on polyethylene total hip arthroplasty

    5. Cemented metal on polyethylene total hip arthroplasty Corrent answer: 4

    Based on the radiographs and current literature, the best intervention is an uncemented metal on polyethylene total hip arthroplasty.

     

    Avascular necrosis of the hip may be idiopathic in nature or associated with alcoholism, steroid use, or as in this case, sickle cell anemia. The Ficat staging system is used to classify avascular necrosis of the hip. Changes in treatment are driven by development of symptoms as well as the development of subchondral bone collapse (Ficat Stage 3). In those with with femoral head flattening (Ficat Stage 4) and acetabular degenerative changes (Ficat Stage 5), total hip replacement has good to excellent outcomes.

     

    Mont et al. review surgical options for avascular necrosis of the hip. Head preserving procedures are generally reserved for those patients where the femoral head has not collapsed. Collapse and associated arthritis warrant utilization of arthroplasty procedures.

     

    Mont et al. conducted a systematic review to better delineate the symptomatic progression of asymptomatic avascular necrosis of the hip. They found that patients with sickle cell disease have the highest rate of progression to

    collapse. Medium sized, laterally located lesions were associated with a higher frequency of collapse and joint preserving procedures are recommended for these.

     

    Figure A shows radiograph of a patient with avascular necrosis; note the femoral head flattening, narrowing of the joint space and acetabular sclerosis.

     

    Incorrect Answers:

    Answer 1: Conservative measures in this patient would not improve this patient’s outcome give the degree of the femoral head collapse and presence of acetabular degeneration.

    Answer 2: Bisphosphonates can be used in patients with avascular necrosis of the hip prior to collapse. Current data is conflicting as to whether they prevent collapse or not.

    Answer 3: Outcomes for patients undergoing hemiarthroplasty for avascular necrosis of the hip in the young patient are poor; and as a result, this has been largely abandoned.

    Answer 5: Higher failure rates have been seen in patients undergoing cemented total hip arthroplasty in treatment of avascular necrosis of the hip.

     

     

     

    OrthoCash 2020

     

  108. The function of which of the following structures is to resist internal tibial rotation with the knee in full extension?

    1. Anterior cruciate ligament

    2. Iliotibial band

    3. Popliteus tendon

    4. Popliteofibular ligament

    5. Posterior oblique ligament Corrent answer: 5

    The primary function of the posterior oblique ligament is to resist internal tibial rotation with the knee in full extension.

     

    The posterior oblique ligament is a structure within the posteromedial corner of the knee, with attachments proximally to the adductor tubercle of the femur and distally to the tibia/posterior knee capsule. The posterior oblique ligament and posteromedial capsule play a significant role in the prevention of additional posterior tibial translation in the knee in the setting of posterior cruciate ligament injury. They also act to resist internal tibial rotation with the knee in full extension.

    Griffith et al. reports that the posterior oblique ligament provides significant resistance to valgus and internal rotation forces with knee extension. They used a cadaver model and demonstrated that the superficial MCL resists valgus and external rotation forces more than the posterior oblique ligament, while the posterior oblique ligament is more involved in resisting internal rotation.

     

    Tibor et al. reviews the anatomy of the posteromedial corner of the knee. They report that failing to recognize injury to these structures may cause failure of cruciate ligament reconstruction surgery, and that reconstruction or repair of the posteromedial corner may be indicated in the face of multiple ligament injuries.

     

    Illustration A shows the posteromedial corner of the knee, including the posterior oblique ligament.

     

    Incorrect answers:

    1-4: These structures are not primary restraints to internal tibial rotation in full extension.

     

     

     

     

     

     

    OrthoCash 2020

  109. Increasing the porosity of a cement spacer for an infected total knee arthroplasty leads to which of the following?

    1. Increased strength

    2. Increased elution of antibiotics

    3. Increased cement density

    4. Improved cement-prosthesis bonding

    5. Increased reinfection rate Corrent answer: 2

    Elution of an antibiotic is increased with increased porosity of a cement spacer. This porosity increase can be obtained with hand mixing and avoiding the use of a vacuum-type mixing device.

     

    Joseph et al. reviews antibiotic-impregnated cement in hip arthroplasty. They note that use of this cement in one- or two-stage revisions has lowered reinfection rates, with the spacers acting to reduce dead space while stabilizing the joint.

     

    Cui et al. reviews antibiotic impregnated cement for TKA and THA. They report that use of greater than 2 grams of antibiotic per 40 gram unit of cement weakens the cement and that use of two antibiotics in conjunction may potentially increase elution.

     

    The reference by Stevens et al compared Simplex and Palacos bone cement in regards to elution in a TKA mold model. They found that initial as well as weekly (9 weeks total) elution rates were greater in the Palacos spacers than the Simplex models. They recommend use of the Palacos cement in TKA model to target antimicrobial delivery while limiting the potential for systemic antibiotic-related toxicity.

     

    Illustrations A and B show an antibiotic spacer in a two-stage revision TKA. Illustration C shows a PROSTALAC in a two-stage revision THA.

     

    Incorrect Answers:

    Answer 1: Increasing the porosity acts to weaken the biomechanic characteristics of the cement.

    Answer 3: Increasing the porosity acts to decrease the cement density. Answer 4: Increasing the porosity does not improve cement-prosthesis bonding.

    Answer 5: Increasing the porosity has not been shown to alter reinfection rate.

     

     

     

     

     

     

     

     

     

     

    OrthoCash 2020

     

  110. A 65-year-old patient was treated with an open reduction/internal fixation for a left femoral neck fracture sustained 25 years ago. Five years ago he developed hip pain and was converted to a left hip hemiarthroplasty. He presents with complaints of groin pain for the past 6 weeks. A recent radiograph is shown in Figure A. The patient’s physical exam is limited secondary to pain. Serum laboratory values are WBC-8.0, ESR-20, CRP-0.5. A synovial fluid aspirate of the hip demonstrates < 500 cells (60% PMN). What is the most likely cause of this patient's symptoms?

     

     

     

     

     

    1. Acetabular protrusio

    2. Infected hip hemiarthroplasty

    3. Lumbar radiculopathy

    4. Impingement of the hip hemiarthroplasty

    5. Iliopsoas tendinitis

     

    Corrent answer: 1

     

    Based on the history, radiographs, and laboratory values, the patient has developed failure of his hip hemiarthroplasty. At this point in time he warrants a conversion to a total hip arthroplasty.

     

    Avascular necrosis (AVN) of the femoral head after traumatic injury to the femoral neck occurs at an incidence of 10-45%. Although the risk increases with failure to anatomically reduce the fractue, it can still occur in non displaced settings. Treatment of avascular necrosis in older patients includes hip hemiarthroplasty or a total hip replacement. With the former, development of acetabular protrusio can contribute to groin symptoms. Functional outcomes have been reported to be higher in those receiving total hip replacement for AVN of the femoral head.

     

    Lee et al. prospectively compared the use of bipolar hip hemiarthroplasty versus total hip arthroplasty for advanced stages of AVN of the femoral head (Ficat Stage 3). Total hip scores were most improved in the total hip arthroplasty group. Migration of the outer head in the hemiarthroplasty group was seen in 23% of patients. They recommend use of a total hip arthroplasty in patients with Ficat Stage 3 AVN of the femoral head

     

    Ito et al. evaluated the outcomes of patients who underwent bipolar hemiarthroplasties for femoral head avascular necrosis. They found that proximal migration and acetabular degeneration were risk factors for groin symptoms. They also found that outcomes were inferior to patients who had undergone total hip arthroplasty for AVN of the femoral head. They recommend use of total hip arthroplasty in advanced osteonecrosis of the femoral head

     

    Diwanji et al. evaluated outcomes of patients who underwent a conversion from a bipolar hip arthroplasty to total hip arthroplasty in 25 patients. Thirteen (52%) patients were revised to THA because of acetabular erosions. Follow up was completed for an average of 7.2 years. At final follow-up, they found improvement of the Harris Hip Scores and improvement of the pain portion of the WOMAC index. They recommend use of total hip replacement as an option to salvage failed bipolar hip hemiarthroplasty

     

    Figure A shows the radiograph of a hip hemiarthroplasty where acetabular protrusion has developed.

    Incorrect Answers

    Answer 2: There is no evidence of infection based on laboratory results. Answer 3: There is no evidence of lumbar based pathology in this patient. Answer 4: While impingement could be a cause of pain, it is not as likely given the history, clinical findings and radiographs seen here.

    Answer 5: While irritation of the iliopsoas could occur, it is not as likely given the radiograph seen here.

     

     

     

    OrthoCash 2020

     

  111. A 38-year-old female patient presents to your office three years after a hip resurfacing. She complains of worsening left hip discomfort for the last 6 months. Her ESR is 12 (normal 0-20) and CRP is 1.2 (0-5). A radiograph and axial and coronal MRI scans are shown in Figures A, B, and C. What is the most likely diagnosis?

     

     

     

     

     

     

     

     

    1. Infection

    2. Type I Hypersensitivity reaction

    3. Femoral neck fracture

    4. Prosthesis Loosening

    5. Pseudotumor

     

    Corrent answer: 5

     

    The clinical presentation is consistent with a young woman who has developed a symptomatic pseudotumor following hip resurfacing. Her hip discomfort is related to a mass that has developed around the left hip.

     

    Pseudotumors, also referred to as Aseptic Lymphocyte-Dominated Vascular-Associated Lesions (ALVAL), are sterile inflammatory lesions that most commonly occur from metal-on-metal articulations. They occur at an incidence of 0-39% with metal-on-metal resurfacing hip components. The exact mechanishm of formation is unclear, however excessive wear is considered the initiating process, leading to the release of microscopic metal particles. These are cytotoxic to macrophages once phagozytised, leading to necrosis within the lesions and the development of semi-solid or fluid-filled masses around the implant. Lymphocytes are thought to be responsible for the tissue reaction.

    Patients often do not complain of pain, but present with a mass around the hip that causes discomfort.

    Hart et al. performed a case-control study comparing patients with well-functioning metal-on-metal hip resurfacing to those who have painful prostheses. They found no significant difference between the painfree and painful groups with MRI diagnosed pseudtumors (61% vs. 57%). They concluded that the presence of a pseudotumor should not automatically necessitate revision surgery.

     

    Daniel et al. reviewed the current concepts surrounding pseudotumor. Risk factors associated with pseudotumor formation and failure are female gender, age under 40, hip dysplasia, metal hypersensitivity, and small components.

    Larger components have been found to decrease the risk of failure.

     

    Figure A is an AP pelvis radiograph of a patient following a left hip resurfacing surgery. Figures B and C are axial and coronal MR images demonstrating a large pseudotumor around the left hip resurfacing. Illustrations A and B identify the large pseudotumor as outlined in red.

     

    Incorrect Answers:

    Answer 1: While infection should always be ruled out with symptomatic prosthetic joints, the radiograph and MRI clearly represent a pseudotumor from a metal-on-metal hip resurfacing.

    Answer 2: Hypersensitivity reaction would be a rare presentation 2.5 years following a hip resurfacing.

    Answer 3: Femoral neck fracture is a risk in the initial post-operative period (<20 weeks).

    Answer 4: The radiograph shows a hip resurfacing device in good position with no evidence of loosening.

     

     

     

     

     

     

     

     

    OrthoCash 2020

     

  112. After total hip arthroplasty (THA) for osteoarthritis a patient is unable to dorsiflex her ankle or extend her great toe. She is treated conservatively with an orthosis and after 3 months on physical therapy she ambulates with a "slapping gait." What is the most appropriate next treatment option?

    1. MRI of her spine

    2. Ankle Fusion

    3. Continue Ankle-Foot Orthosis

    4. Revision total hip arthroplasty

    5. Sural nerve grafting Corrent answer: 3

    The patient has suffered from a peroneal nerve injury most likely from errant retractor placement during the hip replacement resulting in a foot drop. The most appropriate next treatment is an ankle-foot orthosis.

     

    The ankle joint of an ankle-foot orthoses (AFOs) should restrict plantarflexion to prevent foot drop during the swing phase. In a patient who can not actively dorsiflex the foot the AFO keeps the foot in a neutral position during gait allowing for uninterrupted swing during ambulation.

     

    Park et al reviewed common peroneal nerve injury after THA. Only one-half of the patients in the study who developed common peroneal nerve palsy following total hip arthroplasty recovered fully. The mean time to recovery was approximately one year for partial peroneal palsy and one and one-half years for complete palsy. Obesity adversely influenced the nerve recovery. Thus, at 3 months, the nerve should continue to be monitored and the use of an AFO would assist in ambulation.

     

    Yokoyama et al. developed an AFO with an oil damper to adjust the plantarflexion resistive moment as excessive plantarflexion resistance will cause excessive knee flexion during the stance phase. They found the AFO with the oil damper achieved sufficient plantarflexion of the ankle and mild flexion of the knee by adjusting a proper plantarflexion resistive moment during initial stance phase, and provided a more comfortable gait than did the traditional AFOs.

     

    Illustration A shows the location of the sciatic nerve relative to the short external rotators when performing a posterior approach to the hip.

     

    Incorrect Answers:

    Answer 1: Immediate foot drop following a total hip replacement is likely related to the procedure, not the spine.

    Answer 2: Although an ankle fusion would eliminate the need for dorsiflexion while ambulating, it is more invasive than an AFO and does not consider the potential for nerve recovery over time.

    Answer 4: Revising the hip would increase the risk of peroneal palsy.

    Answer 5: It is too early to consider nerve grafting.

     

     

     

     

     

     

    OrthoCash 2020

     

  113. Which of the following statements is true about racial disparities in total joint arthroplasty?

    1. The rate of surgical intervention for African American males is lower than white or Hispanic males

    2. The rate of surgical intervention for Hispanics is higher than that for whites

    3. The rate of surgical intervention for white males is lower than for African American males

    4. There is no difference in the rate of surgical intervention between whites, Hispanics, or African Americans

    5. The rate of surgical intervention is equal for Hispanic and white males Corrent answer: 1

    The rate of surgical intervention for African American males is lower than either white or Hispanic males.

     

    Numerous studies have shown clear racial disparities in the utilization of total joint arthroplasty for the treatment of osteoarthritis. African American and Hispanic patients undergo total joint arthroplasty at a rate much lower than

    white patients, even in areas where insurance coverage is more equitable. Currently, little is known about the reasons for such disparities.

     

    Skinner et al. reviewed the Medicare claims between 1998 through 2000 to determine any racial or ethnic disparities amongst patients undergoing total knee arthroplasty. Amongst other things, they showed that the arthroplasty rates for black men were consistently lower than white men in nearly every region.

     

    Nelson reviews health disparities in orthopaedic surgery. Amongst other things, they discuss how African American patients and white patients perceive the same pain and functional limitations for similar radiographic disease. Thus, ethnic differences in perception of symptoms cannot explan the racial disparities noted in total joint arthroplasty.

     

    Incorrect Answers:

    Answer 2: The rate of joint arthroplasty for whites is higher than Hispanics. Answer 3: The rate of joint arthroplasty for whites is higher than African Americans.

    Answer 4: There are significant differences in the utilization of joint arthroplasty where comparing different races.

    Answer 5: White males, on average, undergo total joint arthroplasty at a rate higher than Hispanic males.

     

     

     

    OrthoCash 2020

     

  114. A 65-year-old man presents with aseptic loosening 3 years after total knee arthroplasty. The surgeon reviews radiographs of his knee and takes him to the operating room for revision total knee arthroplasty. During surgery, the exposure technique shown in Figure A is used. Which of the following radiographs (Figures B-F) has the greatest likelihood of needing this exposure technique?

     

     

     

     

     

     

     

     

     

     

     

     

     

     

     

     

    1. Figure B

    2. Figure C

    3. Figure D

    4. Figure E

    5. Figure F

     

    Corrent answer: 3

     

    Figure A shows a tibial tubercle osteotomy (TTO). Patella baja (Figure D) is an indication for a TTO.

     

    In revision total knee arthroplasty (TKA), surgical exposure should be extensile. Different exposure techniques have been described (see below). Patella baja may indicate that there is patellar tendon contracture. In this instance, a TTO can be used to prevent inadvertent patellar tendon avulsion which is difficult to repair and may lead to loss of function. Further, proximal transfer of the osteotomized tibial tubercle may be used to correct patella baja, bearing in mind that excessive superior translation alters the mechanics of the knee by making the quadriceps less efficient.

     

    Younger et al. reviewed surgical approaches in revision TKA. They include quadriceps snip, patellar turndown, TTO, femoral peel, medial epicondylar osteotomy and quadriceps myocutaneous approach.

     

    Mendes et al. reviewed the results of TTO in revision TKA. They advocate TTO for cases where the patellar cannot be retracted laterally with knee in 90deg of flexion. Complications include nonunion, tubercle fragment fracture and displacement, and tibial metaphyseal fracture (at the level of the distal cut of the osteotomy).

     

    Della Valle et al. reviewed surgical approaches for revision TKA. They advocate TTO because repair is stronger than patellar turndown, there is less tension on the tibial tubercle in flexion than on the quadriceps tendon, and where multiple operations are required (as multiple VY approaches lead to excessive scar, making the approach difficult) or where stemmed tibial components need to be removed.

     

    Illustration A shows tibial tubercle osteotomy hinged on a lateral periosteal flap. Illustration B shows quadriceps snip. Illustration C shows patellar turndown. Illustration D shows medial epicondyle osteotomy.

     

    Incorrect Answers

    Answer 1: Figure B shows posterior dislocation. This is not an indication for a TTO. The knee can be approached through a standard medial parapatellar arthrotomy.

    Answer 2: Figure C shows patellar tendon avulsion (with patella alta). There is no need for a TTO as patellar subluxation (or even eversion) is easy in this

    situation.

    Answer 4: Figure E shows rotating platform polyethylene spinout. This is not an indication for a TTO.

    Answer 5: Figure F shows knee recurvatum in a cruciate-retaining implant because of attenuation of the PCL and posterior capsule. This is not an indication for a TTO.

     

     

     

     

     

     

     

     

     

     

     

     

     

     

     

    OrthoCash 2020

     

  115. Which of the following is the most common intraoperative complication in a patient with sickle cell disease undergoing a total hip arthroplasty?

    1. Periprosthetic fracture distal to the implant

    2. Iatrogenic fracture causing pelvic discontinuity

    3. Perforation of the femoral canal

    4. Cardiac arrest from fat embolization to lungs

    5. Injury to the sciatic nerve Corrent answer: 3

    Perforation of the femoral canal during preparation of the femur is not an uncommon complication, with rates ranging from 4.9-18.2%.

     

    While total hip arthroplasty is extremely effective for pain relief in patients with osteonecrosis of the hip secondary to sickle cell disease, the procedure carries a higher rate of complications compared with non-sickle cell disease patients. Particular attention should be given to the preparation of the femur as femoral medullary widening from chronic marrow hyperplasia adjacent to patchy areas of dense sclerosis can make preparation of the canal difficult.

    Some surgeons prefer to ream over a guide-wire to avoid perforation.

     

    Jeong et al. reviewed total hip arthroplasty in patients with sickle cell disease. Amongst other things, they discuss the difficulties associated with preparation of the femoral canal, quoting a perforation rate between 4.9-18.2%. They also state there are no prospective studies comparing cementless to cemented THA, but retrospective data has shown promising results with cementless components.

     

    Hernigou et al. retrospectively reviewed 244 patients with sickle cell disease that underwent cemented total hip arthroplasty. They had a 3% infection rate, a relatively low rate of revision for aseptic loosening, and a 27% rate of medical complications. Overall, they viewed their results as favorable.

     

    Illustration A shows a patient with bilateral AVN secondary to sickle cell disease. Note the areas of patchy dense sclerosis in the metaphyseal region of the proximal femur.

     

    Incorrect Answers:

    Answer 1: Periprosthetic fracture usually occurs at the area of perforation, not distal to the implant.

    Answer 2: Acetabular fractures are more common in this patient population as well, but the rate of iatrogenic pelvic discontinuity is lower than that of femoral perforation.

    Answer 4: The rate of cardiac arrest from fat embolization to the lungs is quite low.

    Answer 5: While injury to the sciatic nerve is possibly, it has not been shown

    to be more common in this patient population. The rate of post-operative hematoma causing sciatic nerve dysfunction may be higher in this patient population.

     

     

     

     

     

     

     

    OrthoCash 2020

     

  116. A 63-year-old patient presents with periprosthetic joint infection 3 years after primary total knee arthroplasty. A radiograph of her knee is seen in Figure A. She undergoes 2-stage revision total knee arthroplasty. Radiographs taken at the time of explantation are seen in Figure B. An articulating antibiotic spacer is placed. Two months later, she is deemed to be free of infection and is taken to the operating room for the second stage operation. Intraoperatively, it is noted that the collaterals are intact and the previous tibial tubercle osteotomy had healed. What is the most appropriate surgical strategy at this point?

     

     

     

     

     

     

    1. Address epiphyseal defects with impaction particulate bone grafting

    2. Address metaphyseal defects with structural allograft and uncemented, unstemmed implants

    3. Address metaphyseal defects with uncemented, porous metaphyseal

      sleeves and uncemented, stemmed implants

    4. Address diaphyseal defects with porous metal cones and uncemented, stemmed implants

    5. Address diaphyseal defects with cemented stemmed implants Corrent answer: 3

    This patient has massive metaphyseal defects following resection of primary TKA implants. Metaphyseal defects may be addressed with uncemented, porous metaphyseal sleeves and uncemented stemmed implants.

     

    In revision settings, metaphyseal bone is often deficient. The Anderson Orthopaedic Research Institute classification (AORI) is most commonly used to classify defects. Stemmed implants are necessary to divert stress away from deficient metaphyseal defects to structurally sound cortical bone. These may be cemented or uncemented.

     

    Haidukewych et al. reviewed metaphyseal fixation in revision TKA. For large defects, they advocate structural allograft, porous metal cones, and stepped metaphyseal sleeves.

     

    Bush et al. reviewed managing bone loss in TKA. They cautioned that joint line elevation, distal femoral bone loss, and femoral prosthesis downsizing leads to flexion instability. They advocate cement filling for Type I defects, modular augments for Type 2, impaction grafting for Type 1 or 3, structural allograft for Types 2 and 3, metaphyseal filling or megaprosthesis for Type 3, including porous metal implants.

     

    Figure A shows an infected primary TKA with a stemmed tibial component with medial augments. This suggests that the revision implant will require at least a stemmed, augmented component. Figure B shows massive metaphyseal defects (AORI Type 2) at the time of explantation. Illustration A comprises postop images of osseointegrated metaphyseal sleeves and stemmed implants. Illustration B depicts the AORI classification (see Review Topic for detailed description). Images courtesy of Haidukewych et al (Ref 1).

     

    Incorrect Answers:

    Answer 1: While there indeed is an epiphyseal defect, it is the metaphyseal defect that needs to be addressed for implant stability. Impaction bone grafting may be used for contained Type 1 defects.

    Answer 2: Structural allografts are an option. Because there will be no ingrowth at the allograft-implant interface, cement is necessary. Stems are necessary to bypass large metaphyseal defects and transfer load to diaphyseal cortical bone.

    Answer 4: Porous metal cones can be used to address metaphyseal defects. They are used together with stemmed components, which are cemented inside the cones. The cones, in turn, are press-fit into the metaphysis.

    Answer 5: The defects addressed during TKA revision are predominantly metaphyseal. Stemmed implants may be cemented or uncemented.

     

     

     

     

     

     

     

     

    OrthoCash 2020

     

  117. During templating for a total hip arthroplasty, placing the femoral head center of rotation directly superior to the center of rotation of the acetabular component will have which of the following effects?

    1. Increase offset

    2. Decrease limb length

    3. Decrease offset

    4. Increase limb length

    5. No change in length or offset Corrent answer: 4

    Placing the femoral head center of rotation directly superior (above) the acetabular center of rotation will lengthen the limb without changing offset.

     

    When templating the femoral component for a total hip arthroplasty, it is imperative to restore limb length and offset. To change limb length, the femoral component center of rotation (COR) can be adjusted in a superior or inferior direction. If the femoral component COR is superior to the acetabular component COR, the limb will be lengthened (as in the example above).

    Conversely, if the femoral component COR is inferior to the acetabular component COR, the hip will be shortened. A change in offset will be determined by the medial/lateral relationship between the acetabular and femoral components. In the example above, the COR of the femoral component is directly above the COR of the acetabular component. In this situation, there is no change in offset.

     

    Merle et al. performed a retrospective cohort study to identify differences in femoral offset as measured on an AP pelvis radiograph, AP hip radiograph, and a CT scan. They found that femoral offset is significantly underestimated on AP radiographs of the pelvis. In contrast, AP radiographs of the hip are much more accurate in representing true offset.

     

    Della Valle et al. review the importance of preoperative planning prior to total hip arthroplasty. While they mention that templating can be very accurate, determination of stem and cup size should also be determined by tactile feedback during broaching and reaming.

     

    Illustration A shows the femoral head COR inferior to the acetabular COR. This will result in a decreased limb length.

    Incorrect Answers:

    Answer 1: To increase offset, the femoral head COR should be placed medial to the acetabular COR.

    Answer 2: To decrease limb length, the femoral head COR should be placed inferior to the acetabular COR (see Illustration A).

    Answer 3: To decrease offset, the femoral head COR should be placed lateral to the acetabular COR.

    Answer 5: In order to keep limb length and offset the same, the acetabular and femoral offsets must overlap.

     

     

     

     

     

     

    OrthoCash 2020

     

  118. Which of the following intra-operative errors most commonly leads to patellar maltracking during a total knee arthroplasty?

    1. Using the gap balancing technique instead of measured resection technique

    2. Internal rotation of the femoral component

    3. External rotation of the tibial component

    4. Lateralization of the femoral prosthesis

    5. Overresection of the patella

    Corrent answer: 2

     

    Internal rotation of the femoral component increases the Q-angle and will increase the likelihood of patellar maltracking.

     

    Patellar maltracking is one of the most common complications following a total knee arthroplasty. Any alteration that results in increased lateral retinaculum tension or an increased Q-angle may lead to patellofemoral instability.

    Common causes include internal rotation of the femoral or tibial components, medialization of the femoral component, and placement of the patellar prosthesis on the lateral border of the patella. If a patient presents with postoperative maltracking and component rotation is thought to be the cause, a CT scan is the diagnostic study of choice.

     

    Rhoads et al. analyze 7 cadaveric specimens to define the kinematics of the intact knee and to evaluate the effects of prosthetic replacement on those kinematics. Amongst other things, they showed that lateralization of the femoral component improved patellar tracking and prevented dislocation.

     

    Malo et al. review patellar maltracking following a total knee replacement. They discuss the importance of externally rotating the femoral component on the femur relative to the posterior articular condyles to establish a rectangular and balanced flexion gap and to accommodate central patellar tracking.

     

    Illustration A shows how an internally rotated femoral component displaces the patella medially. The blue line is a straight line upwards from the tibial tubercle, and the green line represents a line from the tibial tubercle to the center of the patella. The difference between the blue and green lines in the internally rotated prosthesis is the amount the patella has displaced medially. If you deviate the patella medially, this increases the Q-angle and could lead to patellar maltracking in a total knee replacement.

     

    Incorrect Answers:

    Answer 1: This has not shown to impact the incidence of patellar maltracking. Answer 3: External rotation of the tibial component decreases the Q-angle.

    Answer 4: Lateralization of the femoral prosthesis decreases the Q-angle. Answer 5: Underresection of the patella, not overresection, overstuffs the patellofemoral joint and tightens the lateral retinaculum, which may lead to maltracking.

     

     

     

     

     

    OrthoCash 2020

     

  119. When performing a total knee arthroplasty on a 60-year-old female patient, a surgeon chooses not to resurface the patella. Instead, he performs a patelloplasty by excising the marginal osteophytes and reshaping the patella. All of the following statements comparing the results of patelloplasty to patella resurfacing are true EXCEPT:

    1. There is no difference in relative risk of anterior knee pain.

    2. There is no difference in relative risk for revision surgery involving the tibial and femoral components.

    3. There is an increased risk that she will need secondary resurfacing.

    4. No difference in rates of patellar avascular necrosis or patellar tendon injury.

    5. Total knee arthroplasty improved function regardless of whether the patella was resurfaced.

      Corrent answer: 1

       

      In TKA with an unresurfaced patella, there is an increased risk of anterior knee pain and secondary resurfacing.

       

      Surgeons can choose to resurface or not resurface all patellae, or selectively resurface patellae. In unresurfaced patellae, they may perform a patelloplasty (excise marginal osteophytes and reshape the patella). Unresurfaced patellae have increased risk of anterior knee pain requiring secondary resurfacing.

      Indications for resurfacing include inflammatory arthritis, patella maltracking, patellofemoral osteoarthritis as the main indication for TKA.

      Meneghini et al. reviewed the literature on patellar resurfacing. Prospective, randomized studies show conflicting results with regards to satisfaction rates between groups. Meta-analyses show increased risk of re-operation and anterior knee pain in the unresurfaced group.

       

      Parvizi et al. performed meta-analysis on 1519 knees. They found there was

      (1) lower relative risk of re-operation (resurfaced group), (2) lower relative risk of anterior knee pain (resurfaced group), (3) increased rate of secondary resurfacing (unresurfaced group), (4) no difference in patient satisfaction, (5) TKA improved function regardless of whether the patella was resurfaced, (6) no difference in complications.

       

      Incorrect Answers:

      Answers 2,3: There is no difference in the rate of revision surgery involving the tibial and femoral components. But in the unresurfaced group, there is an increased likelihood that secondary patellar resurfacing will be required (8.7% incidence).

      Answer 4: Meta-analysis studies show that there is no difference in the rate of patellar fractures, avascular necrosis and patellar tendon injury. In more recent publications, there is a decreased risk of complications related to the extensor mechanism for both groups because surgeons are more aware of possible complications and because surgical techniques have improved.

      Answer 5: This statement is true.

       

       

       

      OrthoCash 2020

       

  120. A 55-year-old patient returns for followup 2 years after a left ceramic-on-ceramic total hip arthroplasty. He has no pain or symptoms of instability. The video in Figure V shows him ascending stairs. All of the following factors may contribute to this phenomenon EXCEPT

    1. Impingement

    2. Edge-loading

    3. Loss of fluid film lubrication.

    4. Third-body particles

    5. Subclinical infection

     

    Corrent answer: 5

     

    The clinical presentation is consistent for prosthesis squeaking following a THA. Squeaking is multifactorial and may include impingement, edge-loading, loss of fluid film lubrication, and third-body particles. Subclinical infection does not play a role in squeaking.

    Squeaking is defined as a high-pitched, audible sound occurring during movement of the hip. In ceramic-on-ceramic (COC) hips, the incidence is 0.5-10%. The incidence of revision because of squeaking is 0.5%. Squeaking is less common in metal-on-metal bearing surfaces (4-5%).

     

    Chevilotte et al. reviewed COC bearing surfaces. They found that without lubrication, squeaking occurred with normal gait, high load, stripe wear, material transfer, edge wear and microfractures. In contrast, with lubrication, squeaking only occurred with material transfer.

     

    Finkbone et al. reviewed COC total hip arthroplasty in patients <20-years-old after 52-month (average) follow-up. They found that survival rate was 96% with 1 revision for a loose acetabular component (failure of bone ingrowth). They found no cases of osteolysis, which can theoretically occur because of stripe wear, or femoral neck-on-liner impingement.

     

    Walter et al. reviewed squeaking hips. They found that this phenomenon is associated with (1) walking, bending, rising (cyclical gait movements or extreme flexion), (2) maloriented acetabular components, (3) impingement,

    (4) third-body particles, (5) edge loading, (6) loss of fluid film lubrication and

    (7) thin, flexible (titanium) stem.

     

    The video shows squeaking of a left total hip arthroplasty during hip extension from a flexed position. This is consistent with edge loading. Illustration A shows superior edge loading in the walking position. The arrow represents the loading force. The superolateral edge of the liner is in contact with the superior surface of the head. The dark area (area of edge loading) is anteverted with respect to a line of latitude (dark line) on the head. Illustration B shows posterior edge loading in the bending position. The arrow represents the loading force. The posterior edge of the liner is in contact with the superior surface of the head. The shaded area represents the area of edge loading and the dark stripe indicates wear on the femoral head.

     

    Incorrect Answers:

    Answers 1,2,3,4: Impingement, edge-loading, loss of fluid film lubrication and third-body particles contribute to squeaking.

     

     

     

     

     

     

     

     

    OrthoCash 2020

     

  121. Figure A shows the image of a 72-year-old male who sustained a fall from standing. Past medical history is significant for hypertension. He was a community ambulator without the use of a cane or walker prior to the fall. During the operation, he is noted to have a well-fixed acetabular component without significant wear of his polyethylene liner, but his femoral component is easily extractable. Which of the following correctly pairs his Vancouver classification and appropriate surgical intervention?

     

     

     

    1. Vancouver A, Revision of femoral component to cemented stem with fixation of the fracture

    2. Vancouver B1, Revision of femoral component to cemented stem with fixation of the fracture

    3. Vanvouver B1, Revision of femoral component to a long, porous-coated, cementless stem with fixation of the fracture

    4. Vancouver B2, Fixation of the fracture with a plate and cerclage wires

    5. Vancouver B2, Revision of femoral component to a long, porous-coated, cementless stem with fixation of the fracture

    Corrent answer: 5

     

    Figure A shows a Vancouver B fracture around the femoral prosthesis. Because the prosthesis is noted to be loose during the operation, it is classified as a Vancouver B2 fracture. The most appropriate operation would be revision of the femoral component to a long, porous-coated, cementless stem in addition to fixation of fracture with a plate and cerclage wires.

     

    According to the Vancouver classification, a type B2 fracture occurs around or just distal to a loose femoral stem with adequate proximal bone. Revision of the femoral component is necessary, with uncemented stems showing superior clinical results to cemented stems in most studies. The revision prosthesis should bypass the distal fracture by 2 cortical widths.

     

    Corten et al. reviewed thirty-one patients with Vancouver B2 fractures that

    were treated with a long cemented stem with additional allograft or plate fixation. At 46 months, none of the implants had to be revised, but it should be noted that 43% of the patients died within the first year.

     

    Mulay et al. reviewed 24 patients with Vancouver B2 and B3 fractures managed with a cementless, tapered, fluted, and distally fixed stem. 91% of fractures united uneventfully. Complications included dislocations (5), nonunions (2), and infection (1).

     

    Springer et al. review 116 patients with Vanvouver B fractures treated with revision of the femoral component. The uncemented, extensively porous-coated implants had the highest likelihood of stable fixation and were not associated with any nonunions.

     

    Illustration A reviews the Vancouver classification for periprosthetic femur fractures. Illustration B shows a post-operative radiograph following a Vancouver B2 fracture. In this case, a trochanteric plate with cerclage wires was used to fix the fracture. A long-stemmed, porous-coated, cementless femoral prosthesis was used for the revision.

     

    Incorrect Answers:

    Answer 1: Vancouver A fractures involve the trochanteric region. Answer 2: Vancouver B1 fractures have a well-fixed femoral prosthesis. Answer 3: Vancouver B1 fractures have a well-fixed femoral prosthesis.

    Answer 4: Because the femoral prosthesis was loose, it needs to be revised.

     

     

     

     

     

     

     

     

    OrthoCash 2020

     

  122. A 56-year-old male undergoes revision of his right hip arthroplasty for acute pain and radiographs suggestive of ceramic femoral head fracture. At the time of the revision, multiple fragments of the ceramic femoral head were seen in the joint and soft tissues. The components were noted to be in good position. He was copiously irrigated and the ceramic head was exchanged with a metallic femoral head. 12 months later, the patient presents with insidious onset right groin pain. Radiographs show no gross abnormalities without signs of loosening. Which of the following is the most likely cause of the patient's pain?

     

    1. Periprosthetic infection

    2. Massive third body wear

    3. Pseudotumor formation

    4. Soft tissue metallosis

    5. Iliopsoas tendonitis

    Corrent answer: 2

     

    The most likely cause of the patient's pain is massive third body wear caused by retained ceramic fragments.

     

    Cermamic femoral head fractures create many fragments that are difficult to extract at the time of revision surgery. During the revision surgery, it is imperative to remove all fragments that can be visualized. Despite a thorough debridement, microscopic fragments will still remain. These particles may cause pain through the creation of an inflammatory response in the tissues.

    Exchange of the femoral head should be performed with another ceramic head, as opposed to a metal head. If a metal head is used, abrasive wear will ensue as the microscopic fragments will scratch the femoral head due to differences in hardness.

     

    Traina et al. describe their experiences with revision of ceramic components. Most commonly, fractures of ceramic components occur as a result of trauma, dislocation, or errors in operative technique. These include head-neck taper mismatch, impacting the ceramic head with too much force, debris, and intraoperative damage to the metal neck taper.

     

    Hannouche et al. review ceramics in total hip replacement. They state that if the ceramic is properly manufactured, it can be a highly effective, low-wear solution for the young patient in need of a total hip replacement.

     

    Illustration A shows the typical ceramic femoral head used for a total hip arthroplasty. Illustration B shows a fractured ceramic head in many pieces.

     

    Incorrect Answers:

    Answer 1: The patients history & presentation are not consistent with infection Answer 3: Pseudotumor formation is most commonly associated with metal on metal prostheses.

    Answer 4: This presentation is not consistent with soft tissue metallosis; additionally that would be seen in metal on metal prostheses.

    Answer 5: While iliopsoas tendonitis is in the differential diagnosis for groin pain after a total hip replacement, the clinical situation is more consistent with third body wear as a result of retained ceramic fragments.

     

     

     

     

     

    OrthoCash 2020

     

  123. Figure A and B are radiographs of a 77-year-old patient presenting with right hip and upper thigh pain for the past 3 months. He is an avid golfer and plans to travel south for 6 months on a golf tour. He denies fever, chills or weight loss. His past medical history includes hypertension and a right total hip replacement 15 years ago. Physical examination reveals minimal pain with range of motion. ESR=10 (normal range 0-20) and CRP=4 (normal range 0-10). He does not want any further surgery. The patient is at the highest risk of which complication with non-operative care?

     

     

     

     

     

     

    1. Infection

    2. Pseudotumour formation

    3. Periprosthetic femoral fracture

    4. Periprosthetic acetabular fracture

    5. Dislocation

     

    Corrent answer: 3

     

    This patient has presented with significant osteolysis and aseptic loosening of his femoral THA component. If untreated, he is at an increased risk of a periprosthetic femur fracture.

     

    Indications for surgery for periprosthetic osteolysis include: pathological fracture, impending pathological fracture, symptomatic THA with evidence of osteolysis, and extensive osteolysis that would compromise revision surgery in the future. The goal of surgery is to remove the loose component, repair/bypass/replace bone deficiency, and obtain stable component fixation.

     

    Robbins et al. reviewed the causes of pain in THA. They report that hip pain can originate from the implant, soft tissue, or bone. The use of laboratory tests (e.g. ESR/CRP), radiographic and fluoroscopic imaging, hip aspirate, contrast arthrography and local anesthetic injections can help to determine the origin of pain.

     

    Ollivere et al. report that the most frequent cause of failure after total hip replacement in all reported arthroplasty registries is periprosthetic osteolysis. Osteolysis occurs with the activation of macrophages and a complex biological cascade that results in bone loss.

     

    Hirakawa et al. analyzed the circumstances around retrieved failed THA components. They showed that cement mantle defects, noncircumferential porous coatings, and screw holes are risk factors for osteolysis. They conclude by saying that the formation of a granulomatous tissue that ultimately invades the bone-implant interface is the final step in the pathogenesis of aseptic loosening.

     

    Figure A and B show AP and lateral views of a right THA. The femoral stem shows gross loosening in all zones. Subsidence is obvious with a high-riding greater trochanter. The lateral cement mantle is fractured. There is endosteal erosion distally with the tip of the stem showing radiographic toggle.

    Incorrect Answers:

    Answer 1: Infection should always be ruled-out in cases of osteolysis. In this case, however, there are no infectious symptoms and laboratory analysis is within normal ranges.

    Answer 2: Pseudotumour formation largely occurs with metal-on-metal components.

    Answer 4: Periprosthetic acetabular fracture is less likely. The cup has some

    osteolysis, but it remains well fixed. Acetabular fractures are less likely when there is minimal osteolysis.

    Answer 5: Hip dislocation can occur secondary to massive osteolysis. The long standing history from the index procedure make hip dislocation less likely. He has no other risk factors for dislocation.

     

     

     

    OrthoCash 2020

     

  124. A 60-year-old woman undergoes a total knee arthroplasty for end-stage osteoarthritis. Preoperative knee range of motion is 5 to 100 degrees. Postoperatively, she experiences reduced range of motion. She is scheduled to undergo manipulation under anesthesia. In which of the following scenarios is this procedure best indicated?

    1. Knee range of motion 0 to 60 degrees at 2 months postoperatively

    2. Knee range of motion 0 to 60 degrees at 8 months postoperatively

    3. Knee range of motion 30 to 120 degrees at 2 months postoperatively

    4. Knee range of motion 30 to 120 degrees at 8 months postoperatively

    5. Knee range of motion 30 to 120 degrees at 2 weeks postoperatively

     

    Corrent answer: 1

     

    Manipulation under anesthesia (MUA) can achieve the greatest gains in flexion when performed for patients with less than 90 degrees of flexion within the first three months.

     

    There are many risk factors for postoperative stiffness, the most important being preoperative stiffness. MUA is indicated when flexion is less than 90 degrees. Flexion gains are generally greater when applied early (6-12 weeks postoperatively) rather than late (>12 weeks). In cases with late-presenting stiffness (>12wks), MUA may still be attempted. Failed MUA is addressed with arthroscopic or open adhesiolysis +/- MUA, quadricepsplasty, or component revision.

     

    Namba et al. compared the results of early (<90 days) vs late (>90 days) MUA. They found that: (1) knee flexion improved a mean of 32 deg and 20 deg after early and late MUA respectively, (2) extension improved in the early MUA group, but not the late MUA group, and (3) pain improved after early but not late MUA. Despite early MUA being more desirable, the authors state that patients with limited flexion at 6-12 months may still benefit from late MUA.

     

    Keating et al. assessed the outcomes of MUA in 113 knees at a mean of 10 weeks after surgery. They found that (1) 90% of patients achieved

    improvement in knee flexion of 35 degrees at 5 year followup, (2) there was no difference in flexion gains between early (<12 weeks) and late (>12 weeks) MUA and (3) patients treated with MUA had better pain control than those without MUA. They concluded that manipulation can result in significant and lasting improvement in knee flexion.

     

    Incorrect Answers

    Answer 2: Late MUA (>3 months) is less effective than early MUA.

    Answers 3-5: Loss of flexion is better treated with MUA than loss of extension.

     

     

     

    OrthoCash 2020

     

  125. A 62-year-old man is scheduled for a total knee arthroplasty. In his pre-operative office visit, he asks questions about different tibial components. You tell him that compared with the tibial component shown in Figure A, the tibial component shown in Figure B:

     

     

     

     

     

     

    1. Is less expensive

    2. Has greater durability

    3. Has greater instability because of its monobloc nature

    4. Provides improved short-term functional status, but no difference in long term functional status

    5. Is associated with fewer adverse events because of easier implantation Corrent answer: 1

    Figure B shows an all-polyethylene tibia (APT) component, which is $470 to

    $1650 less expensive than metal-backed tibia (MBT) designs.

     

    It was traditionally thought that modular MBT may have lower survivorship (compared to APT) because of locking mechanism dysfunction, breakage,

    backside wear, and osteolysis. However, many studies now show the two to be comparable, with the only difference being that APT are less expensive.

     

    Voight et al. performed a systematic review comparing APT and MBT. They found that the former was cheaper. There was no difference in adverse events, durability (need for revision or radiographic failure) at 2, 10, and 15 years, and functional status at 2, 8, and 10 years.

     

    Toman et al. compared APT and MBT retrospectively. They found that APT implants perform as well as MBT implants in patients with BMI <37.5. There were 4 tibial implant failures in the MBT group in patients BMI >40.

     

    Dalury et al. examined APT performance in obese patients (125 knees) after a minimum of 7 years. There were no implant failures. There were 5 nonprogressive tibial radiolucencies and 1 case of nonprogressive osteolysis.

     

    Figure A shows a cemented metal-backed tibia component. Figure B shows a cemented all-polyethylene tibia component.

     

    Incorrect Answers:

    Answer 2: There is no difference in durability at up to 15 years.

    Answer 3: Instability (>0.2 mm migration) was reduced by 48% with an APT (compared with MBT) but this was not significant (p = 0.05, Voight study).

    Answer 4: There is no difference in functional status at short-, medium- and long-term followup.

    Answer 5: There is no difference in adverse events.

     

     

     

    OrthoCash 2020

     

  126. Figures A and B show pre- and post-operative radiographs of a sedentary 75-year-old female who underwent surgery on her left hip. Based on the radiographic findings, what was the most likely indication for revision surgery?

     

     

     

     

     

     

    1. Left acetabular fracture

    2. Left acetabular cup osteolysis

    3. Left femoral stem osteolysis

    4. Left hip instability

    5. Left femoral stem valgus malalignment Corrent answer: 4

    Figure A shows a left total hip arthroplasty with eccentric polyethylene wear. Figure B shows that her left hip was revised to a constrained acetabular liner, most likely a result of recurrent instability.

    Revision strategies for hip instability are typically directed at correcting the underlying cause of instability. For example, instability most commonly occurs as a result of poor implant design, positioning or loosening, or the loss of soft-tissue function or tensioning. Operative strategies are designed to correct these etiologies by repositioning or exchanging components, integrating modular designs and improving soft tissue tensioning, etc. Constrained acetabular liners are often used in conjunction with these modalities to address the problem of recurrent instability relating to soft tissue deficiency and dysfunction in the affected hip.

     

    Alberton et al. retrospectively reviewed 1548 revision arthroplasties for the incidence of dislocation. They found the overall dislocation rate to be 7.8%. Factors contributing to increased dislocations were found to be trochanteric non-unions, femoral heads <28mm in diameter and extensive soft-tissue dissection. Protective factors were modular acetabular components or liners, larger femoral heads >28mm and re-establishing abductor tensioning.

     

    Paterno et al. retrospectively reviewed 438 primary and 181 revision total hip arthroplasties for patient factors contributing to dislocation. They found an overall dislocation rate of 6%. 23% of patients with a history of excessive intake of alcoholic beverages (more than six ounces a day) had at least one dislocation. There was no relationship between the variables of age, gender, obesity, or preoperative diagnosis and the incidence of dislocation.

     

    Figure A shows bilateral primary cementless, nonconstrained total hip replacements. The left hip shows eccentric femoral head placement within the acetabulum indicative of eccentric polyethylene wear. Figure B shows the conversion to a constrained, dual-mobility, polyethylene liner. The overall metal component position appears satisfactory.

     

    Incorrect Answers:

    Answer 1: There is no radigraphic finding of fracture. In addition, acetabular fractures would not be treated with conversion to a constrained liner.

    Answer 2,3: The presence of osteolysis in the femoral and acetabular components is not significant based on these radiographic images.

    Answer 5: There is no valgus malalignment of the left femoral implant.

     

     

     

    OrthoCash 2020

     

  127. A 62-year-old woman is brought to the emergency room after falling down a flight of stairs. Prior to her fall, she had no knee pain and was a community ambulator without assistance. Intraoperatively,

    it is determined that the implants are well-fixed. What is the best next treatment step to optimize her quality of life?

     

     

     

     

    1. Closed reduction and long leg casting at 20 degrees of flexion for 6 weeks, followed by hinged-knee brace for 6 weeks.

    2. Open reduction and internal fixation with a distal femoral locking plate

    3. Open reduction and internal fixation with a condylar buttress plate

    4. Distal femoral replacement arthroplasty

    5. Closed reduction and fixation with an antegrade intramedullary nail Corrent answer: 2

    This patient has a displaced far-distal supracondylar fracture around a stable TKA femoral component. Locked plating is the best option for management of this fracture.

     

    Surgical fixation of periprosthetic fractures around a stable femoral component is challenging. Locked plating allows for multiple angle-stable fixation points around stems and lugs and does not depend on TKA design or quality of distal bone stock for fixation. Su Type I fractures may be treated with retrograde or antegrade intramedullary nailing. Type II fractures require retrograde intramedullary nailing or fixed-angle plating. Type III fractures require fixation with a fixed-angle device or revision arthroplasty when bone stock is poor.

     

    Ricci et al. evaluated indirect reduction and locked lateral plating of Vancouver B1 THA fractures without allograft struts. They found that all fractures healed with satisfactory alignment and without implant loosening at an average of 12 weeks. They recommend this technique for stable Vancouver B1 fractures.

    Streubel et al. examined the outcomes of locked plating in treatment of extreme distal periprosthetic supracondylar fractures located proximal to the flange (Su Types I and II) compared with fractures distal to the flange (Su Type III, see Illustration B). They found no difference in delayed union, nonunion, infection and failure rates between the 2 groups.

     

    Figure A shows a Su Type III periprosthetic fracture around a TKA femoral component. Illustration A shows fixation of the same fracture with a distal femur locking plate. Illustration B shows the Su classification of fractures around the femoral component (Type I, proximal to the femoral component; Type II, starting at the anterior flange and extending proximally; Type III, fracture line distal to the anterior flange).

     

    Incorrect Answers

    Answer 1: High rates of malunion and nonunion are associated with nonoperative treatment

    Answer 3: The complication rate after non-locked plating is high and nonunion rates of up to 50% have been observed.

    Answer 4: Distal femoral replacement arthroplasty is a good choice if bone stock is poor and the component is loose.

    Answer 5: There is insufficient distal bone stock for interlocking screw purchase for antegrade intramedullary nailing. Retrograde nailing might be possible with this CR implant using far distal fixed-angle interlocking screws that lie distal to the anterior flange of the prosthesis.

     

     

     

     

     

     

     

     

    OrthoCash 2020

     

  128. Which of the following fractures would most likely require revision arthroplasty with a long-stemmed, uncemented prosthesis?

     

     

     

     

     

     

     

     

     

     

     

     

     

    1. Figure A

    2. Figure B

    3. Figure C

    4. Figure D

    5. Figure E

     

    Corrent answer: 2

     

    Figure B shows a Vancouver B2 periprosthetic femur fracture with an unstable femoral stem that requires revision arthroplasty with a long-stemmed prosthesis.

     

    The Vancouver classification for periprosthetic femur fractures can help guide treatment of these challenging problems. Vancouver A fractures involve the greater and lesser trochanter and can be initially managed with non-operative measures. Vancouver B fractures occur around the stem and are broken down into B1 (stable prosthesis), B2 (unstable prosthesis) and B3 (poor proximal bone quality) fractures. B1 fractures may be treated with internal fixation, B2 fractures require a revision arthroplasty, and B3 fractures often require more advanced reconstruction with a proximal femoral replacement versus revision with a distally fixed prosthesis. Vancouver C fractures occur distal to the stem and require internal fixation.

    Springer et al. reviewed 118 patients who underwent revision arthroplasty for Vancouver B2 periprosthetic fractures. They had a 90% survival rate at 5-years and a 79.2% survival rate at 10-years. The most common reasons for revision were loosening, infection, and non-union.

     

    Illustration A shows the Vancouver classification of periprosthetic fractures about the femur.

     

    Incorrect Answers:

    Answer 1: Figure A shows an interprosthetic fracture with stable components best treated with internal fixation.

    Answer 3: Figure C shows a Vancouver A fracture best treated with either nonoperative management or internal fixation.

    Answer 4: Figure D shows another interprosthetic fracture with stable components best treated with internal fixation.

    Answer 5: Figure E shows a Vancouver B1 fracture with a stable component best treated with internal fixation.

     

     

     

     

     

     

    OrthoCash 2020

     

  129. A 65-year-old female sustains a periprosthetic supracondylar femur fracture proximal to a well-fixed implant. She undergoes direct reduction and locked plating with a titanium distal femoral locking plate via an extensile lateral approach. At 9 months post-operatively, weightbearing is at 50% and is painful. Examination reveals mild swelling and warmth around the distal incision. Erythrocyte sedimentation rate and C-reactive protein are normal. Radiographs taken 9 months post-operatively are shown in Figure A. Which of the following may have increased the risk of this complication?

     

     

     

    1. Neglecting to add topical rhBMP-2 on a carrier-scaffold

    2. Neglecting to use lag screws and cerclage cables

    3. Locked plating instead of locked antegrade nailing

    4. Use of a titanium plate instead of a stainless steel plate

    5. Use of an extensile lateral approach instead of a submuscular approach Corrent answer: 5

    A submuscular approach has been shown to have less risk of nonunion than an extensile lateral approach. There is less disruption of soft tissue attachments and devitalization of fracture fragments with the submuscular approach.

     

    The risks for periprosthetic fractures include notching, knee stiffness, osteoporosis, poor mobility and falls. The risk is higher in females and after revision surgery. The treatment of periprosthetic supracondylar fractures depends on the location of the fracture, fixation of the implant, and bone stock.

     

    Hoffman et al. retrospectively reviewed 36 periprosthetic supracondylar femur fractures treated with locked plating. They found that submuscular plating had reduced nonunion risk compared to an extensive lateral approach. They recommend indirect reduction and submuscular plating to reduce the incidence of nonunion.

     

    Hou et al. retrospectively reviewed 53 fractures fixed with retrograde nailing

    (18) and locked plating (34). They found no difference in blood loss, time to union, operating time and hospital stay. They believe locked plating can

    provide the same favorable results as retrograde nailing and recommend this technique for most patients and prosthetic designs.

     

    Figure A shows nonunion and surrounding osteopenia after locked plating of a periprosthetic supracondylar fracture. Illustration A shows management of these fractures according to the Su classification.

     

    Incorrect Answers

    Answer 1: rhBMP-2 is not FDA approved for femoral fractures. It is only approved for acute, open tibial shaft fractures stabilized with intramedullary nail fixation, or spine fusion at L4-S1 for degenerative disc disease via an anterior approach.

    Answer 2: Lag screws and cerclage cables do not decrease the risk of nonunion. Placement is difficult with short oblique fractures at the metaphysis. Answer 3: There is insufficient distal locking screw purchase for antegrade nailing.

    Answer 4: The metallurgy of locked plates has not been shown to affect nonunion rates.

     

     

     

     

     

     

    OrthoCash 2020

     

  130. Immediately following a total hip arthroplasty (THA), a healthy 55-year-old patient is unable to dorsiflex her ankle or extend her great toe. After 4 weeks she continues to ambulate with a "slapping gait." Examination reveals passive ankle joint dorsiflexion to 10 degrees. What is the most appropriate next treatment option?

     

     

     

     

     

     

    1. MRI of her spine and pelvis

    2. Revision total hip arthroplasty

    3. Ankle-foot orthosis

    4. Posterior tibial tendon transfer to navicular bone

    5. Neurology consult

     

    Corrent answer: 3

    This patient is presenting with foot drop after a THA for hip dysplasia (Crowe 4). The most appropriate treatment at this stage would be providing her with an ankle foot orthosis (AFO) for mobility.

     

    Sciatic nerve injury after THA is an uncommon and difficult situation to manage. Patients with DDH that have undergone a large limb-lengthening procedure are at a greater risk due to the significant stretch of the sciatic nerve. Intra-operative procedures that have been shown to prevent this outcome include good pre-operative planning, limb lengthening <3 cm, subtrochanteric osteotomy, intra-operative wake-up test, and downsizing implant components if presenting with deficits peri-operatively. An ankle-foot orthosis may be used for foot drop when surgery is not warranted or during neurologic recovery.

     

    Prahinski et al. reviewed the Bridle transfer for paresis of the anterior and lateral compartment musculature. Indications for the procedure include no neurological recovery for at least 18 months after injury or 18 months after the most recent attempt at exploration or repair of the nerve.

    Electrodiagnostic changes must indicate permanent damage. In addition, there must be good passive range of motion, with at least 90° of dorsiflexion.

     

    Edwards et al. reported on twenty-three peroneal and sciatic nerve palsies that occurred in patients following total hip arthroplasty. They showed that peroneal nerve palsy occurred with an average lengthening of 2.7 cm (1.9 cm-3.7 cm) in comparison with 4.4 cm (4.0 cm-5.1 cm) for sciatic nerve palsies.

     

    Figure A and B show pre- and post-operative images of a patient with severe hip dysplasia treated with primary total hip arthroplasty.

     

    Incorrect Answers:

    Answer 1: MRI is often not useful due to the signal artifact from the implants around the hip.

    Answer 2: There would be no indication for revision surgery of well positioned implants after 4 weeks.

    Answer 4: Tendon transfer procedures would be considered in chronic paresis situations.

    Answer 5: Neurology consult would be appropriate, but not the most appropriate next treatment option. The primary objective is to help the patient regain function. Neurological consult would be important if concerned of some underlying neurological disease process. Referral would be most appropriate after 3-6 months if no return of function.

     

     

    OrthoCash 2020

     

  131. A 60-year-old male with history of renal transplantation and previous intravenous drug abuse undergoes total knee arthroplasty. Two years later, he begins to have mild knee pain and low-grade swelling that persists for 10 months before he finally comes to the emergency room. Examination reveals no fever. Range of motion is 5 to 70 degrees. Erythrocyte sedimentation rate is 22mm/h, and C-reactive protein is 0.8mg/L. Knee aspiration reveals 12,000/mm3 nucleated cells with 76% neutrophils. Gram stain is negative and aerobic and anaerobic cultures are negative after 4 days in culture. His symptoms do not resolve after 5 days of empiric intravenous antibiotics and he is taken to the operating room for arthroscopic irrigation and debridement. Operative synovial tissue cultures are shown in Figure A. What is the best next step?

     

     

     

     

     

    1. Cessation of immunosuppressant medication, lifelong antimycobacterial suppression

    2. Open irrigation and debridement, implant retention and lifelong antifungal suppression

    3. Open irrigation and debridement, resection arthroplasty, antimycobacterial drugs for 6 to 12 months

    4. Open irrigation and debridement, single-stage exchange, antifungal drugs for 6 to 12 months

    5. Open irrigation and debridement, two-stage exchange, antifungal drugs for 6 to 12 months

    Corrent answer: 5

     

    This patient has a fungal prosthetic joint infection (PJI) with Candida albicans. Optimal treatment involves resection arthroplasty, delayed reimplantation

    arthroplasty, and antifungal drugs for 6-12 months.

     

    Fungal PJI are uncommon. Risk factors include immune suppression and systemic illness e.g. diabetes and chronic renal failure. Candida species is usually the causative organism. The infection is usually indolent and systemic symptoms (e.g. fever) may be absent. ESR and CRP may be only minimally elevated. Two-stage exchange arthroplasty is standard of care.

     

    Phelan et al. described delayed reimplantation in 10 patients with fungal PJI. They found that the median time from resection to reimplantation arthroplasty was 9 and 2 months for total hip and total knee arthroplasty respectively. Two patients had recurrence of infection. They recommend antifungal therapy and delayed reimplantation arthroplasty after confirmation of an infection-free period as the best chance for cure.

     

    Azzam et al. retrospectively reviewed arthroplasty database data to identify 31 fungal PJIs in 6 centers. Delayed implantation was performed in 19 of 29 patients who underwent resection arthroplasty at an average of 7 months.

    They recommend two-stage exchange arthroplasty as the treatment of choice, addition of antibacterial drugs to the cement spacer to prevent superinfection, antifungal drugs for 6-12 months, repeat joint aspirations prior to reimplantation, and optimization of host nutritional status prior to reimplantation.

     

    Figure A is a high-powered micrograph showing synovial tissue covered by fibrinopurulent exudates containing fungal colonies of Candida albicans.

     

    Incorrect Answers:

    Answer 1: Drug therapy alone will only suppress symptoms at the expense of potential toxic side effects, and is unlikely to eradicate the infection. Cessation of immunosuppression and lifelong antifungal treatment will be detrimental to the transplanted kidney.

    Answer 2: Debridement alone, with implant retention, is unlikely to control the infection as most infections are chronic infections in immunocompromised hosts, both of which are recognized causes of failure of debridement alone.

    Answer 3: While resection arthroplasty (without secondary reimplantation) is acceptable therapy, his infection is fungal in nature and should be treated with antifungals.

    Answer 4: Single-stage exchange for fungal PJI has rarely been successful and is ill-advised because of the high recurrence rate. Recurrence rates of 20-25% after two-stage exchange has been reported.

     

     

    OrthoCash 2020

     

  132. Which of the following non-operative treatments for osteoarthritis has the best evidence to support its use?

    1. Combination of supervised and home exercise programs

    2. Hyaluronic acid injections

    3. Lateral heel wedge

    4. Acetaminophen

    5. Glucosamine

     

    Corrent answer: 1

     

    Of the options listed, a combination of home and supervised exercise has the best supporting evidence for the treatment of osteoarthritis.

     

    The AAOS has recently developed guidelines for the treatment of osteoarthritis. Therapies that are recommended by the AAOS include weight loss, home and supervised exercise programs, and NSAIDs/tramadol.

    Therapies that remain inconclusive (lack of supporting evidence) include electrotherapeutic modalities, manual therapy, bracing, acetaminophen/opiods, steroid injections and PRP. Glucosamine, lateral heel wedges and hyaluronic acid injections are not recommended, as current literature has shown them to be ineffective. Keep in mind that these guidelines are subject to change as new literature is published.

     

    Zhang et al. present a systematic review of the literature on arthritis management in the three years following the original OA Research Society International (OARSI) guidelines published in 2006. While weight loss showed an increase in effectiveness with the addition of new studies, electromagnetic therapy, glucosamine, chondroitin sulfate, and hyaluronic acid injections showed a decrease in effectiveness.

     

    Incorrect Answers:

    Answer 2: Current AAOS guidelines recommend against hyaluronic acid injections as they are not supported by evidence.

    Answer 3: AAOS guidelines recommend against the use of lateral heel wedges. Answer 4: AAOS guidelines show inconclusive evidence for the use of acetaminophen.

    Answer 5: AAOS guidelines state that the use of glucosamine is not supported by current evidence.

     

     

     

    OrthoCash 2020

  133. Figure A shows the 2 bundles of the ACL dissected from a cadaveric knee off their bony attachments. They are labeled Bundle A and Bundle B, respectively. Which of the following is true?

     

     

     

     

     

    1. The tibial attachment of Bundle A is anterior to Bundle B. In extension, Bundle B is loose and Bundle A is tight.

    2. The tibial attachment of Bundle A is anterior to Bundle B. In flexion, Bundle B is loose and Bundle A is tight.

    3. The tibial attachment of Bundle B is anterior to Bundle A. In flexion, Bundle B is loose and Bundle A is tight.

    4. The tibial attachment of Bundle B is anterior to Bundle A. In flexion, Bundle A is loose and Bundle B is tight.

    5. The tibial attachment of Bundle B is anterior to Bundle A. In extension, Bundle A is loose and Bundle B is tight.

    Corrent answer: 2

     

    Bundle A is the anteromedial (AM) bundle, which is longer, and is tight in flexion. Bundle B is the posterolateral (PL) bundle, which is shorter, and is loose in flexion. The AM bundle is attached anterior to the PL bundle on the tibia.

     

    The ACL is comprised of 2 bundles. The AM bundle is longer than the PL bundle. Their names reflect their relative anatomic positions on the tibial insertion site. On the femur, the AM bundle begins at the proximal-anterior aspect of the femoral insertion site, while the PL bundle begins at the posterior-inferior part. In flexion, the AM bundle is tight and the PL bundle is loose. In extension, the AM bundle is loose and the PL bundle is tight.

    Bicer et al. reviewed the anatomy of the ACL. They found that the AM bundle was longer (32mm) compared with the PL bundle (18mm). PL bundle carries greater force near full extension, and the AM bundle carries greater force after 15-45° of flexion. Under combined rotatory loads (valgus and internal tibial torque at knee flexion >30°), the AM bundle bore more force than the PL bundle.

     

    Figure A shows the 2 bundles of the ACL. The AM bundle is longer than the PL bundle. The oft referred to length of ACL refers mainly to the length of the AM bundle. Illustrations A and B show the spatial relationships of the AM and PL bundles in a cadaveric knee. Illustration C shows the relative positions of the attachments of each bundle.

     

    Incorrect Answers:

    Answer 1: In extension, Bundle B (PL) is tight, and Bundle A (AM) is loose. Answers 3 to 5: The tibial attachment of Bundle A (AM) is anterior to Bundle B (PL).

     

     

     

     

     

     

     

     

     

     

    OrthoCash 2020

     

  134. An 83-year-old man, who had a total hip arthroplasty performed 13 years ago, is referred to your office for evaluation. He reports worsening groin pain over the past year, which has been increasing in frequency. Prior to this past year, he had no other complaints. His current radiograph is shown in Figure A. If he continues to ambulate with this implant, he is at greatest risk for which of the following?

     

     

     

     

     

    1. Infection

    2. Acetabular component loosening

    3. Femoral component loosening

    4. Dislocation

    5. Periprosthetic fracture

     

    Corrent answer: 4

     

    The patient has eccentric polyethylene wear secondary to component malpositioning. He is at highest risk for dislocation.

     

    Late dislocation following total hip arthroplasty(THA) can occur and has a high recurrence rate, thereafter. Risk factors include eccentric polyethylene, THA at an early age, neurologic decline or associated neurologic conditions (i.e.

    Parkinson's disease), or associated trauma.

    Parvizi et al. noted in this instructional course lecture that eccentric, excessive polyethylene wear is one of the most common reasons for late, recurrent dislocation. Revision is recommended.

     

    Pulido et al. in this review, reiterated that polyethylene wear can lead to increased inflammation, capsular distention, and instability, increasing risk for dislocation.

     

    von Knoch et al. reviewing over 500 dislocated hips, also noted that eccentric wear was one of major causes linked to late dislocation.

    Figure A. exhibits a left total hip arthroplasty with eccentric wear. Incorrect answers:

    Answer 1. This patient is not at increased risk for infection.

    Answers 2 and 3. While this patient is at increased risk for loosening, the risk of dislocation due to eccentric wear.

    Answer 5. This patient is not at increased risk for fracture.

     

     

     

     

    OrthoCash 2020

     

  135. Figure A shows a radiograph of a 62-year-old female that underwent a left total hip arthroplasty 5 years ago. She presents to your office with insidious onset of left groin and buttock pain. She denies trauma, fever or chills. On physical examination, her left hip has mild pain with range of motion. She has a normal gait cycle, normal power across the hip and her vitals signs are stable. A left hip aspirate was performed and results are shown in Figure B. What is the most likely cause of her hip pain?

     

     

     

     

     

     

    1. Periprosthetic bacterial hip infection

    2. Periprosthetic hip fracture

    3. Large-particle wear debris disease

    4. Pseudotumor hypersensitivity response

    5. Abductor tendon tear Corrent answer: 4

    This patient is presenting with a metal induced system hypersensitivity response in the setting of a metal-on-metal total hip arthroplasty.

     

    A hip aspiration of a painful THR is a very useful investigation for the work up of infection, having a sensitivity of 75-85% and specificity of 85-100% for

    infection. Metal-on-metal THA may mimic infection as aspirate results will often show increased inflammatory infiltrate, with synovial WBC counts in the thousands. However, infected THA are more likely to produce higher percentages of PMNs (>70%) in comparison to hypersensitivity reactions/ adverse reaction to metal debris, which are more likely to produce a higher percentage of lymphocytes (>40%).

     

    Campbell et al. looked at the histological features of pseudotumor-like tissues from metal-on-metal hips. They found that the patients with hip pain and suspected metal sensitivity had fewer metal particles but more aseptic lymphocytic vasculitis-associated lesions compared to patients with evidence of metallic wear. They concluded that pseudotumors occur more because of a hypersensitivity reaction than particle wear.

     

    Kwon et al. examined a small cohort of patients with metal-on-metal hip arthroplasties to investigate the incidence and level of metal-induced systemic hypersensitivity. They found that lymphocyte reactivity to Co, Cr, and Ni did not significantly differ in patients with pseudotumors compared to those patients without pseudotumors. This suggests that systemic hypersensitivity type IV reactions may not be the dominant biological reaction involved in the occurrence of the soft tissue pseudotumors.

     

    Figure A shows a patient with bilateral metal-on-metal total hip arthroplasties. There are no identifiable fractures. The position of the left acetabular cup is slightly vertical, which can increase edge loading and particle wear. Figure B shows the results from the hip aspirate.

     

    Incorrect Answers:

    Answer 1: Although WBCs > 3000, the low differential of PMNs and high lymphocytes are not consistent with a bacterial joint infection. Infected THA are more likely to produce higher percentages of PMNs (>70%).

    Answer 2: Radiographs and physical exam do not suggest fracture, although CT scan or bone scan may be useful to detect subtle periprosthetic hip fractures.

    Answer 3: Large-particle wear debris disease most commonly occurs with polyethylene wear. This is a metal-on-metal hip replacement. Metal surfaces are thought to give off smaller particles of debris.

    Answer 5: Abductor tendon tear would present with an abnormal gait and some element of decreased abductor strength. The aspirate would also be negative.

     

     

     

    OrthoCash 2020

  136. A 72-year-old patient is scheduled to undergo revision total hip arthroplasty. A 3D-model of the patient's hemipelvis is constructed for pre-operative planning and is shown in Figure A. A custom-designed implant shown in Figure B is created. Which of the following is TRUE of the planned reconstruction?

     

     

     

     

     

     

     

     

    1. The implant is a bilobed cup.

    2. The most common complication is dislocation.

    3. The acetabular defect can be classified as AAOS Type V.

    4. Radiation-compromised bone stock is a contraindication.

    5. The winged profile of the implant facilitates insertion through both anterior and anterolateral approaches.

    Corrent answer: 2

     

    The patient has pelvic discontinuity that will be reconstructed with a custom triflange acetabular component. Dislocation is the most common complication.

     

    Custom triflange acetabular components are indicated for severe acetabular bone loss and pelvic discontinuity that are not amenable to treatment with off-the-shelf implants such as reconstruction plates, jumbo cups and antiprotrusio cages. Dislocation is common and possible etiologies include extensive dissection, less reliable soft tissue repair, deficient abductors/trochanteric nonunion, superior gluteal nerve stretch neuropraxia, and surgeon reluctance to use constrained liners in the face of poor bone stock.

     

    Christie et al. reviewed reconstruction with the triflange cup in 78 hips with AAOS Type III (combined deficiency) or Type IV (pelvic discontinuity) defects. They found improvement in Harris hip scores, limp, need for walking aids.

    Dislocation was the most common complication (15.6%, 12 patients), and half of these patients (6/12) needed re-operation for recurrent dislocation. They recommend the triflange cup for difficult reconstructions involving severe bone loss.

     

    Taunton et al. reviewed 57 patients with pelvic discontinuity treated with a custom triflange component. They found that 21% developed instability (10 required revision, and 2 treated nonoperatively). Of note, 51% had preop trochanteric escape (nonunion of the greater trochanter to the femoral component or femur with >1cm of displacement. They recommend the custom triflange implant for discontinuity as it provides predictable midterm fixation and consistent healing.

     

    Figure A is a 3D hemipelvis model generated by stereolithography from a patient’s CT scan. It shows massive bone loss and pelvic discontinuity. Figure B is a custom hydroxyapatite (HA)-coated porous triflange acetabular prosthesis with ilial and ischial screw holes. Illustration A shows a bilobed cup and its appearance on an AP radiograph.

     

    Incorrect Answers:

    Answer 1: This is a custom triflange implant.

    Answer 3: The acetabular defect is AAOS Type IV (pelvic discontinuity). AAOS Type V is an arthrodesed hip.

    Answer 4: The implant is especially indicated for radiation-compromised bone stock.

    Answer 5: An anterior and anterolateral approach may allow for screw fixation of the pubic wing, but will make screw fixation of the ischial and ilial wings impossible without detachment of the abductors.

     

     

     

     

     

     

     

    OrthoCash 2020

     

  137. All of the following are risk factors for wear-related failure in total knee arthroplasty when using a polyethylene liner that underwent sterilization via gamma irradiation in air EXCEPT?

    1. Increasing shelf age of polyethylene liner

    2. Younger age of patient

    3. Male gender

    4. Posterior cruciate retaining knee design

    5. Use of a rough tibial baseplate Corrent answer: 4

    Increasing shelf age, younger age, male gender, and a rough tibial baseplate are all risk factors for wear-related failure in total knee arthroplasty when using a polyethylene liner. Posterior cruciate retaining knee design is not a documented risk factor.

     

    Fehring et al reviewed 2091 TKA using the Press fit condylar system and noted that the 13-year survivorship for all patients was 82.6% with a 8.3% prevalence of wear-related failure. Cox hazards analysis revealed five variables that were correlated with wear-related failure: patient age, patient gender, polyethylene sheet vendor, polyethylene finishing method, and polyethylene shelf age. They were unable to identify one factor as the defining reason for these wear-related failures. They cautioned that these findings may only be specific to inserts that underwent sterilization via gamma irradiation in air.

     

    Collier et al followed 365 TKA (PCL-retaining) for 5-10 yrs and noted that

    factors related to polyethylene insert osteolysis included advanced shelf age, sterilization method, and the material from which it was machined. Osteolysis was identified in 34% with an insert that had been gamma-irradiated in air and affixed to a rough baseplate surface, but only 9% when the insert had been gamma-irradiated in an inert gas or not irradiated at all and joined to a polished surface.

     

     

     

    OrthoCash 2020

     

  138. A 65-year-old healthy patient fell 18 years after a total hip arthroplasty and sustained the fracture shown in Figure A. Which of the following would be the most appropriate treatment?

     

     

     

     

     

    1. Percutaneous locked plating

    2. Open reduction internal fixation with a cable plate and allograft strut

    3. Revision to a long femoral stem with allograft bone

    4. Revision to a cemented revision femoral stem that bypasses the fracture site by 5 cm

    5. Three months of non-weight bearing Corrent answer: 3

    The Vancouver classification of periprosthetic femur fractures is based on the fracture site, implant stability, and remaining bone stock. The patient in the question has a type B3 fracture. The cemented stem is loose and there is very poor remaining bone stock. He should be treated with a long, cementless

    revision stem with biplanar strut grafts. A tumor prosthesis or allograft-prosthesis composite would be alternate possibilities. Illustrations A and B are a diagram and table of the Vancouver classification of periprosthetic hip fractures.

     

    Springer, et al. looked at the results and complications of revision total hip arthroplasty for the treatment of acute Vancouver type-B periprosthetic femoral fracture. In their series they treated these fractures in multiple ways, including cemented stems, uncemented stems, allograft-prosthetic composite, or tumor prosthesis. They concluded that the best results were with an uncemented, porous coated femoral stem, and the most common cause of revision was loosening.

     

    Parvizi, et al. concluded that due to the poor bone quality and delayed healing of older patients & their periprosthetic fractures that it is imperative that a strong mechanical construct be achieved in the treatment of these fractures. They “advocate the use of numerous screws with purchase of at least ten cortices and reinforcement of fixation with biplanar strut allografts whenever possible. When a revision stem is used, we ensure that adequate diaphyseal fixation is obtained and the fracture is traversed by at least 5 to 8 cm.”

     

     

     

     

     

     

     

     

     

    OrthoCash 2020

     

  139. A 85-year-old man who underwent hemiarthroplasty 5 years ago now complains of thigh pain for the past four months. Laboratory studies show a normal white blood cell count (WBC), erythrocyte sedimentation rate (ESR) and C-reactive protein (CRP). An aspiration of the hip is performed and is negative for infection. A radiograph is shown in Figure A. Which of the following is the best management option for the femoral implant?

     

     

     

     

     

    1. Bone scan to look for loosening

    2. Touch down weight bearing and physical therapy

    3. Revision with a tumor prosthesis

    4. Revision of femoral component with metaphyseal cement fixation of the stem

    5. Revision to a cementless femoral component with diaphyseal press-fit fixation of the stem

    Corrent answer: 5

     

    The radiograph shows lucency around the femoral stem cement mantle consistent with loosening. There is bone loss in the proximal femur. Diaphyseal fixation is the best option from the choices available. Revision to a cementless femoral stem is the most appropriate management.

     

    Paprosky et al. described their results of revision to cementless femoral components and report 95% survivorship with a minimum of 10 years follow

    up.

     

    Haydon et al showed that despite historical literature discouraging the use of cemented femurs for revision, in their experience cemented femoral revision had 91% survivorship when the cause was aseptic loosening. They found early generation cementing techniques, poor cement mantle, poor bone quality, age of less than 60, and male gender to be risk factors for failure in cemented revisions.

     

     

     

    OrthoCash 2020

     

  140. Figure A is a diagram showing the medial side of the knee. During a total knee arthroplasty, proximal tibia resection results in the transection of the ligament in Figure A along the red line. Intraoperative examination reveals coronal plane instability. What are the best next steps?

     

     

     

     

     

     

     

     

    1. Use of the implant shown in Figure B, and use of a hinged knee brace postoperatively

    2. Suture repair of the torn ligament, use of the implant shown in Figure C

    3. Use of the implant shown in Figure D, and use of a knee immobilizer postoperatively

    4. Suture repair of the torn ligament, use of the implant shown in Figure D, and use of a hinged knee brace postoperatively

    5. Use of the implant shown in Figure C alone Corrent answer: 4

    This patient has intraoperative midsubstance transection of the MCL. MCL repair, use of either a CR or PS implant, and postoperative knee bracing for 6 weeks is recommended. A possible alternative is the use of an unlinked constrained implant.

     

    The MCL is likely to be compromised by medially placed retractors or during medial subperiosteal elevation (tibial avulsion) or injured by oscillating saw-blade during the tibial or posterior femoral condyle cut. There is no consensus for the treatment of intraoperative rupture. Acceptable salvage options include

    (1) direct repair (heavy sutures for midsubstance rupture, and suture anchors for tibial sleeve avusions) and postop knee bracing for 6 weeks with either CR or PS implants, or (2) use of unlinked constrained implants with or without repair.

     

    Lee and Lotke reviewed 37 patients with intraoperative MCL injury out of 1478 patients. They attempted repair in 14 patients, and increased constraint in 30

    patients. They found higher failure rates (regardless of MCL repair technique) for cruciate retaining components. They recommend use of an unlinked constrained prosthesis (with or without ligament repair), especially for midsubstance injuries.

     

    Leopold et al. reviewed 16 MCL injuries in 600 knees. They performed suture or suture anchor repair and used a hinged knee brace for 6 weeks postoperatively. All limbs were stable and did not require bracing beyond 6 weeks, demonstrated acceptable alignment, and did not require revision at 45 months. They recommend the use of primary MCL repair or reattachment and postoperative bracing instead of implants with increased constraint.

     

    Figure A shows MCL transection in its midsubstance. Figure B shows a cruciate retaining implant. Figure C shows a hinged knee prosthesis (linked constrained implant). Figure D shows a posterior stabilized implant. Illustration A shows an unlinked constrained implant.

     

    Incorrect Answers:

    Answer 1: A cruciate retaining implant can be used (together with postoperative knee bracing) provided MCL repair/reattachment is performed. On its own, a CR implant cannot control coronal instability from a ruptured MCL.

    Answers 2 and 5: A hinged knee is not indicated for simple ruptures of collateral ligaments. A hinged knee prosthesis is indicated for moderate to severe instability, ligament deficiency (eg, absence of 1 or both collateral ligaments), severe bone loss, or varus, valgus, or flexion deformities.

    Answer 3: A posterior stabilized implant can be used (together with postoperative knee bracing) provided MCL repair/reattachment is performed. On its own, a PS implant cannot control coronal instability from a ruptured MCL.

     

     

     

     

     

     

    OrthoCash 2020

     

  141. A 65-year-old woman complains of intermittent knee pain 12 years after a total knee arthroplasty. She has no history of fever or recent infections. Radiographs are shown in Figures A and B. Examination reveals minimal warmth and a moderate knee effusion. Range of motion is 5 to 100 degrees bilaterally. The C-reactive protein level is 15 mg/L (normal, 0.0-0.8mg/L), and erythrocyte sedimentation rate is 45mm/h (normal, 0-10mm/h). Arthrocentesis reveals 7500 white blood cells and 90% neutrophils. Gram stain is negative. Cultures are negative at 3 days. What is the next best step?

     

     

     

     

     

    1. MRI with metal subtraction protocol

    2. Arthroscopic debridement

    3. Open debridement and polyethylene liner exchange

    4. Single-stage revision total knee arthroplasty (TKA)

    5. Explantation of components with two-stage revision TKA Corrent answer: 5

    By the updated 2018 Musculoskeletal Infection Society (MSIS) criteria, this presentation is consistent with a diagnosis of periprosthetic joint infection (PJI). The patient has an elevated CRP (2), ESR (1), synovial WBC >3,000 (3), and >80% PMNs (2), for a total of 8 points. Given the chronicity of the infection, the patietn would be a candidate for two-stage revision.

     

    This patient has clinical signs of PJI such as elevated laboratory values and radiographs suggestive of implant loosening. Even in the absence of positive cultures, the next most supported step in management if two-stage revision with explantation of the prosthesis and insertion of an antibiotic spacer.

    Intraoperative cultures should be taken to guide post-operative antibiotic treatment.

     

    Parvizi et al. recently released the updated 2018 MSI crtieria for diagnosis of PJI. The updated criteria (Illustration A) included new diagnostic tests and studies from the seven-year period since the previous criteria were established. Alpha defensin was a new addition. The two major criteria remained, each individually diagnostic of PJI. However the minor crtieria were broken down into pre-operative and intra-operative. The authors showed that a total of 6 points or more had a 97.7% SN and 99.5% SP for PJI.

     

    Huang et al. retrospectively reported the infection control rates in 2-stage exchanges in 55 patients, and compared culture-negative cases with 295 culture-positive cases. They found that infection control in culture-negative cases was 73% at 1-year. Infection control rates were similar in culture-negative and culture-positive cases, and that infection-free survival is highest after 2-stage exchange with postoperative vancomycin. They recommend 2-stage exchange with postoperative vancomycin.

     

    Buller et al. retrospectively assessed traits that would predict the success of debridement and liner exchange for 62 hips and 247 knees. They found that 149 (48.2%) cases failed to eradicate infection. Risks for recurrent infection include longer symptom, higher ESR, previous PJI or infection in the same joint, and an infection by a group 1 (MRSA, VRE, and methicillin-resistant S. epidermidis) or group 2 (MSSA or methicillin-sensitive coagulase-negative Staphylococcus) organism.

    Della Valle et al. discuss the AAOS recommendations on diagnosis of periprosthetic hip and knee infections. They recommend repeat hip and knee aspirations when there is discrepancy between probability of PJI and initial aspiration culture result.

     

    Aggarwal et al. prospectively compared the yield of intraoperative tissue and swab cultures in 74 hip, 43 knee, 30 septic and 87 aseptic cases. They found that tissue cultures had higher sensitivity, specificity, positive and negative predictive values for identifying PJI. Swab cultures had higher false positive and negative values. They recommend not using swab cultures, and only using tissue cultures.

     

    Figures A and B are AP and lateral radiographs showing areas of bony erosion suggestive of loosening of the femoral and tibial components.

    Illustration A is the 2018 MSIS criteria with point values.

     

    Incorrect Answers:

    Answer 1: MRI is not a recommended imaging modality if infection is suspected. Nuclear imaging (labeled WBC, bone scan, FDG-PET, gallium scan) is recommended instead.

    Answer 2: Arthroscopic debridement is only indicated in the absence of infection e.g. adhesiolysis, patellar clunk.

    Answer 3: Debridement and liner exchange is appropriate for acute PJI but will also not address prosthetic loosening.

    Answer 4: Single-stage revision is not the accepted standard and is not as successful with clearance of pathogens as double-stage revision.

     

     

     

     

     

     

    OrthoCash 2020

  142. What are the affects on limb-length and offset according to the total hip arthroplasty template shown in Figure A?

     

     

     

     

    1. Limb-length will stay the same, offset will be increased

    2. Limb-length will be decreased, offset will be increased

    3. Limb-length will stay the same, offset will be decreased

    4. Limb-length will be increased, offset will be increased

    5. No change in either limb-length or offset Corrent answer: 1

    In Figure A, the center of rotation of the femoral component lies medial to the center of rotation of the acetabular component. If these components are implanted as shown, the offset will be increased and the leg-lengths will remain equal.

     

    Offset and leg-length changes during templating and insertion of a total hip replacement are determined by the changes in the center of rotation (COR) of the femur relative to the acetabulum. If changes are made in the horizontal plane (x-axis), a change in offset will occur. If changes are made in the vertical plane (y-axis), changes in leg-lengths will occur. If the femoral COR is templated superior to the acetabular COR, the leg will be lengthened. In

    contrast, if the femoral COR is templated inferior to the acetabular COR, the leg will be shortened. For offset, the same principles apply. If the femoral COR is templated medial to the acetabular COR, offset will be increased. In contrast, if the femoral COR is templated lateral to the acetabular COR, offset will be decreased. One should aim to restore native offset and leg-lengths in uncomplicated primary total hip arthroplasty.

     

    Merle et al. retrospectively reviewed 152 patients to evaluate femoral offset on an AP pelvis and AP hip radiograph compared to a CT scan of the affected hip. They found that AP pelvis radiograph underestimated femoral offset by 13% when compared to a CT scan. In contrast, the AP hip radiograph showed no difference when compared to the CT scan. They recommend obtaining AP of the hip prior to templating for accurate assessment of femoral offset.

     

    Della Valle et al. review preoperative planning for total hip arthroplasty. While they state that templating has a high predictive value in achieving the desired plan, the surgeon should always be prepared to make intraoperative adjustments based on tactile feedback.

     

    Illustration A shows an example where leg-length will be shortened (femoral COR is inferior to acetabular COR) and offset will stay the same (femoral COR and acetabular COR are in the same horizontal plane). Illustration B is a table which summarizes the points we have discussed.

     

    Incorrect Answers:

    Answer 2: This would be true if the femoral COR was templated medial to the acetabular COR, and if the femoral COR was templated inferior to the acetabular COR.

    Answer 3: This would be true if the femoral COR was templated lateral to the acetabular COR.

    Answer 4: This would be true if femoral COR was templated superior/medial to the acetabular COR.

    Answer 5: This would be true if femoral and acetabular COR were templated at the same levels in both the horizontal and vertical planes.

     

     

     

     

     

     

     

     

    OrthoCash 2020

     

  143. A 65-year-old patient is diagnosed with a periprosthetic joint infection 6 years after total knee arthroplasty. He recalls a history of knee realignment surgery many years prior. Examination reveals lateral patellar tracking and passive flexion to 65 degrees. A recent radiograph is shown in Figure A. During the exposure for explantation, a standard medial parapatellar approach is performed through the previous incision. It is found that adequate knee flexion to allow exposure of the prosthesis cannot be achieved even after release of the lateral gutters and excision of the scar. Which surgical exposure technique (depicted in Figures B through F) would provide the best

    surgical exposure for the procedure and preserve the blood supply to the patella?

     

     

     

     

     

     

     

     

     

     

     

     

     

     

     

     

     

    1. Fig B

    2. Fig C

    3. Fig D

    4. Fig E

    5. Fig F

    Corrent answer: 3

     

    A tibial tubercle osteotomy (TTO) would provide the best surgical exposure without compromising patellar blood supply. This patient has patella baja arising from previous high tibial osteotomy, with a scarred, contracted patellar tendon leading to knee stiffness.

     

    A TTO is able to provide good exposure while protecting the extensor mechanism and preventing inadvertent avulsion of a contracted patellar tendon. Further, proximal transfer of the osteotomized tibial tubercle may be used to correct patella baja, bearing in mind that excessive superior translation alters the mechanics of the knee by making the quadriceps less efficient.

     

    Mendes et al. used TTO for surgical exposure in 67 knees undergoing revision TKA. There were good-excellent knee scores at 30 months in 87%. There were no patellofemoral complications, no component malalignments, and no avulsions of the patellar tendon occurred. They advocate TTO for cases where the patellar cannot be retracted laterally with knee in 90deg of flexion.

     

    Whiteside described a series of TTO in 136 TKA. At 2 years, mean range of motion was 94deg. There were 2 tibial tubercle avulsion fractures and 3 tibial fractures (2 in a patient with Charcot arthropathy, and 1 following manipulation after open adhesiolysis. He advises using stemmed tibial components in patients with insensate knees and in cases where manipulation is expected.

     

    Figure A is a lateral radiograph showing severe patella baja. Figure D shows a TTO. See below for Figures B, C, E and F. Illustration A shows the surgical technique for TTO. The distal saw cut angles out of the anterior cortex at a gentle angle to reduce the stress riser effect and risk of postoperative tibial stress fracture.

     

    Incorrect Answers:

    Answer 1: Figure B shows a quadriceps snip and lateral retinacular release. This technique provides good exposure for most revisions and will allow patella flip, but it does not provide as much exposure as a TTO, and may compromise the lateral genicular artery supply to the patella.

    Answer 2: Figure C shows a patellar turndown (or VY turndown). The incision transects the rectus tendon, the vastus lateralis tendon, and the lateral retinaculum. This exposure provides excellent exposure but may compromise patellar blood supply.

    Answer 4: Figure E shows a quadriceps snip. This technique provides increased exposure over a standard medial parapatellar approach but is unlikely to be

    sufficient in the presence of severe patellar baja and patellar tendon contracture.

    Answer 5: Figure F shows a modified V-Y quadricepsplasty. The incision curves along the edge of the vastus lateralis tendon, avoiding the lateral superior genicular artery. While this technique aims to preserve patellar blood supply, superior and medial branches are divided in the process. It will also not provide as much exposure as a TTO.

     

     

     

     

     

     

     

    OrthoCash 2020

     

  144. Figure A show pre- and post-operative radiographs, from left to right respectively, of a 79-year-old male that underwent revision total hip arthroplasty 2 years ago. He presents today for consultation after 4 episodes of right hip dislocation within the past 6 months. Physical examination reveals a trendelenburg gait with no clinical or radiographic limb length discrepancy. An Infection work-up is negative. Results from a CT scan are shown in Figure B. What would be the best treatment option?

     

     

     

     

     

     

    1. Physiotherapy and application of abductor brace

    2. Revision arthroplasty to medialize the cementless cup and surgical repair of the abductor tendon

    3. Revision arthroplasty to a constrained polyethylene liner

    4. Revision arthroplasty to a femoral component with extended offset

    5. Revision arthroplasty to a large ceramic femoral head and offset polyethylene cup

    Corrent answer: 3

     

    On the left, Figure A shows a metal-on-metal (MOM) bearing hip resurfacing. On the right, Figure A shows a large head, uncemented metal-on-polyethylene (MOP) total hip replacement. In this setting, the most appropriate treatment option would be revision arthroplasty with constrained polyethylene liner.

     

    Constrained liners should be reserved for patients demonstrating recurrent instability despite treatment with a large femoral head. Other indications include elderly patients who do not require implant longevity or have a low functional demand, as well as patients with deficient or non-repairable abductor mechanisms.

     

    Sikes et al. report on the results of a series of 41 patients (52 hips) with recurrent dislocations. They recommend that large femoral heads (LFH) be used as a first-line treatment in high-risk patients (patients of any age with dementia, neuromuscular disability, and inability to comply with precautions). Constrained liners should be reserved for patients demonstrating recurrent

    instability despite treatment with an LFH.

     

    Kilampali et al. reviewed late instability of bilateral metal on metal hip resurfacings. They suggest that late instability of hip resurfacing should raise concerns relating to possible local tissue reaction and muscle damage.

    Concerning features include steeply-inclined acetabular components a large abduction angle of more than 55 degrees along with a combination of small size component.

     

    Figure A shows an image of a revised socket which was performed to convert the MOM THA to a MOP THA. Figure B shows normal parameters of THA components. The recommendation for acetabular position is anteversion 20° ± 10° and abduction 45° ± 10°. For the femur, recommendations are 10°- 15° of anteversion and 41mm - 45mm of offset.

     

    Incorrect Answers:

    Answer 1: Conservative treatment would be indicated in patients not suitable for operative intervention.

    Answer 2: Medializing the cup would likely increase the potential for dislocation.

    Answer 4: Revision arthroplasty to a femoral component with extended offset would help to decrease joint reaction forces. However, this patient has deficient abductors, which is likely related to local tissue reaction and muscle damage from the metal on metal implant.

    Answer 5: A large ceramic femoral head and offset polyethylene cup would not help to restore stability.

     

     

     

    OrthoCash 2020

     

  145. Which of the following has been shown to increase the rate of failure of cemented femoral components in total hip arthroplasty?

    1. Stems that are precoated with polymethylmethacrylate

    2. Calcar contact of the collar

    3. Smoother implant corners

    4. Cement mantle of 2 millimeters

    5. Stem material with a Young's modulus higher than 115 GPa Corrent answer: 1

    Precoating a stem with PMMA adds an additional inferface at risk of failure.

     

    Stiffer stem materials (higher Young's modulus) improve performance. Titanium has a Young's modulus of 115 GPa with alloy and stainless steel

    having a higher Young's modulus than titanium. Calcar collar contact adds minimal strength to the construct, but does not lead to premature failure. Smoother corners decrease the rate of failure since they decrease stress risers. The ideal cement mantle is ~2mm. Obtaining less than this would decrease the strength of the construct.

     

     

     

    OrthoCash 2020

     

  146. An 80-year-old male sustains a fall down the stairs and presents with knee swelling. He is a community ambulator who does not use walking aids. Injury radiographs are shown in Figures A and B. What is the next best step?

     

     

     

     

     

    1. Intramedullary nailing

    2. Locked plating

    3. Long leg casting

    4. External fixation

    5. Revision total knee arthroplasty Corrent answer: 5

    This patient sustained a periprosthetic femoral fracture around the femoral component which is now loose. Revision of the femoral component is necessary.

     

    Various classifications exist for periprosthetic fractures around TKA. In general, for the femoral component, treatment depends on fracture displacement, fracture location, bone stock, and whether the component is loose. For loose femoral components, revision TKA using distal femoral replacement prosthesis is an option.

     

    Kim et al. proposed a new classification for periprosthetic fractures. Type IA fractures (good bone stock, well fixed, nondisplaced or easily reducible) are managed conservatively. Type IB fractures (good bone stock, well fixed, irreducible closed) are managed with reduction and fixation. Type II fractures (good bone stock, reducible, loose or malpositioned components) are managed with revision. Type III fractures (poor bone stock, loose or malpositioned components) are treated with distal femoral replacement.

     

    Johnston et al. reviewed the options for treating periprosthetic fractures about the knee. They advocate revision of the femoral component when the prosthesis is loose, where there is poor bone stock, or insufficient bone to gain purchase for locked plates or distal locking screws of intramedullary nails.

     

    Nauth et al. review the current concepts in treatment of periprosthetic fractures. They prefer minimally invasive locked plating unless the fracture is significantly proximal to the anterior flange and amenable to retrograde intramedullary nailing. Then they choose nails with options for distal interlocking screws and locking condylar bolts. In extreme osteopenia, they use intramedullary fibular strut allografts (with locked plating). For loose prostheses or poor bone stock, they perform alloprosthetic composite in younger patients and a distal femoral replacement in elderly patients.

     

    Figures A and B are AP and lateral radiographs showing periprosthetic femoral fracture around a loose femoral component. Illustrations A and B are postoperative radiographs showing revision to a hinged prosthesis with long-stemmed components. Illustration C shows Kim' proposed classification of

    periprosthetic fractures around the femoral component of a TKA.

     

    Incorrect Answers:

    Answers 1, 2, 3, 4: A loose femoral component requires revision.

     

     

     

     

     

     

     

     

     

     

    OrthoCash 2020

     

  147. Utility of the implant seen in Figure A would be best considered in which of the following revision total hip arthroplasty scenarios?

     

     

     

     

    1. Minimal acetabular deformity, intact rim

    2. Superior acetabular bone lysis with intact superior rim

    3. Localized acetabular destruction of medial wall

    4. Absent superior acetabular rim, superolateral migration

    5. Significant acetabular bone loss, pelvic discontinuity Corrent answer: 5

    Paprosky Type 3B acetabular bone defects describes significant acetabular bone loss, with pelvic discontinuity. Type 3 defects often require reconstruction cages (as seen in Figure A) or acetabular distraction techniques

    to treat severe bone loss with an associated pelvic discontinuity.

     

    Deficient acetabular bone stock poses a technical challenge in hip arthroplasty surgery. Paprosky classification for acetabular bone loss to helps guide treatment for revision total hip arthroplasty. The classification is as follows:

     

    Type 1: Minimal deformity, intact rim

    Type 2A: Superior bone lysis with intact superior rim Type 2B: Absent superior rim, superolateral migration Type 2C: Localized destruction of medial wall

    Type 3A: Significant bone loss, superolateral cup migration Type 3B: Significant bone loss, pelvic discontinuity

     

    Sheth et al. reviewed acetabular bone loss in revision total hip arthroplasty. They state that Paprosky Type 1 and 2 defects can usually be managed with porous-coated hemisphere cup secured with screws. Type 3 defects require reconstruction cages to protect with cups and structural augments or custom triflange implants.

     

    Taunton et al. investigated clinical outcomes and cost-effectiveness of using a custom triflange acetabular component to treat pelvic discontinuity in revision THA. They found satisfactory clinical outcomes (81% had a stable triflange component with healed pelvic discontinuity) and cost equivalence with Trabecular Metal cup-cage constructs.

     

    Figure A shows a lateral image of the pelvis with a reconstruction cage and cup construct. Illustration A shows an illustration of the Paprosky classification. Illustration B shows a table of the Saleh/Gross classification. Illustration C shows a table of the AAOS classification.

     

    Incorrect Answers:

    Answer 1: Minimal deformity, intact rim = Paprosky Type 1 defects. These can be treated with porous-coated hemisphere cup secured with screws.

    Answer 2: Superior bone lysis with intact superior rim = Paprosky Type 2A defect. This can be treated with porous-coated hemisphere cup secured with screws.

    Answer 3: Localized acetabular destruction of medial wall = Paprosky Type 2C defect. This can be treated with porous-coated hemisphere cup secured with screws +/- bone grafting.

    Answer 4: Absent superior acetabular rim, superolateral migration = Paprosky Type 2B defect. This can be treated with porous-coated hemisphere cup secured with screws, jumbo cups +/- metal augments +/- bone grafting.

     

     

     

     

     

     

     

     

     

     

     

    OrthoCash 2020

     

  148. A 65-year-old male who had a total knee arthroplasty 8 years ago comes into the office with worsening knee pain. The orthopaedic surgeon is concerned about infection and aspirates the knee. Which of the following are the lowest laboratory values from a synovial aspirate suggestive of infection?

    1. WBC of 500 cells/ml and PMN 25%

    2. WBC of 1,000 cells/ml and PMN 25%

    3. WBC of 1,500 cells/ml and PMN 70%

    4. WBC of 5,000 cells/ml and PMN 70%

    5. WBC of 25,000 cells/ml and PMN 70%

     

    Corrent answer: 3

     

    WBC of 1,500 cells/ml and PMN 70% indicates the lowest synovial aspirate suggestive of infection.

     

    Mason et al in 2003 reviewed 440 revision TKA's of which 86 had preoperative aspirations. The aspirations yield 55 aseptic failures and 31 septic failures. The mean WBC of the aseptic group was 645 cells/mm(3) compared to 25,951 cells/mm(3) for the septic group (P=<.001). The mean percentage of polymorphonuclear cells (PMNs) was statistically higher in the septic group compared with the aseptic group (72.8% vs 27.3%; P=<.001). With these results, the authors concluded that aspirates with a WBC count greater than 2,500 and 60% PMNs are highly suggestive of infection.

     

    However, in a more recent and larger study, Ghanem et al reviewed 161 infected TKA's vs 268 aseptic failures and concluded that aspiration with WBC of >1100 cells/mm3 and PMN > 64% are suggestive of infection. When both tests yielded results below their cutoff values, the negative predictive value was 98.2% (95% confidence interval, 95.5% to 99.5%), whereas when both tests yielded results greater, infection was confirmed in 98.6% (95% confidence interval, 94.9% to 99.8%) of the cases. Thus, according to the most recent literature, WBC >1100 and PMN > 64% should be considered suggestive of infection in a TKA.

     

     

     

    OrthoCash 2020

     

  149. A 50-year-old man with a past medical history significant for diabetes and end-stage renal disease presents with a chief complaint of instability 6-months following a total knee arthroplasty. Preoperative radiographs are shown in Figures A-C. Physical exam at that time was notable for a large effusion, maltracking patella, extensor lag of 15 degrees, medial instability, and gross laxity to anterior and posterior forces. The procedure was uncomplicated, and was completed using a posterior-stabilized prosthesis with tibial augements and uncemented intramedullary rods in both the femur and tibia. Which of the following surgical techniques should have been implemented to avoid this complication?

     

     

     

     

     

     

     

     

    1. Cementing the intramedullary rods in the tibia and femur

    2. Explant with placement of an antibiotic spacer

    3. Taking 5mm of extra bone from the distal femur to elevate the joint line

    4. Use of a hinged total knee arthroplasty

    5. Taking 5mm of extra bone from the tibia to distalize the joint line Corrent answer: 4

    The patient has a neuropathic joint with ligamentous instability and a maltracking patella. The appropriate procedure would have included use of a hinged total knee arthroplasty.

     

    Choosing the appropriate constraint during a total knee arthroplasty ensures the best possible outcome. Hinged total knee arthroplasty prostheses are indicated in the setting of global instability, massive bone loss in a neuropathic joint, oncologic procedures, and hyperextension instability. In a hinged prosthesis, the tibial and femoral components are linked with an axle that restricts varus/valgus and translational stresses. While hinged prostheses are useful in the setting of major revision surgery, they are at increased risk for aseptic loosening due to the high degree of constraint inherent to the device.

     

    Petrou et al. review the results of 100 primary cemented rotating-hinge total knee arthroplasty at 7- to 15-years. At 15 years, survival was 96.1%.

    Complications included DVT (n=3), skin necrosis (n=2), subcutaneous hematoma (n=5), intra-operative fracture of either the femur or tibia (n=4), and early infection (n=2).

     

    Figures A-C show a neuropathic joint with considerable lateral bone loss and a frankly dislocated patella. Illustration A shows an example of a hinged total knee arthroplasty. Note how the tibial and femoral components are linked using an axle.

    Incorrect Answers:

    Answer 1: Cementing the intramedullary rods would not have increased the amount of device constraint.

    Answer 2: There were no concerns for infection based on the information given in the question stem, and instability is unlikely to be the primary complaint in an infected prosthesis.

    Answer 3: Elevating the joint line would not have improved the sensation of post-operative instability.

    Answer 5: Distalizing the joint line would not have improved the sensation of post-operative instability.

     

     

     

     

     

     

    OrthoCash 2020

     

  150. A 63-year-old man returns for follow-up 4 years after metal-on-metal left total hip arthroplasty complaining of mild chronic hip pain with ambulation. He is afebrile and ESR and CRP are within normal limits. Radiograph of the left hip is shown in Figure A. What is the best next step?

     

     

     

    1. Anti-inflammatory medication

    2. Serum cobalt and chromium levels

    3. MRI with metal subtraction

    4. Physical therapy

    5. Revision hip arthroplasty Corrent answer: 2

    Metal-on-metal total hip arthroplasties (THA) have been associated with complications presumably due to metal debris and toxicity. Serum cobalt and chromium levels are recommended as part of follow-up evaluation for patients with metal-on-metal hips, even when asymptomatic.

     

    Many patients with metal-on-metal hips have been found to have elevated serum cobalt and chromium levels, for which MR with metal subtraction is recommended to look for pseudotumors and other pathologies. These solid or cystic masses are thought to be related to metal debris and macrophage infiltration and may be associated with pain in some patients.

     

    Lombardi et al summarize and present on behalf of The Hip Society an algorithmic approach to evaluating and treating patients with metal-on-metal THA in follow-up. They state the goals of care as determining the etiology of any pain, managing any intrinsic problems with the arthroplasty, and reassuring/observing when appropriate. They organize the types of patients seen in followup and components of the evaluation.

     

    Chang et al evaluate the correlation between symptoms and MRI findings and report that symptomatic patients tend to have bone marrow edema and tendon tearing on MRI. They report a 69% prevalence of pseudotumors on MRI after metal-on-metal hip arthroplasty, but did not find a correlation between pseudotumor presence and pain.

    Hayter et al focus on MRI findings in symptomatic (painful) patients with metal-on-metal THA in a review including 31 hip resurfacing and 29 THA. In the THA group, they report 86% rate of synovitis, 10% extracapsular disease, and 24% osteolysis, with no statistically significant difference in rates between resurfacing and THA.

     

    Figure A is an AP view radiograph of a left hip after metal-on-metal total hip arthroplasty with components well positioned and no osteolysis.

     

    Illustrations A and B from Lombardi et al depict a recommended algorithm for the workup and management of symptomatic and asymptomatic patients, respectively, with metal-on-metal THA.

     

    Incorrect Answers:

    Answer 1: Anti-inflammatory medication can be a treatment for pain but the best next step is to continue the diagnostic workup.

    Answer 3: MRI with metal subtraction should be ordered for patients with elevated serum metal levels or to work up persistent pain after metal levels are checked.

    Answer 4: Physical therapy can be a reasonable treatment option after a full diagnostic workup is complete.

    Answer 5: Workup with metal levels should be completed before considering any revision or invasive treatment.

     

     

     

     

     

     

     

     

    OrthoCash 2020

     

  151. A 72-year-old woman sustains a fall onto her knee three years after an uncomplicated total knee replacement. The fracture pattern is seen in Figure A. The operative note reveals that a cemented patellar component was used. On exam, she has a large effusion and an

    inability to straight leg raise. If the patellar component is well fixed, what is the best treatment option?

     

     

     

     

    1. Patellectomy

    2. Extensor mechanism allograft

    3. Revision of the patellar component with cement and bone grafting of any residual defect

    4. Open reduction and internal fixation of the patella fracture

    5. Non-operative treatment in a knee brace locked in extension for 6 weeks Corrent answer: 4

    Displaced, periprosthetic patella fractures with a deficient extensor mechanism and adequate bone stock are best treated with open reduction and suture or implant fixation.

     

    Periprosthetic patella fractures are a rare, but potentially devastating complication associated with total knee arthroplasty. When evaluating patella fractures, it is important to consider 1) is the extensor mechanism intact, 2) is the patellar component well fixed or loose, and 3) is there sufficient bone stock remaining. Stable implants with an intact extensor mechanism should almost exclusively be treated non-operatively in a brace. In contrast, a deficient

    extensor mechanism is an absolute indication for surgical management.

     

    Adigweme et al. review the epidemiology, diagnosis, and treatment of periprosthetic patella fractures. When analyzing patella fractures, they suggest treatment should be based on fracture severity, remaining bone stock, patellar component stability, as well as extensor mechanism function.

     

    Sarmah et al. review periprosthetic fracture around total knee arthroplasty. They provide an algorithm for treatment of periprosthetic patella fractures based on displacement, viability of remaining bone stock, and fracture type.

     

    Figure A is a preoperative lateral radiograph showing a periprosthetic patellar fracture. The distal fragment is comminuted and separated from the proximal fragment by approximately 15 mm. The patellar component appears to be well fixed. Illustration A is intraoperative photograph showing the threads of the suture anchors in the proximal fragment passing through the tunnels in the distal fragment and exiting at the inferior pole of the patella. Illustration B demonstrates anatomical reduction after the knots were tied at the inferior pole of the patella. Illustration C is a lateral x-ray 1 year postoperatively showing fracture union.

     

    Incorrect Answers:

    Answer 1: Patellectomy is reserved for cases where patellar bone stock is insufficient.

    Answer 2: Extensor mechanism allograft has similar indications to a patellectomy. Indications include a deficient extensor mechanism and poor patellar bone stock.

    Answer 3: In this situation, the patellar component is not loose. Therefore, it does not need to be revised.

    Answer 5: Non-operative treatment should only be considered if the extensor mechanism is intact.

     

     

     

     

     

     

     

     

     

     

    OrthoCash 2020

     

  152. Knee pain and osteoarthritis are associated with "metabolic syndrome." All of the following are included in the collection of risk factors known as "metabolic syndrome" EXCEPT:

    1. Peripheral vascular disease

    2. Dyslipidemia

    3. Hypertension

    4. Impaired glucose tolerance

    5. Central obesity

       

      Corrent answer: 1

       

      Peripheral vascular disease (PVD) may develop in patients with metabolic syndrome. However, no direct relationship between metabolic syndrome and PVD is known, and it is not a part of metabolic syndrome itself. Metabolic syndrome has been shown to be associated with knee pain and development of knee osteoarthritis (OA).

       

      Metabolic syndrome is a collection of medical comorbidities that are known to

      be risk factors for developing cardiovascular disease. Metabolic syndrome includes central (abdominal) obesity, dyslipidemia (high triglycerides and low-density lipoproteins), high blood pressure, and elevated fasting glucose levels. There is an increased prevalence of knee pain (and OA) among patients with metabolic syndrome. It is felt that the most important contributing factor to knee pain and OA in metabolic syndrome is obesity. Patients presenting with knee pain or OA and the risk factors included in metabolic syndrome should be counseled on the need to control those risk factors.

       

      Inoue et al. present a study comparing metabolic syndrome and knee OA in a Japanese population. They found that knee OA and metabolic syndrome were highly correlated in females, but not in males.

       

      Engström et al. present a study comparing metabolic syndrome with hip and knee OA. They found no relationship to hip OA, but did find a strong correlation between patients with metabolic syndrome and risk of developing knee arthritis. Patient BMI was the most predictive factor. They also compared prevalence of knee OA to CRP levels, but found no significant relationship.

       

      Incorrect answers:

      Answers 2, 3, 4, and 5: These represent the collection of risk factors known as "metabolic syndrome."

       

       

       

      OrthoCash 2020

       

  153. A 75-year-old male presents with recurrent dislocations of this left hip. He underwent bilateral total hip arthroplasties 12 and 8 years ago. There were no early post-operative complications with either hip. Despite a total of 5 dislocations in 6 months, he does not have pain or weakness across the left hip. On examination, there is a healthy appearing left lateral scar, equal limb lengths, normal gait and full abductor strength. Radiographs of the pelvis are shown in Figure A. His laboratory results show an erythrocyte sedimentation rate of 8 mm/h (reference range, 0-20 mm/h), and C-reactive protein of 3 mg/L (reference range, 0-5.0 mg/L). A hip aspirate culture is negative. What is the next best management option for this patient?

     

     

     

    1. Magnetic resonance imaging of left hip to exclude an abductor muscle tear

    2. Re-aspiration of left hip to exclude a subclinical infection

    3. Continued observation for trochanteric bursitis

    4. Supervised physiotherapy and gait training for abductor strengthening

    5. Left revision total hip arthroplasty for polyethylene wear Corrent answer: 5

    This patient presents with recurrent late hip instability with radiographic evidence of eccentric polyethylene wear. The best treatment option for this patient would be revision total hip arthroplasty (THA).

     

    The etiology of late instability includes polyethylene wear, component malpositioning or loosening, trauma, infection or deterioration in neurological status of the patient. Identifying the cause of late instability will require a thorough work up. A good history, examination and scrutiny of radiographs can identify most causes. Advanced imaging may be requires when bone or soft-tissue pathology is suspected or radiographic evidence of osteolysis or malpositioning needs further assessment. Blood work to assess for an acute inflammatory response (ESR and CRP) should be ordered routinely as elevated markers may indicate an underlying infection.

     

    Parvizi et al. evaluated the outcome of revision arthroplasty for polyethylene wear presenting as late dislocation. They found that revision surgery restored stability to eighteen of the twenty-two patients. Surgical treatment options may include liner-only exchange (contained or unconstrained) +/- soft-tissue repair, or revision of one or all components.

    Berry et al. evaluated the long-term risk of dislocation in 6,623 consecutive primary total hip arthroplasties with a Charnley prosthesis. They found a 7% incidence of late dislocation at 25 years compared to 1% after 5 years.

    Patients at highest risk were females, patients with osteonecrosis of the femoral head or an acute fracture, and nonunion of the proximal part of the femur.

     

    Figure A shows an AP pelvis with bilateral, uncemented, total hip arthroplasties. There is eccentric wear of the left acetabular component. No fracture or loosening of the components can be identified. The components appear well-positioned.

     

    Incorrect Answers:

    Answers 1: Magnetic resonance imaging is effective for the assessment of the periprosthetic soft tissues in patients who have had a total hip arthroplasty.

    This patient has no pain or weakness in the affected hip. Therefore, soft tissues can be evaluated intra-operatively during the revision THA procedure. Answer 2: A hip aspirate would not be warranted. There are no risk factors for infection in this patient (for example, no pain, no early wound complications or antibiotics, etc). Additionally, his inflammatory markers are normal.

    Answer 3: Continued observation can be considered, but recurrent dislocations in the setting of polyethylene wear would be considered an indication for surgery.

    Answer 4: Supervised physiotherapy would be considered in a patient with clinical evidence of weakness in the setting of an initial dislocation.

     

     

     

    OrthoCash 2020

     

  154. A 58-year-old woman undergoes a total knee arthroplasty with a posterior stabilized design. Two years later, she returns with recurrent sterile joint effusions, a sensation of instability without giving way and difficulty with ascending and descending stairs. Examination reveals diffuse tenderness around the pes anserinus and peripatellar region, and increased anterior tibial translation most notable at 90° of flexion. Radiographs demonstrate well cemented implants with 5° of posterior tibial slope. Figure A represents a femoral cutting block with lines 1 through 5 corresponding to femoral bone cuts. The most likely cause of her symptoms is over-resection at:

     

     

     

    1. Resection line 1

    2. Resection line 2

    3. Resection line 3

    4. Resection line 4

    5. Resection line 5

     

    Corrent answer: 2

     

    Over-resection of the posterior femoral condyles (resection line 2) in posterior-stabilized (PS) TKA leads to flexion instability without frank dislocation.

     

    There are 7 bone cuts in a total knee replacement. The posterior condylar cut determines the flexion gap. Flexion instability in PS knees arises because of an enlarged flexion gap (excessive posterior condylar resection, or increased tibial slope), allowing anterior tibial translation, which is pathognomonic. There will not be posterior subluxation because of the cam-post design. Symptoms include sensation of instability without giving way, especially with stair climbing, recurrent knee effusions, and diffuse knee pain. Signs include anterior tibial translation at 90° flexion, tenderness at multiple sites (including pes anserinus, peripatellar, posterior hamstrings), and effusion. Revision surgery is indicated for symptomatic patients.

     

    Clarke et al. reviewed flexion instability after primary TKA. They caution that most cases arise from failure to create symmetric balanced flexion and extension spaces. Treatment is usually revision TKA using the same principles. If this is not possible, increased constraint is required (constrained condylar prosthesis or hinged prosthesis).

    Schwab et al. reviewed flexion instability without dislocation in PS knees in 10 patients. Revision surgery focused on flexion-extension gap balancing and filling the enlarged flexion gaps and successfully relieved pain, and improved stability to anterior tibial translation. Flexion space reconstruction includes using a larger femoral component or posterior augments. Isolated polyethylene exchange is not recommended.

     

    Figure A shows a 5-in-1 cutting block with anterior femoral cut (line 1), posterior femoral cut (line 2), posterior chamfer cut (line 3), anterior chamfer cut (line 4), and distal femoral cut (line 5). Of note, most TKA systems have a 4-in-1 cutting block and the distal femoral cut is made separately. Illustration A shows restoration of the posterior condylar offset (line A) with the femoral component (line B).

     

    Incorrect Answers

    Answer 1: The anterior femoral cut does not affect the flexion gap. Underresection leads to oversizing of the femoral component and patellofemoral stuffing (leading to patellar maltracking and reduced flexion). Over-resection leads to notching. A good cut looks like a grand piano.

    Answer 3: The posterior chamfer cut does not affect the flexion gap. The posterior and anterior chamfer cut are essential for the prosthesis to fit over the distal femur.

    Answer 4: The anterior chamfer cut does not affect the flexion gap.

    Answer 5: The distal femoral cut does not affect the flexion gap. The amount of bone resected should be equal to the thickness of the femoral component. This cut sets the extension gap. Additional bone may be resected to correct a flexion contracture.

     

     

     

     

     

     

     

    OrthoCash 2020

     

  155. Which of the following is true regarding intra-operative fractures during total knee arthroplasty?

    1. They occur more commonly in cruciate-retaining total knee replacements

    2. Fractures of the medial femoral condyle are the most common fracture type

    3. Fractures of the patella are the most common fracture type

    4. Most can be treated without additional fixation at the time of surgery

    5. Tibial fractures are more common than femoral fractures Corrent answer: 2

    Fractures of the medial femoral condyle are the most common type of intraoperative fracture during a total knee arthroplasty.

     

    Intra-operative fractures during total knee replacement are rare, but usually requiring alterations in surgical technique once they occur. The most common time for fractures to occur is during exposure and bone preparation, with fracture during trialing being the next most common. Fractures occur more commonly in posterior cruciate substituting designs, likely due the box cut.

    Osteoporosis, female gender, chronic steroid use, advanced age, rheumatoid arthritis, and neurologic disorders are risk factors for post-operative fracture, but are also thought to be risk factors for intra-operative fractures.

     

    Alden et al. reviewed 17,389 primary TKAs and found an intra-operative fracture rate of 0.39%. Of the 67 fractures, 49 were femur fractures, 18 were tibia fractures, and none were patella fractures. They recommend careful surgical technique in patients at high risk for fracture to avoid such a complication.

     

    Sharkey et al. reviewed 10 intra-operative femoral fractures during primary, cementless total hip arthroplasty. They matched these with 20 patients who did not have this complication. At follow-up, there were no differences found between the two groups.

     

    Incorrect Answers:

    Answer 1: Fractures occur more commonly in cruciate-substituting total knee replacements due to the box cut.

    Answer 3: Intra-operative fractures of the patella are quite rare. In the series reported above they had no instances of patella fracture.

    Answer 4: Most fractures require treatment consisting of a wires, screws, and/or plates.

    Answer 5: Femoral fractures are more common than tibial fractures during total knee arthroplasty.

     

     

     

    OrthoCash 2020

     

  156. A 68-year-old male complains of increasing medial sided knee pain and buckling. The pain is exacerbated by sharp turns while

    running. He undergoes knee arthroscopy. Recent radiographs and an arthroscopic photograph of the medial compartment are shown in Figure A. His pain has worsened since the arthroscopy. Which of the following images (Figures B through F) represents the best treatment recommendation for this patient?

     

     

     

     

     

     

     

     

     

     

     

     

     

     

     

     

     

     

    1. Figure B

    2. Figure C

    3. Figure D

    4. Figure E

    5. Figure F

     

    Corrent answer: 4

     

    This patient has isolated medial compartment osteoarthritis with Outerbridge IV medial compartment cartilage wear on arthroscopy. The best surgical option is a medial unicompartmental knee arthroplasty (UKA).

     

    Indications for UKA include range of motion >100deg with <5deg flexion contracture, angular deformity <15deg valgus or <5deg varus that is passively correctable to neutral. Relative contra-indications include younger patients (age <60), obesity (BMI >30), and ACL deficiency in medial UKA. Asymptomatic patellofemoral chondromalacia is not a contraindication. In general, a UKA is preferred for older, less active patients with minimal varus, more severe arthritis, and no/little knee instability. A HTO is preferred for younger, active patients, with milder arthritis, more malalignment, and AP instability.

     

    Steadman et al. retrospectively examined outcomes of TKA after arthroscopic treatment of OA in 73 patients. They found that mean survival time (conversion to TKA) after arthroscopy was 6.8 years (5.7 years in patients with Kellgren-Lawrence grade 4, and 7.5 years in those with grade 3). They conclude that in patients who want to avoid TKA, arthroscopy may help postpone TKA.

     

    LaPrade et al. examined the results of proximal tibial opening wedge

    osteotomies in 47 patients <55 years old with medial compartment osteoarthritis (OA) and genu varus. They found that there was improvement in knee scores and the mechanical axis at 3.6 years. They recommend this technique for patients with medial compartment OA and malalignment.

     

    Figure A is a composite image showing isolated medial compartment osteoarthritis, and Outerbridge IV cartilage wear with large areas of visible subchondral bone. (See below for Figures B through F). Illustration A is a table comparing the indications of HTO vs UKA.

     

    Incorrect Answers:

    Answer 1: Figure B shows a valgus producing medial opening wedge high tibial osteotomy. This is appropriate for isolated mild medial compartment arthritis with more malalignment in younger, more active patients.

    Answer 2: Figure C shows bicompartmental arthroplasty of the medial tibiofemoral and patellofemoral compartments. This is appropriate for medial tibiofemoral and patellofemoral compartment arthritis with sparing of the lateral tibiofemoral compartment.

    Answer 3: Figure D shows patellofemoral arthroplasty. This is appropriate for isolated patellofemoral compartment arthritis.

    Answer 5: Figure F shows lateral UKA. This is appropriate for isolated lateral compartment arthritis.

     

     

     

     

     

     

    OrthoCash 2020

     

  157. A 58-year-old female, with a BMI of 34 kg/m2, underwent a total knee arthroplasty for osteoarthritis 6 weeks ago. She has been participating in supervised rehabilitation since the procedure. Her preoperative, intra-operative and 6 week post-operative knee flexion are shown in Figure A. Current radiographs are shown in Figure B. What is the best step in management?

     

     

     

     

     

     

    1. Convert to a resurfaced patella

    2. Downsize the polyethylene liner

    3. Arthroscopic lysis of adhesions and release of posterior capsule

    4. Continuous passive motion at home for two weeks

    5. Manipulation under anesthesia Corrent answer: 5

    This patient has early post-operative stiffness after total knee arthroplasty (TKA). The next best step would be manipulation under anesthesia.

     

    Management of stiffness following TKA can be challenging. The standard initial treatment option for post-operative knee stiffness is physical therapy. When this fails to achieve knee range of motion (ROM) greater than or equal to 90°, alternative treatment modalities should be considered, such as knee manipulation under anesthesia (MUA). MUA is a non-invasive treatment shown to achieve dramatic improvement in knee flexion during the early postoperative period (usually considered less than three months). Periprosthetic fracture during manipulation is rare, with an overall incidence less than 1%.

     

    Issa et al. examined a cohort of patients that underwent MUA after TKA. At a mean follow-up of 51 months (range, 24 to 85 months), the mean gain in flexion in the MUA cohort was 33° (range, 5° to 65°). There was one periprosthetic fracture in 134 patients. The authors noted a significant improvement in ROM from pre-manipulation values.

    Manrique et al. reviewed stiffness after total knee arthroplasty. MUA may be considered within the first three months after the index TKA if physical therapy fails to improve the ROM. Beyond this point, consideration should be given to surgical intervention such as lysis of adhesions, either arthroscopic or open.

     

    Maniar et al. looked at the effectiveness of continuous passive motion immediately after TKA. A total of 84 patients were allocated to no CPM; 1 day CPM; or 3 day CPM. They found that continuous passive motion immediately after TKA did not improve short or mid-term knee ROM.

     

    Figure B shows a cruciate sacrificing total knee arthroplasty with implants in a good position.

     

    Incorrect Answers:

    Answer 1: Indications for patellar resurfacing include inflammatory arthritis, patellar mal-tracking and patellofemoral arthritis (as the main generator of pain). Knee stiffness is not an indication for patellar resurfacing.

    Answer 2: Downsizing the polyethylene spacer would not be indicated at this stage. Downsizing the polyethylene liner would increase the flexion and extension gaps. Because the patient had excellent motion intraoperatively, this outcome is unlikely to be related to the spacer size.

    Answer 3: Arthroscopic lysis of adhesions and release of the posterior capsule would be considered after three months if there was persistent knee stiffness after MUA.

    Answer 4: There is currently no high level evidence to suggest CPM as an effective treatment for arthrofibrosis post TKA.

     

     

     

    OrthoCash 2020

     

  158. Which of the following fracture patterns (Figures A-E) would require revision of the femoral component to a long-stemmed, cementless prosthesis?

     

     

     

     

     

     

     

     

     

     

    1. Figure A

    2. Figure B

    3. Figure C

    4. Figure D

    5. Figure E

     

    Corrent answer: 3

     

    Figure C depicts a Vancouver B2 periprosthetic fracture, which is optimally treated with a long-stem, fully porous-coated, revision femoral prosthesis.

     

    The Vancouver classification for total hip periprosthetic femoral fractures takes into account the three most important factors in management of these injuries: the site of the fracture, the stability of the femoral component, and the quality of the surrounding femoral bone stock. Type A fractures include those involving the lesser trochanter or the greater trochanter. Type B fractures occur around the stem or just below it. More specifically, B1 fractures have a well fixed stem, B2 fractures have a loose stem but good proximal bone stock and B3 fractures have a loose stem with proximal bone that is of poor quality or severely comminuted. Type C fractures are well below the tip of the femoral stem.

     

    O'Shea et al. assessed the outcome of patients with Vancouver type B2 and B3 periprosthetic fractures treated with femoral revision using an uncemented extensively porous-coated implant. Union of the fracture was successfully achieved in 20 of the 22 patients. Overall, they found good early survival rates and a low incidence of nonunion using this implant.

     

    Figure A depicts a radiograph of a Vancouver type C periprosthetic femur fracture, occurring distal to the stem of the total hip arthroplasty. Figure B demonstrates a Vancouver type A periprosthetic fracture of the greater trochanter. Figure C is an x-ray of a Vancouver type B2 periprosthetic fracture adjacent to the stem with an unstable implant, but adequate bone stock.

    Figure D depicts a radiograph of a Vancouver type C periprosthetic femur fracture, occurring distal to the stem of the total hip arthroplasty. Figure E is a Vancouver type B1 periprosthetic fracture at the level of the stem that is well fixed. Illustration A shows a table summarizing the Vancouver classification of periprosthetic femur fractures and the corresponding management options.

     

    Incorrect Answers:

    Answer 1: Vancouver type C fractures are best treated with ORIF using a plate.

    Answer 2: Vancouver type A (GT) fractures are typically managed using cerclage wiring or trochanteric claw plating, if displaced

    Answer 4: Vancouver type C fractures are best treated with ORIF using a plate.

    Answer 5: Vancouver type B1 fractures, are managed by ORIF using cerclage cables and locking plates.

     

     

     

     

     

     

    OrthoCash 2020

     

  159. Which of the following maneuvers places the obturator artery at greatest risk during a total hip arthroplasty?

    1. Placement of a posterior retractor along the posterior wall

    2. Placement of an acetabular screw in the posterior-superior quadrant

    3. Placement of an inferior retractor under the transverse acetabular ligament

    4. Placement of an acetabular screw in the anterior-superior quadrant

    5. Placement of an anterior retractor along the anterior wall Corrent answer: 3

    Damage to the obturator artery most commonly occurs from placement of an inferior retractor inferior to the transverse acetabular ligament (into the obtrator foramen), and/or placement of an acetabular screw in the anterior-inferior quadrant.

     

    Vascular injury during total hip arthroplasty is a rare but devastating complication with a reported incidence of 0.1%-0.2%. The obturator artery travels along the quadrilateral surface of the acetabulum and exits the pelvis at the superolateral corner of the obturator foramen. If the vessel is severely

    damaged and bleeding cannot be controlled, ligation of the internal iliac artery has been reported.

     

    Nachbur et al. report on 15 cases of severe arterial injury during hip reconstructive surgery over a period of 8 years. The most common injury was injury to the external iliac artery, the common femoral artery, or main branches of the lateral and medial circumflex femoral artery. These were thought to be caused by the tip of a narrow-pointed Hohmann retractor used for exposure of the hip joint.

     

    Rue et al. review neurovascular injuries during total hip arthroplasty. Among other things, they recommend against placement of screws in the anterior-superior quadrant, prudent retractor placement, and avoiding excessive tension on the sciatic nerve.

     

    Della Valle and Di Cesare review complications resulting from total hip replacement. They state that injury to the obturator artery can occur with acetabular screw fixation in the antero-inferior quadrant or from retractors placed underneath the transverse acetabular ligament.

     

    Illustration A shows the obturator artery as it exits the pelvis at the superolateral corner of the obturator foramen. Illustration B reviews acetabular screw placement and the structures at risk in each quadrant.

     

    Incorrect Answers:

    Answer 1: This retractor places the sciatic nerve at risk.

    Answer 2: This screw places the sciatic nerve and superior gluteal artery at risk. Although injury to these structures is possible, this zone is considered "safe."

    Answer 4: This screw places the external iliac artery and vein at risk, and is considered the "danger" zone.

    Answer 5: Anterior retractor placement with sharp retractors place the external iliac and femoral arteries at risk.

     

     

     

     

     

     

     

     

    OrthoCash 2020

     

  160. A 65-year-old male sustains a fall onto his left hip 3 years after a total hip arthroplasty. A radiograph taken at the emergency room is shown in Figure A. What is the next best step?

 

 

 

  1. Open reduction and internal fixation with locked plates and cables through an extensile approach

  2. Revision with a proximally porous-coated stem

  3. Revision with an extensively porous-coated stem

  4. Nonoperative management

  5. Minimally invasive plate osteosynthesis Corrent answer: 3

The patient has a Vancouver B2 periprosthetic fracture. There is a loose stem that should be treated with revision to an extensively coated stem that bypasses the fracture site.

 

Revision of the femoral component is recommended for Vancouver B2 and B3 periprosthetic fractures. Type B1 fractures are treated with ORIF and stem retention, and proximally deficient B3 fractures may be treated with alloprosthetic composites or tumor prostheses.

 

Springer et al. retrospectively reviewed 118 hips with Vancouver B fractures. Seventy-seven percent of 30 extensively coated stems, 60% of 42 cemented stems, 36% of 28 proximally coated stems, and 61% of 18 tumor prosthesis/allo-prosthetic composite stems were well fixed and demonstrated

fracture union. Nonunion and loosening were the most common complications. They recommend extensively porous-coated stems for better results.

 

Haidukewych et al. review revision of periprosthetic fractures. They found that most acetabular components are well fixed. When the distal fragment has parallel endosteal cortices with >=5 cm of tubular diaphysis (usually with a diameter of <18 mm), they recommend an extensively coated, uncemented, monoblock long-stemmed prosthesis. If the distal diaphysis is divergent, has

<5 cm of parallel endosteal cortex, or large endosteal diameters, a fluted, grit-blasted, titanium, tapered modular stem can be used.

 

Figure A shows Vancouver B2 fracture. The stem has subsided relative to the proximal fracture fragment, indicating that it is loose. Illustration A shows revision of the same fracture with an extensively porous-coated stem.

Illustration B shows the endoskeleton technique using an osteotomy to split the proximal fragment coronally for stem removal, followed by insertion of a modular, fluted, tapered stem and cerclage fixation of the proximal fragments.

 

Incorrect Answers:

Answers 1 and 5: This is appropriate for Type B1 fractures.

Answer 2: It is not enough to achieve fixation in the proximal fragment. Loosening and nonunion rates are higher with this type of revision compared with an extensively-coated stem.

Answers 4: Nonoperative management is not appropriate in the setting of a loose femoral component.